Jump to content

Wikipedia:Reference desk/Science

From Wikipedia, the free encyclopedia

This is an old revision of this page, as edited by 79.122.75.250 (talk) at 14:28, 18 November 2008 (→‎is Jesus's 100% man/100% God dualty like quantum wave/particle dualty?: new section). The present address (URL) is a permanent link to this revision, which may differ significantly from the current revision.

Welcome to the science section
of the Wikipedia reference desk.
Select a section:
Want a faster answer?

Main page: Help searching Wikipedia

   

How can I get my question answered?

  • Select the section of the desk that best fits the general topic of your question (see the navigation column to the right).
  • Post your question to only one section, providing a short header that gives the topic of your question.
  • Type '~~~~' (that is, four tilde characters) at the end – this signs and dates your contribution so we know who wrote what and when.
  • Don't post personal contact information – it will be removed. Any answers will be provided here.
  • Please be as specific as possible, and include all relevant context – the usefulness of answers may depend on the context.
  • Note:
    • We don't answer (and may remove) questions that require medical diagnosis or legal advice.
    • We don't answer requests for opinions, predictions or debate.
    • We don't do your homework for you, though we'll help you past the stuck point.
    • We don't conduct original research or provide a free source of ideas, but we'll help you find information you need.



How do I answer a question?

Main page: Wikipedia:Reference desk/Guidelines

  • The best answers address the question directly, and back up facts with wikilinks and links to sources. Do not edit others' comments and do not give any medical or legal advice.
See also:


November 12

What shape is this?

I was thinking about this some time ago. Imagine a small room that has a door on each of its six faces. On the center is a flower pot and some clothes that I discarded. If I open and enter any of the doors in this room, I will enter a room with the flower pot and the same clothes that I discarded earlier. My question is, what is the shape of the room? I'm pretty sure it is not 3 dimensional. --Lenticel (talk) 00:37, 12 November 2008 (UTC)[reply]

I think you have been trapped in a game. In many old 2D games when you went off one side of the screen you reappeared at the other side. Obviously they have upgraded your game to ake advantage of a graphics processor to do 3D. On the other hand you might like to read the bit at the end of Shape of the Universe,we may be in something far weirder. 01:10, 12 November 2008 (UTC)
A room with 4 walls, a floor, and a ceiling could be considered as a cube with six faces. CBHA (talk) 02:43, 12 November 2008 (UTC)[reply]
This reminds me of a hemicube. Anyway, I'd say either the room is three dimensional, but the geometry isn't Euclidean, or you're on a three-dimensional analogue of a torus. That is, the surface is three-dimensional. — DanielLC 01:22, 12 November 2008 (UTC)[reply]
Topologically speaking, if the room has six walls in addition to floor and ceiling, the floor of the room could be a sphere, a torus, or a projective plane, for example. If it has four walls + floor and ceiling, then the room could be a three-torus or any number of weirder manifolds. Algebraist 01:31, 12 November 2008 (UTC)[reply]
Wow, this weirder than I thought. --Lenticel (talk) 01:37, 12 November 2008 (UTC)[reply]
We could narrow it down further if we knew whether or not going through any of the doors changes you from being right-handed to left-handed, or vice versa (or, equivalently, makes everything else in the room turn into its mirror image, but changing you sounds more fun!). --Tango (talk) 01:46, 12 November 2008 (UTC)[reply]
We also want to know whether going through a door changes your "up". —Tamfang (talk) 07:56, 14 November 2008 (UTC)[reply]
I wonder whether you got this idea from Cube 2: Hypercube which has similarities (but limited logic). PrimeHunter (talk) 01:56, 12 November 2008 (UTC)[reply]
Well I got the idea while I was taking a bath earlier today. The cramp bathroom inspired the idea. By the way, the movie seems to be cool. Maybe I'll try to find a DVD version later.--Lenticel (talk) 02:18, 12 November 2008 (UTC)[reply]
There are ten finite Euclidean 3-manifolds.[1]Tamfang (talk) 05:25, 2 September 2023 (UTC)[reply]

If the answer is "not 3 dimensional", then the word you're looking for is "Hexagon". APL (talk) 14:03, 12 November 2008 (UTC)[reply]

It's not a tesseract is it? It's a non-3-dimensional shape which is connected in ways I don't understand. AlmostReadytoFly (talk) 16:35, 12 November 2008 (UTC)[reply]
A tesseract is 4D, I don't think we have any reason to believe we're working in 4 dimensions. --Tango (talk) 16:42, 12 November 2008 (UTC)[reply]

Potential Energy

For the sake of simplicity, let's consider gravity. Now I understand why the gravitational potential energy of an object has to be negative (as it approaches the source of gravity its potential energy must decrease in order to allow its kinetic energy to increase), but isn't energy the ability of an object to do work? And so it seems odd that an object could have negative energy (when its, say, lying on a table). How would one resolve this apparent contradiction? Thanks. —Preceding unsigned comment added by 76.69.241.185 (talk) 00:46, 12 November 2008 (UTC)[reply]

It doesn't have to be negative, you can set the zero of potential energy to be anywhere you like. The maths is often easier if you set the zero to be at infinite distance and thus have it always negative, but it doesn't really matter. All that matters is the change in energy. For an object on a table it would probably make sense to set the zero to be when the object is on the ground, since that's the lowest it can get. If you do that, then when it's on the table it has positive energy and can do work as you would expect. --Tango (talk) 01:04, 12 November 2008 (UTC)[reply]
For an object A in a gravitational field of another object B, it makes sense to define a zero of potential energy at very large (infinite) distance between the objects, and to measure the velocity relative to the center of mass (CM) of the system of the two objects. Then, if the TOTAL energy of A is negative, then it is on the bound orbit around CM ; and if the TOTAL energy of A is positive, then it is on the open orbit. Gravitational potential energy of A is always negative. If A performs work, its total energy decreases, but its instant potential and kinetic energy may either increase or decrease, provided their SUM (the total energy) decreases and provided the potential energy stays negative. Now, for the AVERAGE potential and kinetic energies of A in the BOUND orbit, you can apply a so-called virial theorem. For a classical gravitating system, it states that AVERAGE kinetic energy Ekin equals minus half the AVERAGE potential energy, Ekin = -Epot/2 ; or, equivalently, Etot = Ekin + Epot = -Ekin = Epot/2 < 0 . In a two-body system the AVERAGE is determined over a closed orbit; in a many-body gravitational system the strict average is over an infinite time, but for a "very large" finite time the result holds with high accuracy, too. Anyway, since E = Epot/2, body A performing work DE > 0 changes E to E - DE, so makes Epot "more negative" (thta is, larger in its absolute value). That means that Ekin also becomes larger in its absolute value! What happens is, the body A, having lost some energy, moves to a tighter orbit where its total energy is indeed lower (larger in absolute value, and negative) but its mean-square speed is higher. Hope this helps. --Dr Dima (talk) 01:47, 12 November 2008 (UTC)[reply]

Camera red eye and white eye

I've noticed that my new camera often causes people to have one red pupil and one white pupil instead of the standard camera red eye. I thought that the red eye was caused by reflection of the red on the back of the eye. So, that doesn't explain how the white eye effect is produced. It never happened with any of my older cameras. So, I am left wondering if it is the camera. Has anyone else here had a camera do this? Is there a fix to make it stop doing it? It is a pain because the auto-removal of red eye doesn't work if the pupil is white. -- kainaw 01:41, 12 November 2008 (UTC)[reply]

Could it be some weird anti-red eye feature going horribly wrong? Try turning off any weird anti-red eye features and see if it stops happening. --Tango (talk) 01:50, 12 November 2008 (UTC)[reply]
Also, have you tried it with different subjects? Maybe the person you are photographing has a strange eye (in which case, they should probably see a doctor!). --Tango (talk) 01:51, 12 November 2008 (UTC)[reply]
I have noticed it with more than one person. About 90% of the times I've seen are with my son. But, about 99% of the photos I take are of him. So, there is no reason to think he is more prone to it. I've tried to turn off all weird functions on the camera. It isn't easy to use because it is designed to be so easy to use that it decides what you want and does it for you - refusing to let you decide what you want. Something I did got it to stop blinking the flash 4-5 times before snapping a photo - which often meant that the cool photo of my son would end up being him covering his eyes and turning away from the evil flashing light. -- kainaw 02:08, 12 November 2008 (UTC)[reply]
This site has any explanation of camera red eye and one reason for white eyes: [2] Rmhermen (talk) 02:11, 12 November 2008 (UTC)[reply]
Thanks. It commented on the reflection being common when the pupil is angled away from the camera, towards the nose. Looking back at the photos I didn't manually correct, I can see that the people with one white eye are looking away from the camera and towards their nose. -- kainaw 02:40, 12 November 2008 (UTC)[reply]
I suppose it's worth asking whether your son is cross-eyed? If the two eyes were not pointing in the same direction - then the reflection off the retina of one, would not occur on the other eye. It's hard to imagine any other reason. SteveBaker (talk) 04:41, 12 November 2008 (UTC)[reply]
It's also possible if the camera is close to the person's face, but they're looking at something well behind the camera. I don't know how close the camera would need to be for this. --67.185.190.46 (talk) 06:36, 12 November 2008 (UTC)[reply]
My understanding is that if the eye is looking straight at the camera the flash will produce a reflection of the vascular, and thus red, back of the retina. However if the eye is not directly aligned with the camera the reflection is of the less vascular parts of the retina producing a white reflection. I believe the phenomenon of 'red-eye' is a human problem associated with the particular vascular anatomy of the human retina. Perhaps it occurs in other primates. Richard Avery (talk) 08:13, 12 November 2008 (UTC)[reply]
Since this is occurring with photos of several people, a medical cause is unlikely. However retinoblastoma can give this appearance. Axl ¤ [Talk] 12:05, 12 November 2008 (UTC)[reply]
Now that I had some search terms for Google (thanks), I understand. This happens with children and isn't very uncommon. If a child is looking away from the camera (by about 15 degrees), the eye that is looking towards the nose will turn white. The other will be dim red - not a complete red-eye, but not white either. If you look at eye, you can see the main optic nerve is not dead center in the back of the eye. It is about 15 degrees off center. So, if the child is looking away at just the right angle, you get reflection off the optic nerve, not the nice red part of the eye. Further explanation that I've found from tons of Google hits is that this is limited to children because the size of the whitish area around the optic nerve is larger in relation to the size of the eye than it is in adults. Also, adults tend to look at the camera, not slightly away - so they don't even get in position for this sort of reflection. Since I've been mainly photographing my 1-year old son and his friends, I've been picking up a lot of kids at just the right angle to get the white-eye reflection. -- kainaw 13:41, 12 November 2008 (UTC)[reply]

Light Has Intelligence?

I saw a "documentary" recently in which a "scientist" stated that light has shown to anticipate experiments in which it is involved. The guy did not seem like a crack pot. What in the world could he be referring to? —Preceding unsigned comment added by 75.67.217.220 (talk) 01:52, 12 November 2008 (UTC)[reply]

Perhaps retrocausality? Jkasd 02:16, 12 November 2008 (UTC)[reply]
See also, freebasing and crack cocaine. Plasticup T/C 02:50, 12 November 2008 (UTC)[reply]
I suspect you're thinking of things like the "wave-particle duality" effect and quantum mechanics. Because of the wave–particle duality of light, the results of certain experiments seem to depend on how you are measuring the results - which leads one to kinda imagine that the light in the experiment "knew" how you were ultimately going to measure it - and changed it's behavior beforehand.
I recommend reading Double-slit experiment - and Quantum eraser experiment.
Suffice to say, the light isn't "intelligent" - but it does behave in ways that seem so far from our normal experience that it's almost impossible to get your head around the implications. This stuff leads us into the ideas of parallel universes and all manner of other weirdnesses.
SteveBaker (talk) 04:37, 12 November 2008 (UTC)[reply]
I'd guess the Elitzur-Vaidman bomb-tester is closest to what he might have been thinking of Dmcq (talk) 09:04, 12 November 2008 (UTC)[reply]
The deal with light is that it is not a particle or a wave, it is its own thing. Its just "light". We have models that treat it like a particle; and those models work to explain some of the behaviors of light. We also have models that treat it like a wave; and those models work to explain some of the behaviors of light. The deal is, that both models fall apart roughly 50% of the time; that is for any set of conditions in which light behaves as a particle, the wave model looks like it doesn't work, and visa-versa. The reality of light is that it doesn't have an analog in the "big" world. You can't say light is like _blank_, where _blank_ is any thing you have the ability to manipulate with your hands. Any "contradictions" in lights behavior, which make it, for example, appear "intelligent" as described (such as the aforementioned Double-slit experiment), are just the result of the faultiness of our models. Its our problem, not light's... --Jayron32.talk.contribs 17:24, 12 November 2008 (UTC)[reply]
The trouble is that this is simultaneously a correct - but also intensely frustrating - answer! You're absolutely right - light is something for which we have no analogs - which leaves us no way to explain it that does not tempt us into dangerous extrapolation-from-experience. Ordinary 'stuff' has a mass and you can stop it moving. Then you can stick it on a weighing machine and you know it's mass. When you push ordinary stuff towards light-speed, it gets heavier and heavier - and if you could somehow get it up to light speed, it's mass would be infinite. So the mass of normal 'stuff' is infinite at the speed of light and something reasonable all the rest of the time. Photons, however can only move at the speed of light - but yet they have actual sane, finite masses. Time is similarly warped - so for the photon, time doesn't exist...neither does distance. It's no wonder that it 'misbehaves' in every way imaginable compared to normal 'stuff'. The very nature of the zeroes and infinities that pop up at the speed of light in relativistic calculations pretty much guarantees that light is going to be weird. SteveBaker (talk) 22:05, 12 November 2008 (UTC)[reply]

How come garbage explodes only when it's buried?

The old lady told me to take out the garbage earlier (always naggin') so I jokingly said I'll chuck it in the backyard. Then she said "You can't throw it in the backyard, ass-h*le, it'll draw rats!" Then I said, "Fine, I'll bury it! That way I wont have to pay a goddam garbage bill!" Many colorful obscenities later she told me that garbage will explode if buried for a short time. I didnt want to agree with her at the time, but I vaguely remember hearing that's why landfills are vented and have flames shootin out of factory whistle lookin thingamajigs. My neighbor 3 trailers down has had garbage all over his yard for years that wasnt buried and he hasnt had no problems. Not even rats. What gives?Sunburned Baby (talk) 02:28, 12 November 2008 (UTC)[reply]

Maybe it explodes on Wednesdays while your friend is at work? —Preceding unsigned comment added by 83.199.126.76 (talk) 02:32, 12 November 2008 (UTC)[reply]
Much of our garbage tends to produce methane when it decomposes. When sitting in the open, the gas just floats away. When buried, it builds up in pockets. If, for some reason, the pocket of gas is ignited, it can make a small explosion. Venting natural gas from landfills isn't done just for safety. There are people who buy natural gas, so the landfill companies bottle up the byproduct for profit. So, why not bury your garbage, stick a hose down in it, and funnel off the gas into some empty wine boxes you have laying around. You might make enough to pay for a whole week's worth of lottery tickets. -- kainaw 03:06, 12 November 2008 (UTC)[reply]
The "some reason" for the gas igniting Kainaw mentioned is often an exothermic reaction in the decomposition process. I tried to find a nice link for you. Although several sites including our Composting page mention the fact that decomposition by anerobic bacteria generates heat, I've failed to find a nice concise description of the process. Anaerobic respiration holds some clues, but not much context. What you get is spontaneous combustion which has a dead end link to Fermentation (biochemistry). Don't get you hopes up for not getting rats when you bury the stuff. I used to live next to a former dump and the critters had no problem burrowing to the larder. You'd just get a chance to ignore what you don't see. Your neighbor who didn't get any might have had a local ecosystem (e.g. snakes, cats, owls etc.) that kept the rat population down to an unnoticeable/non existent size. The amount of biogas you get out of your household waste is not likely to reach marketable amounts unless you wish to build a Mount Trashmore in your back yard. The volume of easily digestible biomass in proportion to stuff that takes decades to break down is likely not very high. To reduce the volume of trash you put out you could deliver glass, plastics, metals and paper to a local recycling center and pepper your back yard with compost barrels (no access for rats) to use the compostable portion of your biodegradable trash. Depending on gas prices and local garbage collection policies and prices you might come out ahead or not. 76.97.245.5 (talk) 07:17, 12 November 2008 (UTC)[reply]

This begs the question of why Mount Trashmore doesn't explode? Let's see those skaters jump that one!Sunburned Baby (talk) 03:02, 16 November 2008 (UTC)[reply]

Methane as cause of global warming

Im looking for references that Methane in the atmosphere causes much more global warming than co2 and therefore that burning natural gas is good for the earth.--GreenSpigot (talk) 02:58, 12 November 2008 (UTC)[reply]

Woooaahh...be very VERY careful how you say that!!
Our Methane article does indeed say (correctly) that "Methane is a relatively potent greenhouse gas with a high global warming potential of 72 (averaged over 20 years) or 25 (averaged over 100 years)." (and there is a reference for that). CO2 has a 'global warming potential' of 1 - so clearly Methane in the atmosphere is a REALLY bad thing.
HOWEVER it is most certainly NOT a good idea to drill for natural gas and burn it in order to save the planet! It's far better to leave it underground where it belongs! Methane deep underground where it's been safely buried for millions of years - is just fine where it is! The only (exceedingly special) time when what you say is correct is if the natural gas is already in the atmosphere - then converting it to CO2 is better than waiting for many decades for it to degrade by itself.
So please don't go around saying "we should all be using natural gas because it's good for the atmosphere" - because nothing could be further from the truth! The kinds of situation we're talking about is when (for example) you have a herd of dairy cows and their poop would normally produce a bunch of methane that would do terrible things to the planet - then it is much better to use that methane as a fuel (both extracting some useful energy - and converting it into much safer CO2 in the process).
SteveBaker (talk) 04:22, 12 November 2008 (UTC)[reply]
Methane is more potent per molecule; however, in absolute terms we've added much more CO2 to the atmosphere than methane, so methane has had less of a global warming impact than co2. Dragons flight (talk) 05:53, 12 November 2008 (UTC)[reply]
Yes - I agree. This may change as the methane clathrate deposits in the deep oceans begin to melt. There has been some evidence that this exceedingly nasty situation is starting to occur - and (reluctantly) one has to say that burning the stuff as fuel (converting it to CO2) before it can ramp up the greenhouse effect (with the potency of methane) might be a last-ditch way to survive that situation. However, it's a decidedly "non-trivial" problem to do that - and we'd want to be very sure that the runaway melting of these deposits was really going to happen before we took such a drastic step. SteveBaker (talk) 14:57, 12 November 2008 (UTC)[reply]

Full moon on birthdays

Over the course of a lifetime of, say, 85 years, how often could one expect their birthday to coincide with the full moon? -- JackofOz (talk) 05:34, 12 November 2008 (UTC)[reply]

Three times on average. Dragons flight (talk) 05:54, 12 November 2008 (UTC)[reply]
Thanks. How did you work that out? -- JackofOz (talk) 07:29, 12 November 2008 (UTC)[reply]
A full moon occurs once every 29.5 days according to its article, so a birthday has one chance in 29.5 of coinciding with one. 85 is almost three times 29.5. (Something for werewolves to look forward to?) Clarityfiend (talk) 07:58, 12 November 2008 (UTC)[reply]
Yep, that's what I originally figured. It just felt way too low, so I thought I must have gone wrong somewhere. Depending on where the birthday falls, you might only manage 2 birthday full moons in an 85-year lifetime. Is there any way to work out, for any particular birthday and a given starting year, when the birthday full moons will occur, or do you have to use an ephemeris and do it "manually"? -- JackofOz (talk) 08:22, 12 November 2008 (UTC)[reply]
It probably sounds low because the moon changes size quite slowly and appears full to the naked eye (at least mine) for several nights a month. Algebraist 08:25, 12 November 2008 (UTC)[reply]
You probably would need an ephemeris; the 29.5 days varies a bit. However, there may be astrological software that can tell you when a solar return (birthday) coincides with a sun-moon opposition (full moon). You would also need to know the place of birth so that you could reduce the time to UT, the time used in most ephemerides.--Shantavira|feed me 08:43, 12 November 2008 (UTC)[reply]

Pneumonia

What is the survival rate for untreated bacterial pneumonia if the victim is an otherwise-healthy young adult? --67.185.190.46 (talk) 06:28, 12 November 2008 (UTC)[reply]

Untreated, bacterial pneumonia kills ~30% of the afflicted [3]. Dragons flight (talk) 06:45, 12 November 2008 (UTC)[reply]
But it's occurrence and observation is, I assume, much higher in the sick and elderly. Being in good physical condition and having a robust immune system bacterial pneumonia may manifest itself as only a cough and I may not even go to the doctor. Any measurements (short of those from controlled experiments) is going to suffer monstrous selection bias. Plasticup T/C 15:05, 12 November 2008 (UTC)[reply]
Pneumonia describes an illness that necessarily has certain symptoms, i.e. fluid in the lungs, almost always accompanied by fever and some degree of difficulty breathing. If you only have a cough, then you might have an infection, but you do not have pneumonia. Dragons flight (talk) 16:22, 12 November 2008 (UTC)[reply]
It's certainly possible that a person whose only symptom is a cough has pneumonia. It's especially possible in the elderly or others with immune dysfunction, but not what you'd expect in the young: I think Darongs flight's point is that those with vigorous immune responses fight off infection before it becomes pneumonia. Since the question asks specifically about the young, we can't give a reasonable answer if the population studied includes the elderly and we don't have the data to correct for age. 30% is certainly high for a young population. - Nunh-huh 17:26, 12 November 2008 (UTC)[reply]
Lots of folks (especially kids) get so called walking pneumonia, otherwise identified as atypical pneumonia, often with mycoplasma as the causative agent. As a result of the slow growth rate of this organism, a generally healthy patient can exhibit nothing more than a nagging cough and certain degree of lethargy as they go about their daily business, which doesn't clear up after a long period of time. Most parents I know (who are reasonably frequent consumers of healthcare) are quite familiar with it. I'm not sure how it resolves itself if untreated, what I see is parents who see those symptoms in their kid for a couple of weeks, go to the doc and get the diagnosis, and get it treated with specific antibiotics. But if you're the type of person who avoids doctors unless absolutely necessary, for whatever reason, you probably wouldn't be worried enough to seek attention.Gzuckier (talk) 17:45, 12 November 2008 (UTC)[reply]

Query about today's featured picture.

What a fantastic picture, but that apart can anyone explain the blue/green light circle at the rotor tips. I understand that it is caused by lights but what is the purpose of the lights, surely they are not decorative. Are they part of some safety system that shows the position of the rotors at night to prevent..um.. unpleasant accidents? Richard Avery (talk) 08:23, 12 November 2008 (UTC)[reply]

Link to the image. Just in case this will not be answered today.--Lenticel (talk) 09:32, 12 November 2008 (UTC)[reply]
That's definitely an amazing photograph! This pdf linked from the V-22 Osprey talk page mentions "upgrading rotor tip lights & formation lights for improved night vision goggle (NVG) compatibility" and this page also mentions that the "prop-rotors have lights in the rotor tips for night safety." --LarryMac | Talk 14:23, 12 November 2008 (UTC)[reply]
See these pictures 1 and 2 of the B-25 Mitchell, an American bomber during the Second World War: it's long been a common practice to have something light at the edges of blades, although as you can see, 60+ years ago they simply painted the propellor tips rather than having lights at the edge. Nyttend (talk) 14:42, 12 November 2008 (UTC)[reply]

Certainly it's so that the pilot can visualise where the rotor tips are - that's especially important in this aircraft because it's a tilt-rotor craft - those engine nacelles swivel through roughly 90 degrees between hovering and forward flight. When hovering in confined areas - it's really useful to be able to see that your rotor tip is just about to whack into a tree limb or a lighting pole or something. I strongly suspect that they only turn them on in those specific circumstances because military aircraft don't generally want to be seen at night. SteveBaker (talk) 14:50, 12 November 2008 (UTC)[reply]

Thanks guys and gals, kind of what I thought but you have collectively sourced and expressed it so well. Richard Avery (talk) 15:21, 12 November 2008 (UTC)[reply]

Mars

How long could an unprotected human survive on the surface of Mars? —Preceding unsigned comment added by 195.188.208.251 (talk) 12:35, 12 November 2008 (UTC)[reply]

According to drowning, lack of oxygen takes about six minutes to cause brain death, so at most that long. I think that'll kill you before the cold does, but I don't know for sure. Algebraist 12:57, 12 November 2008 (UTC)[reply]
I'm not sure drowning is a fair comparison.
In any case the article on Time of Useful Consciousness, says for altitudes over 50,000ft, you've only got 9 to 12 seconds before you lose the capacity for rational thought. The surface of Mars would be similar or worse. So, you'd want to get back inside pretty quickly. APL (talk) 13:59, 12 November 2008 (UTC)[reply]
The atmospheric pressure on the surface of Mars is a shade less than one percent of the air pressure at Earth's sea level. For the purposes of sustaining life, it's better described as a low-quality vacuum than an atmosphere. In that vein, you'll probably be interested in our articles on the Armstrong limit, Human adaptation to space#Unprotected effects, and space exposure. Briefly, you would expect somewhere in the neighbourhood of 10 seconds of useful consciousness. Based on animal experiments, if your friends dragged you into the airlock and repressurized you within about 90 seconds, you'd probably survive without permanent injury. Beyond that threshold, you're probably toast. (I would expect rapid, serious damage to the lungs.) TenOfAllTrades(talk) 15:53, 12 November 2008 (UTC)[reply]
What would cause that damage? If you survive the initial decompression (most importantly, don't try and hold your breath), the barotrauma from a difference of (effectively) one atmosphere shouldn't be too extreme. I think it's hypoxia that would kill you by damaging the brain (it important to note that hypoxia is worse in hard vacuum than when drowning - the oxygen is kind of sucked out of you rather than just being used up, hence 90 seconds rather than the 6 minutes Algebraist mentioned). Since the Martian atmosphere isn't much of a thermal conductor (there simply isn't enough of it), the cold would take longer to affect you so we can probably ignore it. --Tango (talk) 16:10, 12 November 2008 (UTC)[reply]
From Space exposure : "But severe symptoms such as loss of oxygen in tissue (anoxia) and multiplicative increase of body volume occur within 10 seconds, followed by circulatory failure and flaccid paralysis in about 30 seconds.[1] The lungs also collapse (atelectasis) in this process, but will continue to release water vapour leading to cooling and ice formation in the respiratory tract." APL (talk) 22:13, 12 November 2008 (UTC)[reply]

Leaves changing color at different rates

Here in Beaver Falls, Pennsylvania, most leaves have fallen, but there's a large maple with large leaves (I'm guessing Norway Maple) behind my house. Reading Autumn leaf color, I note that the article says that "Often the veins will still be green after the tissues between them have almost completely changed color." However, this tree is odd, with leaves that are more extreme than this: some of its leaves are totally yellow, while leaves farther up the branch are almost totally green still. Is it simply because these leaves are closer to the trunk, or because they're (most of the time) shaded by the leaves at the edge of the tree, or some other reason? And is this an unusual phenomenon, or is it common and I've really not been paying attention all my life? I checked last night, by the way: they really are on the same branch, so it's not as if there are leaves from two trees intermingled. Nyttend (talk) 14:36, 12 November 2008 (UTC)[reply]

Leaves change color because the sap is "running", as they say in the maple sugaring industry. Basically, all of the sap (i.e. sugary water) is drained out of the tree into storage in the roots. This makes sense because the roots are insulated by the ground, where as the leaves are basically big heat-dissipators, and so are VERY suceptible to freezing. The process is relatively slow; it can take several days to weeks to complete, depending on the size of the tree. For very large trees, the leaves at the periphery (i.e. farthest from the trunk) are "drained" first, so the leaves there die off earlier. For very large trees, the leaves at the tippy-top can have changed and fallen before those near the bottom and close to the trunk have even begun to start changing. --Jayron32.talk.contribs 17:37, 12 November 2008 (UTC)[reply]
Not sure where there's stuff about this but the colour in those leaves is put there to protect the leaf from sunlight whilst the tree absorbs back anything useful. Putting in that golden colour costs the tree. So the outermost leaves and those facing south will be coloured most. You might notice leaf buds are often brown or red too to protect them whilst developing. Dmcq (talk) 18:02, 12 November 2008 (UTC)[reply]

human fat

in Volume how does a pound of human fat compere to lard —Preceding unsigned comment added by Seanbaguley (talkcontribs) 15:30, 12 November 2008 (UTC)[reply]

Both are slightly more than 0.9g/c^3, a little less than water. Lard is slightly denser. SDY (talk) 15:35, 12 November 2008 (UTC)[reply]

length of Vibrio cholerae

What's the typical size, i.e. length of a cholera bacterium? --Ayacop (talk) 16:21, 12 November 2008 (UTC)[reply]

2-3 microns in length, 0.5 microns in width. - Nunh-huh 17:15, 12 November 2008 (UTC)[reply]
Could you add that to the article Vibrio cholerae? Rmhermen (talk) 23:18, 12 November 2008 (UTC)[reply]

yeast need salt?

In order for yeast dough to rise does it need to contain salt? RJFJR (talk) 17:04, 12 November 2008 (UTC)[reply]

No. See fermentation. 93.132.179.55 (talk) 17:14, 12 November 2008 (UTC)[reply]

Salt is used in baking to control the rate of fermentation, but is not strictly necessary for fermentation proper. However, "without salt, the yeast acts very rapidly and peters out too quickly. Too much salt will stunt yeast activity" [4]. "Salt controls yeast activity to achieve a slow, steady rise and it strengthens the dough structure; eliminating salt can result in a baked bread that has collapsed." [5] - Nunh-huh 17:17, 12 November 2008 (UTC)[reply]

Thank you! RJFJR (talk) 18:22, 12 November 2008 (UTC)[reply]
Just as an aside, bread made with no salt tastes terrible. Robinh 21:53, 12 November 2008 (UTC)
The Tuscans would probably disagree with you. (Classic Tuscan bread is made without salt.) -- 128.104.112.72 (talk) 23:32, 13 November 2008 (UTC)[reply]

Strength difference men-woman

What is the strength difference between men and woman?--Mr.K. (talk) 18:52, 12 November 2008 (UTC)[reply]

Estimates vary, particularly based on how you define "strength". this USMC report suggests the average woman's strength is 40%-70% of her male counterpart's. This report (with a higher average age) notes 50%, and then proceeds to note that males are also heavier and that strength-per-weight (if you elect to use that sort of "strength" definition) has a completely different result -- they find that males and females are effectively equal in terms of strength-per-muscle-mass. — Lomn 19:41, 12 November 2008 (UTC)[reply]
No surprise there, though, right? I don't think it's headline news to announce that folks with bigger muscles tend to be stronger... Matt Deres (talk) 17:29, 13 November 2008 (UTC)[reply]

Animal abuse

Wikipedia is preventing me from linking directly to the image I'm talking about. But on that site, encyclopedia dramatica, when you search "Animal Abuse", at the bottom you see an animated gif of a cat in some terrible device that shoots some liquid from the bottom into a chamber that the cat is in. What is that device that the poor cat is in? 98.221.85.188 (talk) 21:06, 12 November 2008 (UTC)[reply]

Encyclopedia dramatica is basically a joke site; I would not judge anything I read there too seriously. --Jayron32.talk.contribs 21:12, 12 November 2008 (UTC)[reply]
The Pet Spa [6]. Hydromassage, bath, and blowdry machine for dogs and cats. Edison (talk) 21:23, 12 November 2008 (UTC)[reply]
That's kind of a relief. But why was the cat so upset when it was in there? Was the water too hot? Was the machine being misused? 98.221.85.188 (talk) 22:04, 12 November 2008 (UTC)[reply]
Most house cats can't stand getting their fur wet. There are exceptions to this rule, of course, but I have a hard time imagining your average house cat enjoying a "hydromassage". APL (talk) 22:09, 12 November 2008 (UTC)[reply]

What's the problem with linking to encyclopediadramatica? —Preceding unsigned comment added by 83.199.126.76 (talk) 21:51, 12 November 2008 (UTC)[reply]

If I'm reading the spam black and whitelists right, you can't link anywhere on ED except the Main Page, because (a) it's a haven for anti-Wikipedia trolls, and (b) it's a haven for trolls in general, and has tended to be linked only to harass someone or to act as a shock site. I may be wrong, but I suspect Main Page was whitelisted only after the actual article on the site finally passed its 1000th deletion review. Confusing Manifestation(Say hi!) 22:33, 12 November 2008 (UTC)[reply]

Why is 350°F the standard for cooking food?

Almost every recipe I see wants to be cooked in an oven that is 350°F. Some call for temps up to 450°F. But I've never seen any recipe that calls for a temp outside of these extremes. (I'm sure there are exceptions) What's so special about this temperature range and cooking food? --70.167.58.6 (talk) 22:21, 12 November 2008 (UTC)[reply]

Related article or place to start deduction from: Temperature (meat). Mac Davis (talk) 22:30, 12 November 2008 (UTC)[reply]
Actually, that article doesn't help.
I think the temperature has more to do with the smoke point or flash point of fats and oils. Lard, for example, has a smoke point of about 370°F. Vegetable shortening smokes around 360°F. You don't want all the fat burning away while the food cooks.
Higher temperatures are useful for different cooking techniques, such as searing the outside of a piece of meat so that the inside continues to cook after it is removed from the oven. ~Amatulić (talk) 22:48, 12 November 2008 (UTC)[reply]
Note the temperatures in the Temperature (meat) article are the meat temperatures when cooked, not the oven temperatures.
When oven cooking foods of any significant thickness (such as cakes, pies, roasts of meat, puddings, large potatoes) using too low a temperature will not cook them through to the middle, or will take an unnecessarily long time to do so. Using too high a temperature is likely to result in the outside being dried out or burnt before the centre is cooked. The recommended temperatures are compromises between these two problems. CBHA (talk) 23:04, 12 November 2008 (UTC)[reply]
If you'd like to use different temperature settings, try a convection oven. NB Acrylamide is produced in increasing amounts at temperatures exceeding 350 F. The temperature settings of ovens are not a reliable indicator of actual temperatures inside (and those tend to vary significantly from one point in your oven to the other). You may find recipes for conventional ovens that tell you on what rack / distance from the heating element to place your food. To be sure your food is cooked properly use a meat thermometer. To make sure baked goods are done, the traditional method is to stick a knitting needle in and declare it done when no dough sticks to the needle anymore. (OR by my granny :-) 76.97.245.5 (talk) 00:26, 13 November 2008 (UTC)[reply]
Generally, 350 deg F is a temperature which is hot enough to encourage the Maillard reaction to occur (aka "browning"), without causing substantial burning to also occur. If you get up into the 500 deg F range, the food will char or burn (i.e. produce ash) which imparts a bitter and unpalatable flavor. Below probably 325-350 deg F, and there is likely not enough heat for the vital Maillard reactions to occur; it's these reactions that produce those brown tasty bits that form when meat is properly cooked. Its also why most slow-cooking methods, such as "crock-pot" cooking, require you to brown the meat seperately before adding to the crock pot. The crock pot gets hot enough to cook the meat (generally up to around 180-190) but never gets hot enough to brown it; which means you miss an entire flavor component of the food. --Jayron32.talk.contribs 03:15, 13 November 2008 (UTC)[reply]


November 13

anaerobic motor /propulsion

i would like to know whether an anaerobic motor /propulsion really exists. To me anaerobic refers to organisms. furthermore: the described process releases oxygen besides heat . can such a process/reaction called anaerobic? clear question: can an oxygen producing reaction be called anaerobic or what can it be caled instead?

finds in wikipedia: http://en.wikipedia.org/w/index.php?title=Special%3ASearch&search=%22anaerobic+propulsion%22&ns0=1&fulltext=Search

http://en.wikipedia.org/wiki/Ictineo#Ictineo_II

http://en.wikipedia.org/wiki/Ictineo_II

thanks--Stefanbcn (talk) 00:38, 13 November 2008 (UTC)[reply]

Anaerobic - means "not needing air" (basically) - so any motor that doesn't need air (like an electric motor - or a clockwork motor) is "anaerobic". A coal fired steam engine - or a gasoline powered car is "Aerobic"...it needs oxygen from the air to work. A motor that used some chemical process to produce oxygen and burned that to make motion would probably be called "anaerobic" too. SteveBaker (talk) 00:51, 13 November 2008 (UTC)[reply]

Artificial holographic sun

Using sulfur lamps and rotating mirrors, would it be possible to create a false window with a nice holographic landscape, with a completely realistic rectangle of direct false "sunlight" striking the floor/walls? (this is for people with just a brick wall to look at and no direct sunlight) and could mirrors make it plane parallel? —Preceding unsigned comment added by Trevor Loughlin (talkcontribs) 04:34, 13 November 2008 (UTC)[reply]

Well, it has been done - some really expensive flight simulators have used laser-projected displays that are bright enough to simulate natural sunlight at real-world candela values - and I've worked on them.
However, it's freakishly expensive and insanely dangerous. Remember that if you stare into the real sun - it's so bright that you will damage your eyes if you don't look away within a few seconds. Now consider a display device (a projection TV or something) that put out enough light to produce that same effect - plus enough to project all of the rest of the world at natural brightness levels too. In fact, it would need more energy even than that because it would have to scan over the scene at least 60 times a second - and the screen itself would not be perfectly, 100% reflective - so the display is putting out a lot more energy than the sun - within the small range of angles that covers the scene. If you think for a moment about the amount of heat that the sun puts out as it shines onto your skin - the display would probably have to chuck out that much heat too!
The result would have to be an amazingly powerful laser or something very similar. If you were to happen to catch a glimpse of the light from the laser itself (rather than it's dispersed, reflected image) - you'd be blinded instantly...no chance to blink or look away. When I worked with such a system a few years ago - everyone who entered the room when it was turned on had to go through a 30 hour laser safety course - the doors had to have automatic locking devices to stop people coming in when the laser was turned on - and there were all manner of handrails and such stopping people from going where the laser was operating. We nicknamed the gigantic water-cooled laser "The Death Ray of Ming the Merciless" because it looked exactly like something from the lair of a 1950's SciFi super-villain.
There is another issue here - which is for an outdoor scene to look completely real (ie not like a super-high-def TV screen) - the light has to appear such as to cause your eyes to focus at the correct depth. For an outdoor scene where nothing comes within (say) about 30 feet of the viewer, it's enough to 'collimate' the light so it appears to come from a source that's infinitely far away. This is tough to do. You either need a dome to project it onto that's at least 30' away - or a curved mirror such that the path from the laser projector to the eye is at least 30 feet - or you need some large, expensive glass lenses that are the size of your window to perform the same task. The difficulty with all of those things is that the scene only looks truly real when your head is at the "designed eyepoint" of the system - if you step a few feet to the side, the illusion is destroyed. It's kinda possible to correct for that - but the viewer needs to be wearing some kind of tracking device - and only one viewer gets a perfect view at a time.
So, yes - it's certainly possible - I've seen it done and the result is highly realistic and extremely compelling. But I don't think we'll see it happening as an entertainment device anytime soon.
SteveBaker (talk) 15:23, 13 November 2008 (UTC)[reply]


I don't mean to bring the topic back up, but what if replaced with a projector, it was an LED array. LED don't emit heat, just light I thought. 66.216.163.92 (talk) 23:37, 13 November 2008 (UTC)[reply]
They aren't really bright enough to create the effect of sunlight. Even packed tightly together...I don't think it would work. SteveBaker (talk) 16:04, 14 November 2008 (UTC)[reply]

Suphur lamps give the full spectrum of sunlight, LED's can't and have nowhere near the efficency. I have not heard of the laser device, it sounds fascinating. My idea would be have a pane of rapidly rotating tapered interleaved reflective spirals with a plasmonic projector changing the image on the turn of the spirals to create the hologram. That way your head would be torn off like putting it into an industrial dough mixer well before you managed to blind yourself looking into the projectors at the top and bottom of the machine. Another method would be projecting on to translucent hemispheres over a hole on a spinning washing machine cylinder. I will get this second hand, though I will have to be careful not to get too close to it.

A laser projector is actually three lasers, one red, one green and one blue - the light from them is combined into one colored beam with some mirrors and the resulting light is aimed into a pair of spinning hexagonal prisms - with mirrors on each of the 6 facets. One mirror spins horizontally at (say) 60/6 = 10 rotations per second - and that causes the lasers to scan vertically downwards and then jump back 60 times a second. The second mirror spins at 1000 times that speed(!) and causes the lasers to scan from left to right and then jump back one thousand times in every 1/60th second. Modulating the light from the lasers can be managed in a variety of ways. The result is that the laser scans a rectangular area exactly like the electron beam in a TV set and produces a very bright, full-color picture. Aiming that onto the inside of a large dome which is 'painted' with microscopic spherical glass beads results in a really compelling image of whatever TV/video picture you care to provide.
But as I said - such systems are huge, hideously costly, and vastly too dangerous to be "consumer" equipment. SteveBaker (talk) 16:04, 14 November 2008 (UTC)[reply]

Bird identification

Hello, I was at Taronga Zoo today and I took a photo of this little sparrow-ish thing. I would be very grateful if someone could help me with identifying its species. Thanks! 122.106.197.159 (talk) 04:43, 13 November 2008 (UTC)[reply]

That looks a bit like a silvereye to me. Tonyrex (talk) 06:02, 13 November 2008 (UTC)[reply]

rectilinear perspective

Do people actually see in rectilinear perspective, or is this just a convention of western art? For example, I've seen Looney Toons cartoons in which when viewing a tall rectangular building from the bottom, the lines start curving together at the top (rather than pointing straight at the vanishing point). --VectorField (talk) 06:11, 13 November 2008 (UTC) I guess an example of what this would look like can be found at fisheye lens. --VectorField (talk) 06:18, 13 November 2008 (UTC)[reply]

See also Perspective (visual) and for Cartesian rectilinear perspectives and variations on this see Perspective (graphical). In traditional Japanese and Chinese art, the perspective is constructed differently, more like parallel perspective. Julia Rossi (talk) 10:05, 13 November 2008 (UTC)[reply]
It's more than just convention. The idea is that the canvas should be like a window through which you can see the three-dimensional scene, and if you work that out in detail for a flat canvas (as described in Perspective (graphical)) you get the rules of perspective. It's the flatness of the canvas that matters, not anything going on inside the eye or brain. I don't think that people see in rectilinear perspective (I don't think the idea even makes sense—I think it's an instance of the homunculus fallacy) but the theory of perspective makes no such assumption. It does, however, assume that you stand in the correct location and have only one eye.
If you work out the rules for a cylindrical canvas instead, you get cylindrical perspective. (More precisely, if you follow the correct perspective rules for a cylindrical canvas with the viewer in the center and then unroll it, you get what's normally called cylindrical perspective.) The advantage of a cylindrical canvas is that you can get a wider field of view. A flat canvas is limited to a 180° FOV, and as you approach that limit the canvas size goes to infinity or the distance from canvas to viewer goes to zero, both of which are inconvenient. If you stand at the wrong distance from a large-FOV flat image (and any convenient distance will be wrong) it will look very distorted. A cylinder doesn't have that problem. Any small enough part of a cylinder is roughly flat, and so an unrolled cylindrical perspective is suitable for a very wide image that's meant to be looked at only a bit at a time (walking from side to side). A wide or tall background in a cel-animated TV show will normally be drawn in cylindrical perspective since it's designed to be panned over. -- BenRG (talk) 14:45, 13 November 2008 (UTC)[reply]


Oooh! Good question!
Perspective of some kind is a necessary part of any practical visual system. We really do see things that way. The different kinds of perspective come about through the shape of the 'screen' onto which the scene is projected - and the nature of the lens and how it gathers the light.
  • In an idealised 'pin-hole' camera, the film (or digital imaging device) is flat - and since light travels in straight lines through a notional zero-sized pin-hole at the front of the camera - and then onto the flat surface inside, all straight lines in the world outside project into straight lines in the image - and we have perfect rectilinear perspective.
  • In a practical camera - the lens isn't a pinhole and it has to bend the light to make it focus properly - that results in a non-linear mapping of real world onto the flat film plate - and depending on how much the light is bent, straight edges turn into curves and you no longer have rectilinear perspective.
  • In our eyes, the imaging surface (the retina) isn't flat - so straight lines in the real world (like the sides of a tall building) don't project into straight lines on the retina. However, our visual system isn't a matter of a bunch of pixels on the retina being "absorbed" somehow by the brain. It's MUCH more complicated than that. One of the things our visual system does is to compensate for those complicated curves so that we "see" straight lines where the lines are straight...we're not aware of the weird mapping that goes on because of a non-zero pupil diameter and a non-flat retina. We are aware at some level that things seem "smaller" the further they are away - but we're also unconsciously correcting for that - so we don't think that a car is tiny because it's further away. The mathematical fact of perspective has been converted by our visual system into something that takes on different meanings depending on the context about which we are considering them. There are several optical illusions that play on that to demonstrate that we don't "see" what is really there.
  • In art, the painting itself is generally flat - and artists generally want to give the impression that the rectangular frame of the painting is like a hole cut into the wall and the art is like an image coming through that hole - which is then percieved by our eyes. So then the mathematically "correct" thing for the artist to do is to pretend that the painting is a pinhole camera image and use rectilinear perspective - and then let our eyes process that image as if it were really light coming through a hole in the wall. Because that image then enters the eye in the same way that the light from a real object would - and the painting "looks real" to us (well, not quite because of issues of depth of focus and "collimation" of the resulting light).
  • In 3D computer graphics, (my speciality - I'm a graphics programmer for the games industry) no matter how wide-angle the "virtual camera" becomes - since the screen is flat - and we use rectilinear perspective because the math is simplest that way. Especially - we want to map straight lines onto straight lines - because our graphics algorithms are much simpler that way. And (fortunately) it all looks right for the same reason that art looks right. But we do see "fish-eye distortion" in computer graphics - and this is true even though the computer is translating straight lines into straight lines (actually, the graphics hardware is incapable of directly drawing curved edges - rectilinear perspective is built into the 3D circuitry at a fundamental level!). We perceive wide field-of-view images as distorted (and some people will even go so far as to claim that everything is curved even when that is a physical/electronic impossiblity!) The reason for that is that we are taking an image from a 'virtual camera' with a wide field of view (say 120 degrees) and presenting it on a screen that only subtends (perhaps) 30 degrees at your eye. This is not a natural thing - and our brains have to somehow interpret this as if the image were being seen through a distorting "fish eye" lens when all it's really seeing is through an idealised pin-hole camera. Our mental compensation for that imagined distortion (which is not present in the mathematically "correct" rectilinear perspective) results in a strong impression of curvature.
A similar problem occurs with narrow-angle images (eg taken with a telephoto lens) where the relative lack of perspective foreshortening leaves us with a wrecked sense of distance. Film makers love to use this. In a romantic scene with the moon in the background, they'll pull their camera WAY back from the actors - then zoom WAY into them - and the resulting screwup of our mental idea of perspective makes the moon look HUGE. This is used in action shots too - when the hero is running away from the burning car - which explodes behind him hurling him towards the camera...you place the camera WAY back from the car - put the actor fairly close to the camera and a VERY safe distance from the car...then the telephoto lens screws up the perspective for us - and we think the car is really close. It's still very close to being strictly rectilinear perspective - but our brain's inability to compensate for the distortion that results from the mismatch of the field of view means that we see things in way that they really are not.
SteveBaker (talk) 15:03, 13 November 2008 (UTC)[reply]
I think it's interesting that how we see things is generally governed by how we expect to see them. We know what size a car is, so we see it as being that size regardless of how far away it is (to use your example). However, if you're in an unusual situation, for example seeing cars on a road when looking out of the window of a plane, your brain doesn't really know what to expect and you need to consciously work out what you're seeing and you do notice the fact that cars are all so tiny (the standard cliché is to compare them to ants). --Tango (talk) 15:33, 13 November 2008 (UTC)[reply]
I'm going to take issue with the idea that our brains "compensate" for the curved shape of the retina. There's no one in there looking at the image projected on the retina. All the brain gets is a bunch of electrical signals down the optic nerve. If the retina were flat and the distribution of cones and rods were tweaked accordingly (to keep the visual acuity fixed), you'd get exactly the same collection of signals going to the brain (except in slightly different focus). I don't think the brain "knows" the shape of the retina—it doesn't need to know.
To put it another way, once you've projected onto one canvas you can re-project onto any other without needing the original scene as a reference. Fisheye Quake and PanQuake take advantage of that to produce realtime 3D in a mathematically accurate fisheye/cylindrical projection, by first rendering to a cube map and then rearranging the pixels. You could do the same thing in hardware these days (using one big rectangle and a pixel shader for the second pass). So, although modern graphics cards aren't designed for non-rectilinear projections, they can certainly produce them, probably at full frame rate, and it would be a pretty neat gimmick to have in a new game. So get cracking, Steve. :-) -- BenRG (talk) 21:22, 13 November 2008 (UTC)[reply]
To put this simply, our eyes use elliptic geometry and a flat canvas uses Euclidean geometry. Trying to put the same picture on both will require some warping. — DanielLC 15:56, 13 November 2008 (UTC)[reply]
The extent to which rectilinear perspective correctly replicates a 3D view, at least for a fixed observer and small field of view, can be judged by the success of trompe-l'oeil illusions. Gandalf61 (talk) 16:02, 13 November 2008 (UTC)[reply]
Recent coverage of drawing on curved canvases here. —Preceding unsigned comment added by 190.220.104.35 (talk) 19:21, 13 November 2008 (UTC)[reply]


Some painters used "Chinese perspective" where things are higher in the picture to indicate distance,or which are more axonometric and others used "landscape perspective" where distant objects are in more subdued hues, without the mathematical device of our modern geometric perspective. It is a modern mathematical invention. Edison (talk) 19:54, 13 November 2008 (UTC)[reply]

And, of course, painters like Picasso don't bother much about the rules of perspective at all. That's how it is with geniuses - they break the rules and sometimes make new ones. —Preceding unsigned comment added by 98.17.34.3 (talk) 01:10, 14 November 2008 (UTC)[reply]

Ceiling fan

Hello. If a Ceiling fan is switched on at the wall but it is actually off then does it still use electricity? Sorry if its a stupid question. ```` —Preceding unsigned comment added by AreDeeCue (talkcontribs) 13:49, 13 November 2008 (UTC)[reply]

No. The circuit has to be closed to use any noticeable amount of electricity (I'm ignoring cases such as worn out wires sparking against nails in the attic and such). Any switch anywhere in the line that opens the circuit will cause the flow of electricity to stop. -- kainaw 13:58, 13 November 2008 (UTC)[reply]
Your question implies that there are two (serial) switches to operate the fan? If this is the case, both switches must be on for the fan to work. Therefore the fan is not using electricity. Sometimes the wall-mounted control has an LED to indicate that it is on. However this LED uses a trivial amount of electricity. Axl ¤ [Talk] 14:01, 13 November 2008 (UTC)[reply]
The ceiling fans that use an infrared remote (to avoid the need to add house-wiring for houses that don't have a second switch and circuit) DO use a tiny amount of electricity when they're turned on at the wall but off using the remote. Some piece of circuitry has to be powered in order to pay attention to the InfraRed receiver. It's pretty tiny - but with a bajillion appliances around the home all eating small amounts like that - it does add up. The kind of ceiling fan that has a pull-cord to turn it on and off shouldn't be consuming any electricity at all when it's turned off there instead of at the wall. SteveBaker (talk) 14:26, 13 November 2008 (UTC)[reply]
The OP might also consider reading Standby power.--Lenticel (talk) 00:34, 14 November 2008 (UTC)[reply]

Echinoderm diversity

Estimations of the diversity of extants echinoderms vary widely, from 6,000 to 10,000 species, and even reliable sources differ a lot. Which is the most authoritative source on echinoderms, and what figure does it give? Thanks. Leptictidium (mt) 14:56, 13 November 2008 (UTC)[reply]

dangerous situation arising from killing germs?

If you kill all the germs in a place germs like to live, won't it just create natural evolutionary pressure toward germs that aren't affected by death? —Preceding unsigned comment added by 83.199.126.76 (talk) 15:22, 13 November 2008 (UTC)[reply]

Not being affected by death is tricky, but antibiotic resistance certainly arises in this way. Algebraist 15:47, 13 November 2008 (UTC)[reply]


Well - I don't think they can be "unaffected by death" - that's kinda silly! But I guess what you mean is that these critters are not killed by whatever means you've been using to kill off the others. If so - then yes! This happens all the time. In hospitals particularly, there are antibiotics everywhere - in the air and on all of the surfaces. Bacteria are mostly killed by this stuff being everywhere - but one in a trillion (maybe) survives - and this causes an evolutionary effect that results in types of bacteria that are immune to all of the common antibiotics. It is therefore necessary for the drug companies to continually come up with new antibiotics that the bacteria has not yet been exposed to - and which can therefore kill them...until a few more years have gone by. Doctors also recognise this and they don't prescribe the newest and most powerful antibiotics until they know that the older (and by now, better-resisted) have failed. This is an attempt to keep the newer kinds of treatment in reserve for the most resistant bugs. It's not just 'germs' either - strains of rats and mice that are immune to the common kinds of rat and mouse poisons are also appearing. There is no doubt that evolution does this all the time. SteveBaker (talk) 16:52, 13 November 2008 (UTC)[reply]
Also, it depends on just how you kill them. For example, very few living things can survive a good autoclaving. Naturally, a hospital bug that could stand it would have an enormous evolutionary advantage — but that matters little if there are no bugs that would stand even a chance of doing that in the first place. Simple broad-spectrum disinfectants like hypochlorite or hydrogen peroxide are similar: some organisms do resist them better than other, but in sufficient concentrations and quantities they kill pretty much everything indiscriminately.
A common trait of such non-specific disinfection methods is that they can only be used on non-living objects, or at best can only be applied externally to e.g. localized areas of skin. That's because, if they were applied e.g. to the entire human body in concentrations sufficient to kill bacteria, they'd kill the human too. Antibiotics, on the hand, can be administered internally, because they specifically kill bacteria while not being toxic to humans. However, this very specificity also allows bacteria to develop resistance to them: since a useful antibiotic must target some biochemical feature specific to bacteria, rather than indiscriminately killing every living cell, it's usually possible for bacteria to develop mutations such that the specific features the antibiotic attacks are no longer present.
Mind you, it is possible for organisms to develop resistance to even non-specific poisons, at least to some extent, over sufficiently long timescales. For example, ethanol is toxic to most living cells at sufficient concentrations, but most species, including humans, have developed some degree of resistance to it, since it's so commonly found in nature e.g. as a product of fermentation. An even more striking example is free diatomic oxygen: to anaerobic organisms, it's as deadly as ozone or fluorine, but over the 3 billion years since the evolution of photosynthesis, most living things have developed elaborate mechanisms for tolerating and, eventually, even making use of it. Even so, oxidative stress remains a significant source of cellular damage in organisms, including humans, that are exposed to high concentrations of oxygen. —Ilmari Karonen (talk) 20:02, 15 November 2008 (UTC)[reply]

What do you call this camera trick?

OK, I'm not sure how to describe something visual in words, but I will try. In movies and TV shows, they have this camera trick where the object in the forefront (usually a person) stays stationary but the background somehow moves. It seems like some sort of trick of perspective. This technique is usually used to convey shock, something unexpected or when the person makes a sudden realization. Sorry I can't give a better description, but this technique is used enough that hopefully someone will know what this is named. 216.239.234.196 (talk) 15:32, 13 November 2008 (UTC)[reply]

Dolly zoom APL (talk) 15:45, 13 November 2008 (UTC)[reply]
Yep, that's it! Thank you. (Comment: So they actually move the camera? Wow.) 216.239.234.196 (talk) 15:58, 13 November 2008 (UTC)[reply]
I've always heard it called a "Hitchcock zoom" because he was the one who first popularized it. It requires zooming the camera either in or out and simultaneously moving the camera either backwards or forwards. Done correctly (which is tough), the person in the foreground stays at exactly the same size - but the background zooms in or out. It has the effect of separating the character from their surroundings - which Hitchcock used to great effect to get across emotional state and such. Adding 'rim lighting' or other inconsistent lighting for the character is another way to separate a character from the background that Hitchcock used effectively. SteveBaker (talk) 16:57, 13 November 2008 (UTC)[reply]
Specifically, Hitchcock developed it for the movie Vertigo to illustrate Jimmy Stewart's character's acrophobia. I think I remember reading in Truffaut's book on Hitchcock that he conceived it in connection with one of his earlier films but was then told it could not be done. --Anonymous, 23:03 UTC, November 13, 2008.
Yep - in an era before computer-controlled cameras and dollies it would have required great skill (and possibly MANY retakes!) to pull off the effect - the dolly would have been pushed by a bunch of guys while the cameraman adjusted the zoom...coordinating those actions to the required degree of accuracy would be nightmarish! You can see why some people would say it was impossible. These days, you could just as easily cheat by standing the actor in front of a blue-screen and zoom into the background with a separate camera - but we have had computer controlled cameras that could do it since about the time of the first StarWars movie. SteveBaker (talk) 15:51, 14 November 2008 (UTC)[reply]
You could do it with an all-mechanical system. One option would be a zoom lens with a custom-cut thread (rather than the standard helical thread) so that a constant-speed rotation of the zoom ring produces the needed zoom rate, at which point a pair of electric motors (one to move the camera around, and one to drive the zoom lens) will give you the effect you want. If you don't have electric motors, you could move the camera on a rack-and-pinion track with the rack driving the zoom lens. I'm sure there are a number of other ways to set up a mechanical linkage between the zoom and the camera motion. --Carnildo (talk) 22:07, 14 November 2008 (UTC)[reply]
Sure - I'm sure it could be done - but it wasn't. I believe they also had to coordinate refocussing the camera while zooming and dollying. SteveBaker (talk) 05:45, 15 November 2008 (UTC)[reply]

Energy drink ingredients

Do any ingredients in energy drinks other than sugar and caffeine have proven desirable short-term effects? NeonMerlin 15:43, 13 November 2008 (UTC)[reply]

You're probably best to just run through the list of ingredients in your favourite energy drink and check the Wikipedia articles. The major active ingredient in guarana is caffeine. Taurine has been shown to axiolytic effects in some animal studies, but no effect on human beings has been observed. The NIH has reported that supplements containing gingko biloba have no measurable benefit when taken as directed. The list goes on, but you can probably find what you're looking for by following the links from Red Bull, Rockstar, and the others. TenOfAllTrades(talk) 16:25, 13 November 2008 (UTC)[reply]

Visual acuity of hawk's eye: why/how?

Hi, is there an article that explains why the hawk's eye has such great visual acuity? I can't find any info on WP on such a fascinating subject. Kreachure (talk) 16:52, 13 November 2008 (UTC)[reply]

Maybe you should read the article you linked to, then. Matt Deres (talk) 17:41, 13 November 2008 (UTC)[reply]

Yeah, thanks, but I was looking for more information, like the one provided by the bird vision article which I just found. Kreachure (talk) 18:09, 13 November 2008 (UTC

I think visual acuity is provided by the number of cone cells in the fovea, and presumably by how other bits of the eye work. Hawks presumably have a higher percentage of cones in the fovea than other species. This is just complete assumption. —Cyclonenim (talk · contribs · email) 18:54, 13 November 2008 (UTC)[reply]
I understand that some hawks and falcons actually have a slight concavity in the retina which allows for greater focus on distant objects and more visual cells than there would normally be, although I can't remember the details. 137.151.174.128 (talk) 21:56, 13 November 2008 (UTC)[reply]

resistors

today i encountered with a resistor named 3k9,4k7,and 100R.can any body plz tell me what values of these resistance and what is the type of these resistors . -- 116.71.186.13 (talk) 17:36, 13 November 2008 (UTC)[reply]

3900 ohm, 4700 ohm and 100 ohm. It's a funny bit of notation that is standard in the industry, but makes sense easily enough. They're just bog standard resistors as far as I can tell from that information. 137.108.145.10 (talk) 18:56, 13 November 2008 (UTC)[reply]
Is there a ref for that notation? It seems a bit uninformative, lacking an indicator of the precision rating like the color coding on resistors has. Edison (talk) 19:49, 13 November 2008 (UTC)[reply]
It is mentioned in the Resistor article briefly and also at Electronic color code#Other schemes. The notation used seems to follow BS 1852. Nanonic (talk) 19:56, 13 November 2008 (UTC)[reply]
BS 1852 now superseded by BS EN 60062. Latest version seems to be 2005.--GreenSpigot (talk) 13:09, 15 November 2008 (UTC)[reply]

Fructose Malabsorption

Disclaimer: I have no intend to ask for medical advice here. If you think you cannot answer without giving medical advice please just ignore my questions. If you feel obliged to tell me to see a doctor please give me name and address, too, as I have seen many doctors and none of them has even mentioned fructose malabsorption.

The one section of the fructose article (fructose malabsorption#Fructose Metabolism) states that fructose is absorbed using GLUT-2, the rest of the article states it is absorbed by GLUT-5. This seems to be a bit of a contradiction, or can GLUT-2 absorb fructose, too, but GLUT-5 is specialized on it?

It is not clear to me from the articles if fructose is normally (without f.m.) absorbed in the small intestine, the large intestine, or both, and if f.m. affects only the absorption in one of those or both. Fructose#Malabsorption gives the medical advice that Exercise can exacerbate these symptoms by decreasing transit time in the small intestine, resulting in a greater amount of fructose being emptied into the large intestine which I know is not true from my own experiences. Fructose not being absorbed and rotting in the large intestine as well as in the small would account for this experience.

Are ther many different forms of f.m. and is the absorption via GLUT-2/5 totally absent or only reduced?

Fructose is a small molecule, isn't it absorbed by pinocytosis or is it that the pinocytosed amount is too small? Thanks 93.132.177.64 (talk) 19:24, 13 November 2008 (UTC)[reply]

As our fructose article states, "The mechanism of fructose absorption in the small intestine is not completely understood. Some evidence suggests active transport, because fructose uptake has been shown to occur against a concentration gradient. However, the majority of research supports the claim that fructose absorption occurs on the mucosal membrane via facilitated transport involving GLUT5 transport proteins. Since the concentration of fructose is higher in the lumen, fructose is able to flow down a concentration gradient into the enterocytes, assisted by transport proteins. Fructose may be transported out of the enterocyte across the basolateral membrane by either GLUT2 or GLUT5, although the GLUT2 transporter has a greater capacity for transporting fructose and therefore the majority of fructose is transported out of the enterocyte through GLUT2." In case this isn't clear, it's essentially saying that fructose is primarily transported from the lumen of the small intestine into the cells lining it by GLUT5, and transported out of those same cells into the bloodstream by GLUT2. - Nunh-huh 19:43, 13 November 2008 (UTC)[reply]

Poisoning through the ear

(This is merely a question of curiosity with no intended "medical" application.) Is it actually possible to poison someone, as King Hamlet was killed, by pouring poison into their ear? If so, what poisons are effective that way and how, physiologically, does the poisoning take place? Was it a common belief in Shakespeare's time that someone could be poisoned through the ear, and would Shakespeare have known if this depiction of poisoning was accurate or not? 137.151.174.176 (talk) 20:33, 13 November 2008 (UTC)[reply]

There was a long discussion on this very topic on here last February. --140.247.249.93 (talk) 20:52, 13 November 2008 (UTC)[reply]
Thanks for the link. The interpretation that it's symbolic (poison in the ear representing evil words) instead of based on medical knowledge is quite plausible-- the result of the poison, in which the King suddenly becomes covered in leprous scabs, certainly doesn't seem like a literal description of something that could really happen. 137.151.174.128 (talk) 21:51, 13 November 2008 (UTC)[reply]
Also see eustacian tube. A good read on this topic. --Jayron32.talk.contribs 21:08, 13 November 2008 (UTC)[reply]
Though I agree with the symbolism interpretation discussed previously, I would also point out that perforated eardrums would have been much more common in Shakespeare's day than they are now. Someone with a perforated eardrum would be quite susceptible to poison poured into the ear, because it would pass right through the middle ear and into the throat. --Scray (talk) 03:31, 14 November 2008 (UTC)[reply]
I realize this is wandering off topic. Would perforated eardrums have been more common in Shakespeare's day than they are now because a) they could not be treated or b) eardrums were more often damaged or c) both? Thanks, CBHA (talk) 03:51, 14 November 2008 (UTC)[reply]
I'm not sure you;d need a perforated ear drum for the poison to work. The poison, after all, doesn't have to reach the digestive tract; just the blood stream. While your digestive tract provides a relatively easy way to reach the blood stream, other venues (the lungs, the nose, the eyes, and yes the ears themselves) may provide a relatively easy means by which to get the poison into the blood. Indeed, the poison may not have to even get past the ears; it could probably be absorbed directly through them straight to the blood stream... --Jayron32.talk.contribs 04:16, 14 November 2008 (UTC)[reply]
While there are poisons that can pass through intact skin (including external ear), much more permeable are the mucous membranes (like the linings of lungs, nose, eyes). A nightshade poison like the one employed in Hamlet would not cross intact stratified squamous epithelia (such as skin or intact external ear) efficiently, but would be absorbed through a mucous membrane (such as nasopharynx after passing through a perforated eardrum). @CBHA: Perforated eardrums are associated with untreated middle ear infections, which can result in spontaneous rupture and release of "pus under pressure". In addition, I don't know when European physicians started puncturing eardrums to provide relief, but my guess is that the practice has been around for a very long time. --Scray (talk) 04:47, 14 November 2008 (UTC)[reply]

spider-silk 'stronger' than steel. What does that mean?

I heard that spider-silk is stronger than steel. What exactly does that mean, and if it's true, why don't we spin spider-silk into threads, then strands, then long thick cables, coat them in something to keep the weather out and make ****in suspension bridges and ***t out of em... —Preceding unsigned comment added by 83.199.126.76 (talk) 20:33, 13 November 2008 (UTC)[reply]

It generally means that, pound for pound, spider silk has a higher tensile strength than steel. Its just that if you made steel threads as small as spider silk, the silk would be stronger, or alternately if you made cables of spider silk, those would be stronger than similarly weighted steel cables. However, getting enough spider silk together to build a suspension bridge is likely impossible. --Jayron32.talk.contribs 21:07, 13 November 2008 (UTC)[reply]
The fact we're not doing it (yet) is not for want of trying [7] [8]. As our page spider silk points out there are several different types of spider silk, all with their unique properties. At least for capture-spiral silk some of its resilience comes from lengths of silk that are coiled up in the glue drops and sort of act like giving a lassoed animal some rope when it pulls. The extra silk uncoils while the spider prey is decelerated when it gets caught; that absorbs some of the kinetic energy. Although the material of spider silk itself is already amazingly strong, see BioSteel, the real stuff is not just strands of the material spun into threads etc. but a complex construct of interlinked strands and bits. It's probably going to be a while till the process of producing steel cables has become as expensive as that of manufacturing spider silk. A cautionary note, though, before you start heading for a future in spider silk bridge construction: being based on organic chemistry, evolution has so far only produced very few critters with a special appetite for iron [9] that could pose a risk to steel bridges, but protein is a whole different matter. Apart from critters that would already consider a tall protein structure delectable, it's a very small evolutionary step for lots of species to specialize on protein bridge cables. Scientists would have to run hard to stay ahead of the game to keep your bridges from getting eaten as fast as you could build them. 76.97.245.5 (talk) 22:59, 13 November 2008 (UTC)[reply]
Thanks! I didn't even consider the delicious aspect... —Preceding unsigned comment added by 83.199.126.76 (talk) 23:02, 13 November 2008 (UTC)[reply]
Note that steel is widely used as a construction material not just for its tensile strength, but also for its high compressive strength and high Young's modulus (stiffness). Even if we could make suspension bridge cables from spider silk, we would still need to use steel for the towers (where compressive strength is required) and decks (where stiffness is required). Gandalf61 (talk) 10:21, 14 November 2008 (UTC)[reply]

Polar deserts

Is it possible for a hypothetical planet to have polar deserts instead of ice caps? As in - not in the sense Antarctica is a rainless desert, but rather a hot Gobi-esque desert. Lady BlahDeBlah (talk) 21:10, 13 November 2008 (UTC)[reply]

If it was tilted over like Uranus and tidally locked to its star so that one of the poles always faced the star, then that would probably be a polar desert. For a normally tilted planet, the poles are always going to be colder than the equator because the sunlight hits them at a shallower angle. --Tango (talk) 21:12, 13 November 2008 (UTC)[reply]
The first case is impossible. It the planet does rotate, none of its poles can always point to the star. Preservation of angular momentum ensures that the axis of rotation is stable. If one of the poles would always point to the sun, the axis of rotation would rotate itself. And in particular, if the planet is tidally locked, the axis of rotation (one per planetary year) is necessarily perpendicular to the orbital plane. --Stephan Schulz (talk) 21:55, 13 November 2008 (UTC)[reply]
Stephan is right (as usual) that you can't have a pole tidally locked to a star. However, if a planet is tilted more than ~60 degrees, then one expects that the poles have a higher annual average temperature than the equator because they spend more time in direct sunlight than any equatorial location. In fact, a highly tilted planet should form an equatorial ice cap instead of a polar one. Of course the poles will still get cold during the half of the year they are pointed away from the star, but just not cold enough to offset being pointed directly at the star for half the year. Dragons flight (talk) 23:48, 13 November 2008 (UTC)[reply]
I heard once that the critical angle, where the poles and equator are equally insolated, is 54°; but I never did get around to working it out. —Tamfang (talk) 08:17, 14 November 2008 (UTC)[reply]
Of course it's impossible... in my defence, I was suffering from a 24 hour stomach bug yesterday... The best you could get would be the poles going between hot desert and cold desert as the planet orbits the star. --Tango (talk) 10:50, 14 November 2008 (UTC)[reply]

So a planet with a tilt of more than 54 degrees would do it, kinda? Lady BlahDeBlah (talk) 13:55, 14 November 2008 (UTC)[reply]

...or of course, a planet that's devoid of water and closer to its star - even though the poles would still be colder than its equator, the poles could be hot and dry by our standards. I bet the poles on Venus are pretty hot and dry. But I like Tango's answer better. SteveBaker (talk) 21:43, 13 November 2008 (UTC)[reply]
I was working under the assumption that the whole planet being a desert doesn't count - it is something of a trivial solution! --Tango (talk) 10:50, 14 November 2008 (UTC)[reply]
I'm altogether in favor of trivial solutions! SteveBaker (talk) 15:33, 14 November 2008 (UTC)[reply]

Newton and the travel time of light from the Sun

I read last night that Sir Isaac Newton estimated that it took 10 minutes for light to travel from the Sun to the Earth...the true number is around 8 minutes - so that was a pretty good estimate for the era. The trouble is that he didn't know the speed of light (not even approximately) - and he didn't know how far the Sun is from the Earth either.

So how the heck did he come up with a number that's so accurate?

I'm scratching my head trying to come up with any way he could have estimated this time at all...let alone being so amazingly accurate about it.

SteveBaker (talk) 21:48, 13 November 2008 (UTC)[reply]

Ole Rømer made the first reasonably-accurate measurements of the speed of light in 1676. Newton lived until 1726/7, and he would have had access to Rømer's result. The method is described at Ole Rømer#Rømer and the speed of light, and there's a history of speed-of-light measurements at Speed of light#Measurement of the speed of light. TenOfAllTrades(talk) 21:57, 13 November 2008 (UTC)[reply]
I think knowing Kepler's laws did the rest. T^2 = D^3. where T is the Earth's orbital period and D is the planet's mean distance from the Sun. (I am not sure of the units to use... 152.16.15.23 (talk) 05:45, 14 November 2008 (UTC)[reply]
T^2 is proportional to D^3. Determining the proportionality constant (and thus, the scale of the solar system) was a major problem in astronomy for a long time. --Carnildo (talk) 22:17, 14 November 2008 (UTC)[reply]
They're equal in the appropriate units, and the anon did say they weren't sure of the units, so technically they were correct. I think the units would be years and AUs, thus the formula says 1=1, which is indeed true. --Tango (talk) 23:02, 15 November 2008 (UTC)[reply]
Rømer calculated the speed of light compared to the speed of the earth in its orbit. This gives the time for light to reach the earth with a little maths without needing to know the distance to the sun. Dmcq (talk) 11:40, 14 November 2008 (UTC)[reply]
Indeed. Our Ole Rømer article says that, prior to Rømer's more accurate measurements, Cassini used observations of Jupiter's satellites to conclude in 1675 that "light seems to take about ten to eleven minutes to cross a distance equal to the half-diameter of the terrestrial orbit". The calculation is based on the apparent variation in the satellites' periods depending on whether the Earth is approaching or receding from Jupiter - essentially, it is a Doppler effect measurement. Gandalf61 (talk) 11:57, 14 November 2008 (UTC)[reply]

LHC

Does the Large Hadron Collider only operate at night? 99.226.138.202 (talk) 23:31, 13 November 2008 (UTC)[reply]

No. Dragons flight (talk) 23:51, 13 November 2008 (UTC)[reply]
Is there any reason to suspect that the LHC only operates at night? --98.114.98.124 (talk) 13:15, 14 November 2008 (UTC)[reply]
If you run it at night, fewer people in Europe will notice when the black hole sucks them up, because they'll be asleep. It's purely pragmatic. --98.217.8.46 (talk) 01:51, 15 November 2008 (UTC)[reply]
One of the large colliders only opperates in Summer when the power consumption is lower. In Winter the Electricity is needed for heating... This might be the case to work in the night, although I never heared of that.--Stone (talk) 13:52, 14 November 2008 (UTC)[reply]
There is too much involved in operating a large accelerator to be able to power it up and down every day. In general, large colliders run 24-7 and the people who work on/with them get scheduled in shifts that run around the clock. Dragons flight (talk) 15:38, 14 November 2008 (UTC)[reply]
Actually - it doesn't operate at all right now because it's broken and is going to take many more months to fix. It's possible that they might want to operate it at night in order to place the bulk of the earth between the machine and the sun in order to shield it in some way - but that wouldn't be true for all of the experiments it does. SteveBaker (talk) 15:31, 14 November 2008 (UTC)[reply]


November 14

non-enzymatic catalyst

I was justing thinking, some biological enzymes like lysosome catalyze the substrate conversion to product by preferentially binding to the transition state. This transition state binding lowers the Gibbs free energy of TS, therefore speeding up the reaction. My question is why is it UNLIKELY that non-enzymatic catalysts operate by this fashion? —Preceding unsigned comment added by 128.252.173.86 (talk) 01:16, 14 November 2008 (UTC)[reply]

See Enzyme#"Lock and key" model. It has to do with the complexity of enzymes as molecules. Consider the difference between an enzyme, which is basically structured to recognize a SINGLE transition state based on its peculiar protein structure, versus "non-specific" catalysts, such as MnO2 or suface catalysts like Platinum or Paladium. Enyzmes are the only molecules even CLOSE to the complexity necessary to "recognize" a specific transition state. Non-organic catalysts just aren't that "picky". --Jayron32.talk.contribs 03:35, 14 November 2008 (UTC)[reply]

Neem oil as insecticide

Resolved

I find references for Neem used as an insecticide. Plus some pharmaceutical company had a patent lawsuit going, so there's evidence it works. But how does it kill the bugs? Inhibit reproduction? Glomm up their breathing mechanism? Damage their organs? Other? 76.97.245.5 (talk) 01:40, 14 November 2008 (UTC)[reply]

Neem oil is a type of horticultural oil which also is said to have insecticidal properties. Take a look at these links [10] and[11] --Eriastrum (talk) 23:48, 16 November 2008 (UTC)[reply]
Thanks. The links help a lot. 76.97.245.5 (talk) 11:14, 24 November 2008 (UTC)[reply]

creatively, how heavy might I make pure heat (historical caloric) in liquid form?

Resolved

Hi guys. Last time I asked about the temperature of pure heat and got very good responses, including historical things like caloric, etc. This time I would like a creative answer to how much, hypothetically, caloric might weigh, even though the whole idea is at complete odds with about two hundred years of detailed observations. Thinking back before then, to the seventeen hundreds and before, what might be a plausible weight scientists could have given to heat? Thank you. —Preceding unsigned comment added by 83.199.126.76 (talk) 03:53, 14 November 2008 (UTC)[reply]

Um, heat is not a substance... I thought we went over this. Heat is just a form of motion. Motion doesn't have a temperature, or a weight. --Jayron32.talk.contribs 03:55, 14 November 2008 (UTC)[reply]
I suppose you could apply Einstein's Energy = mass*(speed of light^2) equation to figure out how much mass an object's thermal energy would be equivalent too. This has no bearing on reality: adding energy to a system via heat does not increase the system's mass at all. (except perhaps in the world of freaky-particle physics that I know nothing about and assuming we are staying within classical velocities). You can calculate the amount of thermal energy has been added to a system by knowing how much the temperature changed (Celcius) and the object-you-are-heating's heat capacity. Then you can find out how much mass has an equivalent amount of energy by using E = mc^2. If you keep your units straight you should wind up with a rather unimpressive small number (or if you find small numbers astounding an impressively small number). 152.16.15.23 (talk) 04:31, 14 November 2008 (UTC)[reply]
Adding heat certainly adds mass; see my post below. --Tardis (talk) 15:28, 14 November 2008 (UTC)[reply]
When did this become Wikipedia:Reference desk/Nonsense? 24.76.161.28 (talk) 04:18, 14 November 2008 (UTC)[reply]
Seriously dude, if you are writing a fictional story ANYWAYS, well, just make it up. Are you really just asking people to give you ideas for your story? I mean, if you aren't interested in writing a story where the laws of nature bears any resemblence to reality, why bother even asking us?!? Just make it up yourself! --Jayron32.talk.contribs 04:21, 14 November 2008 (UTC)[reply]
I didn't say it shouldn't bear any resemblance to reality. If scientists never had any instruments, science would still be in the dark ages... And caloric isn't from the dark ages, but much later. :)
Please follow-up at pseudoscience refdesk, not here. --Scray (talk) 04:22, 14 November 2008 (UTC)[reply]
Where can I find the pseudoscience refdesk? 216.239.234.196 (talk) 13:56, 14 November 2008 (UTC)[reply]
haha, but obviously of the actual refdesks science is the closest :)
Yikes, 24.76 please don't get so grumpy (the internet does not transmit emotions well). While you may have intended to come off differently, it seemed like you were ranting, which isn't going to make people want to answer your question. (Just to be sure I'm trying to be friendly myself so imagine a calm soothing voice with this text) Some friendly advice from someone who has asked many off the wall questions here myself, it would help to phrase "odd" questions like this so refdeskers people know you want a creative answer rather than a scientific one. I realize you tried by posting pseudoscience in the header, but it may have also been useful to ask in a more hypothetical manner like "How might I be able to express heat as mass? I know that doesn't make any sense realistically but hypothetically can anyone think of some way to do it?" 152.16.15.23 (talk) 05:00, 14 November 2008 (UTC)[reply]
Thank you, you're an angel (even though they don't exist). I'm deleting my 'rant' and changing the subject. I love you very much.
Why would energy have mass? 216.239.234.196 (talk) 13:56, 14 November 2008 (UTC)[reply]
It doesn't I think, they're just interchangable per E=MC2. A bit of mass is lost when atoms are formed from protons and neutrons because they need energy for the strong nuclear force to hold them together. That's the basic gist of it, anyway. To the OP, please understand, you cannot ask for a real world answer to something that bears no resemblence to real life. We can't weigh heat, we can only describe it as moving particles. —Cyclonenim (talk · contribs · email) 14:36, 14 November 2008 (UTC)[reply]
You have a couple of things backwards there. Nuclear fusion releases energy, rather than requiring it from the incoming nucleons. (In general, it doesn't take energy to hold things together; instead, the fact that they are held together implies that it would take energy to pry them apart, so they must have less energy when together than when apart.) The energy that is released was potential energy before the particles combined; that the result has less mass than the sum of the constituents implies that the now-absent potential energy had — or was — mass. The same applies to all forms of energy; a hot brick has more mass than a cold brick, and a charged battery has more mass than a discharged one. With mass (perhaps "as mass"?) comes inertia and gravity; even light gravitates (it has to; otherwise gravitational lensing would violate conservation of momentum).
I find it next to useless to try to partition the "amount" of something into "mass" and "energy"; energy is just a term that implies certain behavior (like being transferrable) for some portion of the "stuff" present. You can consider them different "currencies" with an exchange rate of if you like, but then considering an object's amount of each is much like asking how much of the worth of a computer should be measured in euros and how much in yen. The answer depends on where it is and what you're doing with it and is clearly unrelated to the intrinsic value of the object. --Tardis (talk) 15:28, 14 November 2008 (UTC)[reply]
Dear OP: The last time you asked (essentially) the same question, we tried to tell you - as politely as possible - that this is a bloody stupid question and consequently it can't be answered. You pigheadedly insisted that we still try to answer it - lacing it with caveats to the point of meaninglessness. So this time - I'll just come right out and say it: It's a bloody stupid question and it STILL can't be answered. There is no such thing - nor can there ever be such a thing as "liquid heat" - I don't know how you got this concept wedged into your brain - but you need to get it un-wedged. As for everyone else - please don't feed the troll. SteveBaker (talk) 15:25, 14 November 2008 (UTC)[reply]
SteveBaker, I'm at a complete loss. I like your answers here and at other reference desks, they're highly informative and well-reasoned. But I just don't understand why this question (or the previous one) bothers you. Would it bother you as much if I were asking about astrological signs, making it clear that I wasn't looking for a scientific answer, which is that astrology == nonsense? If I said "to those who believe in astrology, which would be good signs for a Gemini's partner"? It might be "essentially" the same question as coming back and asking "okay, other than the people who told me to go **** myself because of how unscientific astrology is, I got very good responses, and now would like to ask about who good partners are for a Libra.". Seriously, I don't see how you can consider me a troll for asking for something whimsical. Do you think I believe in liquid heat? No. Do you think I'm pretending to believe in liquid heat? No. If one day I asked about spiderman's powers, and a few days later I asked about superman's powers, making clear that I meant my quesiton "in-universe" style and not from the point of view of an actually, really existing spiderman or superman, would you call me a troll? No. In this case my question has been answered below, "caloric was also thought of as a weightless gas". If the spiderman article told me that he is able to stick to buildings through tiny, sharp hairs that come out of his fingertips, would you deny me this information just because it's nonsense? Of course not. so be nice. you might have a point if I pretended to believe in this stuff, or if my question wasn't very clear on the point that I'm asking for pseudoscientific answers...
If you were truly asking for a non-scientific answer (which admittedly you are claiming to do) then there would be no problem - but you're asking for things like masses and temperatures which are firmly in the realm of science - they are measurements - and then you are demanding "non-scientific measurements" - which is a contradictory demand. If you were asking for (say) historical information about liquid heat or what the world of cuisine feels about heat as a liquid - then that might (maybe) be appropriate. But demanding to know the weight of something that quite clearly doesn't exist - is just bloody stupid. It's like asking how much a ghost weighs or what the mating rituals of mauve elephants are. So you're repeatedly asking for things that you KNOW can't be answered (if only because we explained that to you the first time you asked). Worse still...you're deliberately asking the question on the SCIENCE reference desk - after I've pointed out that it's inappropriate to do that. It's a pure waste of everyone's time - and I know full well that you know that - because this is precisely the behavior of a troll. Repeatedly asking something in order to waste people's time and create discord is Trollish behavior - which we don't tolerate here. If you continue to exhibit this kind of behavior - you're going to get very unpopular around here - and that can only lead to "Very Bad Things". So kindly drop it. SteveBaker (talk) 21:55, 15 November 2008 (UTC)[reply]
You're being incredibly rude to me. I asked for a creative answer to the weight aspect of caloric and received this, below, in the form of the answer Caloric was also thought of as a weightless gas that could pass in and out of pores in solids and liquids. Honestly, if I were asking about whether the moons of a planet (other than Earth, such as Jupiter's moons), would have been considered to move around that planet, or around Earth, in the geocentric world view, would you consider that a troll? Honestly, I think you're being just incredibly rude. I am clearly not making any impression of believing in caloric (or geocentrism). And these are things that REAL SCIENTISTS really believed hundreds of years ago. I think my question is EXACTLY the same as asking from a "cooking" perspective, since I am asking from a historical perspective that really existed. Why this would bother you is utterly beyond me. —Preceding unsigned comment added by 83.199.126.76 (talk) 01:10, 16 November 2008 (UTC)[reply]


Then what the hell is this? Matt Deres (talk) 20:45, 14 November 2008 (UTC)[reply]
Anti-freeze with a stupid brand name? --Tango (talk) 21:21, 14 November 2008 (UTC)[reply]

Perhaps you would be interested in reading Caloric theory. Here is a quote from the very first line (emphasis added) : "Caloric was also thought of as a weightless gas that could pass in and out of pores in solids and liquids.". So there's your answer. Even back when scientists thought heat was a liquid, they thought it was weightless. I'm sure they came to this conclusion by noticing that hot objects do not get lighter as they cool off. APL (talk) 04:24, 15 November 2008 (UTC)[reply]

Thank you for the answer. I had indeed missed that part of the article! I'm marking this question resolved.

CONTRADICTION TO RELATIVITY

question accidentally deleted by new question poster. Restored by 152.16.15.23 (talk) Harshagg (talk) 03:57, 14 November 2008 (UTC)harshagg[reply]


Hello,

question is something tricky pls. try to understand it

Question :-

Consider 2 persons A and B .A is at rest wrt earth and B is moving with velocity of light wrt earth.
  • I A observe B then time for B will be infinity wrt A.
  • And if B observe A then for A time will be infinity wrt B.

According to this time for A and B will increase wrt each other and time increase will be same so time cannot be infinity for either A or B wrt each other. Which actually contradict relativity.

Is this concept true if yes then relativity doesn't exit and if not what is wrong in this???

(Email removed to protect from spam; answers will be provided here. --Bowlhover (talk) 04:05, 14 November 2008 (UTC))[reply]

Not a contradiction. This was answered about 3-4 days ago; look above or at the archives. Basically, in order for A and B to "compare notes" one or the other must accelerate to reach each other's frame of reference, and this acceleration brings them into the same frame of reference, and removes the paradox. I am sure SteveBaker will be along shortly to give you the right answer, and tell you how wrong I am, but I am pretty sure I have the basic idea here... --Jayron32.talk.contribs 04:11, 14 November 2008 (UTC)[reply]
The main problem is that the theory of relativity says no object with mass can move at the speed of light. PrimeHunter (talk) 04:28, 14 November 2008 (UTC)[reply]
Sorry for being pigheaded about this but it can create all kinds of confusion if people leave out "speed of light in a vacuum". The experts are going to say "of course" but ordinary folks might spend years trying to figure some phenomena out till they get a "now they tell me" revelation. This doesn't affect OP's question though. 76.97.245.5 (talk) 09:41, 14 November 2008 (UTC)[reply]
And massless objects going at the speed of light cannot be decelerated to sublight speeds, so there's never a chance to 'compare notes'. Algebraist 11:58, 14 November 2008 (UTC)[reply]

Retrocausal information transfer.

Trevor Loughlin (talk) 04:54, 14 November 2008 If a certain device dramatically cut the low frequency component of consecutive incorrect predictions of a one bit random event generator (or deterministic and chaotic events for that matter) over billions of results, turning a bell shaped curve into a spike with rarely more than three incorrect predictions, would it be fair to call this process a form of intelligible faster than light signaling? —Preceding unsigned comment added by Trevor Loughlin (talkcontribs) 04:51, 14 November 2008 (UTC)[reply]

Hi Trevor, you accidentally deleted someone's question when you posted yours. Don't worry, I fixed it! These refdesk pages are really active so it is easy to accidentally overwrite someone else's questions. You can avoid this by clicking the ask new question link at the top of the page which creates a separate little section all for you. If you want to edit it later, scroll down to where you asked the question and click the edit button on the right hand side of your subject header rather than click the edit this page tab on the top of your screen. It cuts down on edit conflicts that way... 152.16.15.23 (talk)
I'm not sure if I understand your question entirely. (I don't know what the "low frequency component of consecutive incorrect predictions" is specifically) If you are asking if hypothetically we could predict the outcome of a random 2 state event better than chance will we have succeeded in superluminal information transmission? For that to be the case, some form of information needs to be transmitted at superluminal speeds. I am trying to think of a way to exploit this to create a superluminal situation, but I am not having any luck so far. Can you clarify when hypothetically during this process of random generation and subsequent transmission, the value of the bit is being correctly guessed at? 152.16.15.23 (talk) 05:23, 14 November 2008 (UTC)[reply]
As long as the value of the bit can only be predicted after it has been generated (i.e. you can't see the future), and it moves at slower than light speeds then no. 152.16.15.23 (talk) 05:39, 14 November 2008 (UTC)[reply]
And a warning that a low pass filter cannot work in real time, but can only produced a delayed result. Graeme Bartlett (talk) 06:00, 14 November 2008 (UTC)[reply]
Clearly if any device predicted the future, even to the slightest extent, by increasing th accuracy of predictions, it have immense practical effects. I would install a bank of them and bleed the Las Vegas casinos dry by predicting the roulette wheel outcomes, while making a jillian dollars on Wall Street. Edison (talk) 06:15, 14 November 2008 (UTC)[reply]

I have, though not quite on that scale (yet) because you have to start with a reasonably low bet and not be greedy even with this device, or in the short term you could still lose your bankroll. I turned £2 into £85 at 1 pence per go on a casino which is now closed (due to loss of American business, not me.) It took a week and got boring in the end. Beating the lottery would be more fun. My main aim is to increase the accuracy still further to predict and prevent (as in minority report ,but without the precogs) abductions and terrorist attacks before they happen(multiple universe theory gets rid of any paradox)-but only for nations that respect basic human rights! A sort of terrorism in reverse. The device works by sorting randomness a bit like Maxwell's Demon sorts atoms, but after 27 years work I will make a bit of money and rule the world for a while before I give more details! Of course the most intriguing aspect would be to send ASCII code from a future wikipedia page to bring transhuman technology from the far future to right now, eliminating inconvenience like aging and death. The device can be considered an infinite improbability drive. It alters probability enough to reverse the arrow of time on a macroscopic scale, and i wonder, though this is pure speculation, whether this could be enhanced enough to reverse the thermodynamic arrow for physical objects. I will leave you to think about the implications of this. Incidentally the device is not a filter in the common sense of the word, I am talking about the result of the predictions. This device DOES work in real time. The low pass cut-off refers to the analysis of the previous results (a histogram of how many incorrect predictions there were in a row, and the fact that the distribution is non-random. The device waits until it has made its prediction and then waits for the user to enter the predicted binary result via two form buttons, and in the process it compares the input to the past prediction (more often correct than not) and makes a line graph and frequency polygon from these results. It can also work in continuous test mode, using a separate electronic true random number generator plugged into the computer. The device uses its own true random number generator to provide the bits for the forced synchronicity mathematical (negentropy) software I have developed. User:Trevor Loughlin|Trevor Loughlin]] (talk) 14:03, 14 November 2008 (UTC)[reply]

This is incredible! CBHA (talk) 14:19, 14 November 2008 (UTC)[reply]
Excellent use of definition #1. — Lomn 14:33, 14 November 2008 (UTC)[reply]

I think this proves that the visual basic 6.0 programming language is not for dummies after all. The enormous complexity of the mathematics such as designing software logic gates that have not been thought of before, such as the "analogue XOR gate" in the quantum neural network would not have been possible without the ease of the VB6 programming environment. I can program visual c++ with MFC but if I had to use a less intuitive compiler I would have got nowhere. Bring back classic VB6.0!Trevor Loughlin (talk) 14:42, 14 November 2008 (UTC)[reply]

At this point I think it best to remind you that the Reference Desk's core purpose is to provide referenced answers to questions, not serve as a soapbox or venue for promoting original research. You seem to be drifting pretty far from the original "superluminal information" concept. — Lomn 16:07, 14 November 2008 (UTC)[reply]
I'm not sure this filter will work at all like you want it to. Even though you can plot the number of times you guess wrong consecutively, you won't improve your guessing rate from 50-50 (over a large number of guesses). If you think about it, that same curve also applies for the number of times you guess correctly consecutively. With enough guesses, I bet if you plot them they will be exactly the same, indicating you have roughly the same number of right answers as wrong answers. 152.16.15.23 (talk) 16:18, 14 November 2008 (UTC)[reply]


Yes it is entirely possible to cheat an online casino by predicting their random number generator. [12] No it does not count as superluminal communication, even if the casino is on Mars. APL (talk) 04:16, 15 November 2008 (UTC)[reply]

Also, Yes it is possible to constantly come out ahead at online poker against stupid humans, simply by playing the odds. People make bots for this all the time. No prediction needed. APL (talk) 04:18, 15 November 2008 (UTC)[reply]

WRONG ON BOTH COUNTS. First if a casino uses a psuedo-random number generator OF COURSE they are asking for trouble, because NO finite computer program can EVER produce truly unpredictable numbers,which is why nearly every sensible casino now uses totally unpredictable electronic quantum random number generators-look this up to understand the difference. Secondly it is true that a good robot will beat a novice player because poker has an element of skill and the robot never misses a trick or gets tired, but that is an entirely different issue.No amount of skill will predict true random events. Also some robots actually HACK into the system, there is no prediction involved in this, just cheating, and the security flaw has already been corrected. By the way, if anyone sells or gives you a robot to play online roulette you will lose money. Unlike poker bot's they cannot work in the long term. If they did work the people who wrote the software would keep it to themselves.

My device does not have any internal model of what it is going to predict, because it does not predict, rather it TRANSMITS data from the future to the present by modulating probabilty waves. Thus it is capable of "predicting" things that no amount of skill could predict, such as things with no or virtally no previous history like the twin towers attack. —Preceding unsigned comment added by Trevor Loughlin (talkcontribs) 06:08, 15 November 2008 (UTC)[reply]

Do you mean to tell us, Trevor, that you can make a 100,000 binary predictions of random events that have 50:50 odds of going one way or another, and be right even just 50,700 times (ie 0.7% better than pure luck)? I hereby bet you $58,000 to your $460 that you can't. —Preceding unsigned comment added by 83.199.126.76 (talk) 14:21, 15 November 2008 (UTC)[reply]
Care to tell us how you picked those numbers? —Tamfang (talk) 18:26, 20 November 2008 (UTC)[reply]

I will make a web page with a client program linked to my device, and then you can test the device before you want to take on such a bet! —Preceding unsigned comment added by Trevor Loughlin (talkcontribs) 16:01, 15 November 2008 (UTC)[reply]

Listen, if you want to rule the world with your device, you'd better NOT make any sort of public interface, otherwise you may lose your competitive advantage. I suggest you take only my private bet, and make the interface available only to me, since I will not be asking any questions about your device (so you don't lose your competitive advantage), and $58,000 will certainly help you start ruling the world. The bet is, you get $58,000 if you make 50,700 correct predictions out of 100,000 guesses -- ie a 0.7% advantage over blind luck -- as to which of two possibilities my next random number will be (you don't even have to show me your device or tell me anything about it) against your $460 if you lose. I am more than happy to test this using a web interface you make available only to me (again, I don't suggest you make it public) if you can allow me to use it to predict the results of my own hardware random number generator. I hereby take your bet on these terms.

first we'll try your web interface, which will fail to predict my result at least 50,700 times out of 100,000 guesses (ie fail to have even a 0.7% advantage over blind luck), and then we'll talk about how to show you that I'm really using a random number generator and that you're really failing to guess its results better than 50,700 times in a 100,000, ie any better than 0.7% better than blind luck. Then you can pay up. Alternatively, I am happy to send you $58,000 within a fixed number of days if you are able to guess correctly even 50,700 times out of 100,000 guesses, ie if you show even a 0.7% advantage over pure luck) -- no questions asked about your device. —Preceding unsigned comment added by 83.199.126.76 (talk) 17:14, 15 November 2008 (UTC)[reply]

I recommend hurrying up and collecting the JREF prize while it's still being offered. APL (talk) 17:45, 15 November 2008 (UTC)[reply]

Just so you know, Travor, the JREF prize is a scam. It will NOT pay out money if you do something supernatural in a scientific, repeatable way. It will just say "well, since it's so repeatable under rigorous, controlled experiments, it's just part of science now, whether we understand it or not. sorry...not paranormal.... but thanks for advancing science! We have hard work ahead now, because what you showed is really ****ing weird, so please, let us get to work. Don't let the door hit you on the way out. We might call you back for even more experiments, though -- this thing has us completely puzzled, it's not supposed to be possible! We have to revise all our rules." That's right. Revise all their rules. After the fact. In reaction to your demonstration. And their rules will include the possibility of whatever you just did, since it's now part of the empirical record (or will be once Harvard, Princeton, etc.... scientists are through with their even more detailed experiments on you.) You'll note that this extra step is part of the protocol. Of course, Harvard, Princeton, Yale scientists won't be there at the preliminary step, but only after you've "passed" that step. They will be there to make sure whatever you can do is now part of official, accredited science, and that therefore you will not get one penny of your million.
I might as well say : "I'll give you $1,000,000 if you get half of America to use a non-word at all levels of speech and writing, and get the non-word into all the dictionaries". This is fraud, a scam, because if you succeed they will just say "thanks for the effort, but since half of America is using it in all levels of speech and writing and it's in all the dictionaries, it certainly is a word, regardless of the fact that you are its coiner. The million dollars is for doing it with a nonword. Go ahead: we are so firm in our belief that Americans use only real words, and that the dictionaries have only real words, that if you can show half of America using a nonword in all levels of speech and writing and in all the dictionaries, we'll pay you ONE MILLION DOLLARS". It's a fraud. It's a scam. It's not a real challenge. It's a shell game, where you're told that you can win money by pointing to the proper one of the three cards. It's a lie, because you can't win money by pointing to any of the cards. This is a scam in the EXACT SAME WAY: you're told you'll win a million dollars by doing something paranormal in a scientifically rigorous way, but that is by definition impossible, just as through the trickery of the dealer it is IMPOSSIBLE to point at the king. Not impossible because you couldn't have done anything paranormal, impossible because EVEN IF YOU DID YOU DON'T WIN. The reason it's a fraud is because EVEN IF you point at the proper card you don't get the money. EVEN IF you do something paranormal you don't get a million. By the way, just so you know, James Randi used to be a MAGICIAN. It's why he knows how to develop fraud protocols such as his challenge.
There is no CHALLENGE, they are just ripping people off. He says : "if you demonstrate something supernatural in a scientifically rigorous way"... Look, there are plenty of things that science is surprised by, doesn't understand at all. If you had said you would do any of them before science produced that result, scientists would have said "no way! that's supernatural". but now that they know the results, they call it a part of science, even if unexplained. So there is no way to win with James Randi. You could clap your nude arms and have a perfect gold sphere with a diameter the size of your head fall out from your hands every time you did that, as many times as you did that, after you went nude through a metal detector into a room surrounded by a thousand scientists, and stood on a scale brought by them, to show you're not even carrying any extra weight, and clap and have gold spheres fall out as many times as you wanted, and they wouldn't pay out: as soon as you've proved that this effect is part of the universe in a rigorous, reproduceable, scientific way, they will say "well, damn is that unexplained, but now that we have so much evidence, it's not paranormal anymore. It's just an unexplained part of nature." STAY AWAY FROM THAT FRAUD, JAMES RANDI.
I, on the other hand, bet you $58,000 to your $460 without a scam protocol. You make that interface, I go on it, and if you predict even 50,700 of my random results out of 100,000 guesses I pay you $58,000, no questions asked. you don't have to show me your device. there is no second step. there is no fraud protocol. there ain't nothin but the money, which I will transfer to you within a specified number of days. But that won't happen: what will actually happen is that your device will fail to be right even 0.7% of the time, even 50,700 times out of 100,000, and once I show this to you as well, you will have to pay up $460. Do we have a deal? —Preceding unsigned comment added by 83.199.126.76 (talk) 19:29, 15 November 2008 (UTC)[reply]
The problem with this long rant about the Randi foundation prize (which is $1,000,000 - not $50,000) is that none of it is actually true. I can prove that. The fundamental premise (set out in the second and third sentences of the rant) is false - and everything from that point onwards fails because of that. User:83.199.126.76 says:
"It will NOT pay out money if you do something supernatural in a scientific, repeatable way. It will just say "well, since it's so repeatable under rigorous, controlled experiments, it's just part of science now, whether we understand it or not. sorry...not paranormal.... but thanks for advancing science!""
Well, that's simply not true, '83 is either woefully ill-informed - or is quite simply feeding us bullshit in order to make his/her bizarre case. Quoting directly from the contest rules at http://www.randi.org: "The JREF does not involve itself in the testing procedure, other than helping to design the protocol and approving the conditions under which a test will take place. All tests are designed with the participation and approval of the applicant. In most cases, the applicant will be asked to perform a relatively simple preliminary test of the claim, which if successful, will be followed by the formal test. Preliminary tests are usually conducted by associates of the JREF at the site where the applicant lives. Upon success in the preliminary testing process, the "applicant" becomes a "claimant."".
So, the conditions for winning the money are agreed in advance by both the participant and the foundation - those conditions are written down and signed by both parties before the testing starts. The prize itself is held in escrow - so if the test is passed - the Randi foundation can't prevent the money being paid. So the possibility that 86.199.126.76 brings forth that someone might pass the test and that they'd not get the money because the foundation would claim it was "just science" can't happen. Sure, if someone ever passed the test then either:
  1. The Randi foundation would have to admit that they'd made a HORRIBLE mistake and somehow allowed a perfectly normal, scientifically understood thing to be tested - and hand over the megabuck anyway...or....
  2. ...they'd have to admit that something "supernatural/paranormal" had really happened - and to be sure, scientists around the world would want to investigate it. The Randi foundation would still be on the hook for the $1M. Presumably, some major new scientific breakthrough would be made - and indeed that particular branch of the supernatural/paranormal would eventually be understood and become a part of mainstream science.
Look at our article and you can see (with the detailed example of what happened with the water dowsing test) what typically happens. The foundation and the dowsers agreed in advance every tiny detail of the test. On the day of the test, the dowsers inspected the apparatus and agreed that they could easily pass the test - and the testers agreed that if they do, they'll hand over the money...they do the test and fail MISERABLY - they don't come even close to passing - not by a mile - they perform exactly as you'd expect by chance alone. Now that they've failed (and only now) the dowsers claim that sunspots and other such crap interfered with the test. Quite how they know this now - and didn't find that out when they were practicing - and quite why they didn't specify a particular sunspot configuration for doing the test (if it's known to be so important to the outcome) is unclear.
This is pathetic, 83.199.126.76 - just pathetic. We have higher standards of research here on the science desk. The Randi foundation's prize is quite genuine...and your tirade is a ridiculously unfair disinformation campaign. SteveBaker (talk) 21:12, 15 November 2008 (UTC)[reply]
Look, it's quite obvious if you read the thread that the $58,000 is MY challenge. I said $1,000,000 or the word million about the Randi prize many times above. Anyway, $1,000,000 is only 17 times my offer -- but my offer only takes 3 minutes so set up if Trevor already has his machine hooked up to his computer and knows what he's doing. That means if the Randi prize takes anything longer than 51 minutes to complete from start to finish, my deal is a better value. Please admit to Trevor for the record that there is no way the Randi protocol would pay off with a total investment on his part of 51 minutes. In fact, there is no way to win the Randi protocol within the day, but he can win my bet within one hour of accepting it. Further, my offer pays him $58,000 no questions asked, no negotiation, doesn't have to show me the device, etc, and ALL he has to do is predict over the web 50,700 correct results out of 100,000, ie 0.7% better than dumb luck. Again, no questions asked. Please admit to him that if he has a device capable of retrocausal information transfer, it is much more worth his time to take my bet under the radar, than Randi's extensive, elaborate, and involved procedure, along with the associated publicity. Remember: Trevor wants to rule the world with his device, so the less publicity he gets the better. Admit it that my bet is an amazing opportunity for him, should he have such a device. —Preceding unsigned comment added by 83.199.126.76 (talk) 00:55, 16 November 2008 (UTC)[reply]
That's irrelevent. Do you admit that all of that rhetoric against the Randi foundation was completely unfounded? SteveBaker (talk) 01:10, 16 November 2008 (UTC)[reply]
You know, I just skimmed the JREF article and it includes the line "The foundation now requires a demonstrated media profile as well as the support from some member of the academic community before it will discuss the challenge with claimants". So unless Trevor is already famous in the media, the James Randi Foundation won't even speak to him. Does this seem scientific to you? After reading the JREF article, I have many, more specific criticisms of JREF. But they won't help Trevor, who they wouldn't even talk to. Just tell Trevor, the original poster, that unless he is famous in the media Randi foundation wouldn't even talk to him. —Preceding unsigned comment added by 83.199.126.76 (talk) 01:19, 16 November 2008 (UTC)[reply]
Sure - but every crackpot and nutjob writes to them - they simply don't have the staff to test all of those claims. So why not let the press sort out the ones with something to show. Also (since realistically, their mission is to inform people's opinions of the supernatural) it's more productive to prove that Uri Geller is a fraud than Granny Arkwrite who claims she can read people's future in tealeaves. There is a risk they might miss something truly interesting - but when faced by a tidalwave of undoubted frauds and crazies and on a limited budget - what would you have them do? If the supernatural/paranormal is "real" then surely ONE of the proponents would have made it into the media and have a shot at the Randi challenge. What's more relevent (IMHO) is that none of the really big-name claimants of supernatural powers will agree to take the test - that speaks volumes for how credible these people actually are. SteveBaker (talk) 16:18, 16 November 2008 (UTC)[reply]
IIRC, another reason given by the foundation was that they felt they feared many of the people whose requests they had been entertaining until they introduced the requirement weren't really frauds per se but likely people with genuine delusions or other mental issues which required professional help and they weren't helping anyone by giving undue attention to and wasting time and money on such people Nil Einne (talk) 06:49, 17 November 2008 (UTC)[reply]
At risk of answering the question and avoiding all this side-talk about predicting random-number generators...
Simplify this to a common analogy. I have two boxes. I place a left-hand glove in one and seal it. I place a right-hand glove in the other and seal it. I give one box to Julian and one box to Adaline. Julian hops on a spaceship and travels to Pluto (let's assume he has a real cool ship so can get there at the speed of light). Now, he tries to predict what is in Adaline's box - not his. He has two choices. It could be a right-hand glove or a left-hand glove. Statistically, if this is repeated millions of times, he will be correct 50% of the time. But, we assume he has some magic way of being correct 75% of the time - as the question states, the wrong answers are avoided more often. Does this mean that Julian has created superluminal transfer of information? No. The information is in the box. He took it with him at the speed of light. When he guesses, no information is being transferred. He is just guessing. When he opens the box, he sees in the time it takes light to get from the box to his eyes what he has. That information travelled with him at the speed of light to his destination. From that, he deduces what is in Adaline's box - he does not see Adaline's box at superluminal speeds. From here, you can expand this scenario to include 3 options in the box, 4 options in the box, 5 options in the box... as many as you like. Anything you do will show that the only information transfer is what it is in Julian's box, which travels with him at the speed he travels, which is not faster than the speed of light. -- kainaw 20:01, 15 November 2008 (UTC)[reply]

I have a device that gets a small percentage above chance for the first 10000 goes and then over the next million a decline sets in but then at a fraction of a percent, then no further decline occurs over billions of results. But the consecutive miss spectrum I have published is my new device, which I have reason to believe is much more efficient. I am at this very moment testing the new device for the running total score and percentage of hits and misses (relevant to the challenge you suggest), also the running total score of the Martindale system played at one to twelve different stop-losses limits (relative to roulette gaming) and also programming in a hit spectrum to compare it to the miss spectrum (as suggested) but previous experiments show a big contrast in the hits spectrum, which has a long tail showing greater than average long hits sequences compared to the short miss sequences, so I would imagine this effect will be more pronounced but cannot comment until the test is complete. Since I have four ComScire true random number generators (souped up to double speed by the manufacturer, the only other person who knows about my experiments) each working at 2 MHz and the device works at about 5 predictions a second, it should take till the end of Sunday to know if the new device is efficient enough to meet the challenge. I will be improving the device still further. As for compromising this technology, since it requires a powerful dedicated server and specialized quantum based hardware, a client, public or private would merely automate the transfer of information to and from the device, it would contain none of the software mathematics and (obviously) none of the hardware either. As all this money you are offering me, I predict that you will use my predictions to play the stock market or on-line casinos and make ten times the money you are offering me-but I am O.K. with this! I will email you a PRIVATE client or otherwise if everyone gets the data (whats left) of the financial system will be destroyed! For every winner there is a loser, and irresponsible use of this device could have bad consequences. On the other hand if you are taking a gamble with money you do not have (unlike Randi, who uses insurers I believe) then I think it is only fair to give you a free test (equivalent to fun games in a casino) because when I decide to take the challenge it is highly improbable that I will fail. As for Randi, I could always pretend I had psi abilities! I have the $ 460 in cash ready and have no problems losing it,because it will be payment for testing my theories. e-mail me at james.edward@tesco net and you can personally test my device. And thank you for giving me a bit of incentive, because once I have basic proof of a concept I get bored and start thinking up new ideas instead of develop what I already have-though the greatest incentive I have is the thought of preventing this years inevitable child abduction and the enormous damage these incidents are doing to society. I do question the morality of keeping this idea to myself sometimes-a few clicks and all could be revealed on this site, and well financed, better scientific minds nearly as great as mine would bring this project to 100% accuracy in a month. I did ask the Exeter weather center to connect my hardware to their supercomputer but they were not interested, and I am pretty sure that with their help the Asian tsunami could have been predicted. —Preceding unsigned comment added by Trevor Loughlin (talkcontribs) 03:15, 16 November 2008 (UTC)[reply]

Thanks for the answer. I've sent you an e-mail, waiting to hear your results with the new device. 23:47, 16 November 2008 (UTC) —Preceding unsigned comment added by 86.217.32.24 (talk)

P.S. Kainaw, I think Bells Theorem is similar to what you are talking about. But Bells theorem only creates correlations that can be measured after the event, as in modern quantum teleportation experiments. But supposing you encrypted real information to look as it was random but knew the code one step ahead, developed a pure mathematical method of warping probabilty waves(!) to predict the random bits, and then decoded them with the bit you are about to use (which is in a SIPO shift register. Then you have a different proposition, as I have found out. My device forces Bells theorem to transmit any form of data in a closed timelike curve.Trevor Loughlin (talk) 03:51, 16 November 2008 (UTC)[reply]

Personally - I'm betting you have a bug in your testing system. If you program in Visual Basic - you clearly don't have solid computer programming skills (it's just not a programming language that serious programmers use - so I KNOW you don't know your stuff in this regard) - so the odds are extremely high that you have a screwup in there someplace. Before you start making claims to make a killing on the stockmarket and save humanity from tsunami's - you need to step back and examine your testing procedures a LOT more rigorously. Realise that the odds of you overturning most of science on the basis of a single Visual Basic program is as near zero as anyone could imagine. It's OVERWHELMINGLY likely that you've made a teeny-tiny slipup - perhaps something as simple as a roundoff error in your "success average" calculation that's producing these tiny skews to your results. If you could truly predict random numbers in advance then making perpetual motion machines using the "Maxwell Daemon" approach would also be possible - and we know that's not possible. So to be very clear - I completely discount your claims - they are simply not possible. On the other hand - as a computer programmer of 35 years - I can't count the number of tiny, subtle errors I've seen that caused people to make outrageous claims. Remember - extraordinary claims require extraordinary evidence. The scientists who are ignoring you are 100% correct to do so. The probability that you're right rather than making some naive slipup is so astronomically overwhelming that it's simply not worth their effort to investigate further. Making these large, grandiose claims is only making you seem more like a typical nut-job. So dial it back a bit - inject some healthy skepticism and humility for those who have been there before. You need to behave like a scientist and start from the ASSUMPTION that your code is wrong (trust me - either you are lying to us - or it IS wrong). If after a couple of months of careful investigation you can't find a problem - then get someone else with decent programming skills to review it. (And before you ask: No - I'm not going to do it - I think you're a nut-job!). SteveBaker (talk) 16:18, 16 November 2008 (UTC)[reply]
Better plan - if you're so sure it works, take it to Las Vegas and make a few million dollars. After that, people will believe you. --Tango (talk) 19:05, 16 November 2008 (UTC)[reply]
I don't know about that, if you start to beat the odds people are going to pay a lot of attention to you. AFAIK, Vegas is supposed to be relatively clean nowadays so worst case you should get banned and perhaps investigated there's always the risk you'll piss off the wrong people and find yourself at the wrong end of criminals. Far better to make your money off the lotto where this is IMHO a lot less likely Nil Einne (talk) 06:56, 17 November 2008 (UTC)[reply]
To win either in Vegas or in state lotto, doing slightly better than chance (as Trevor claims) is not enough: you need to beat the house advantage – which is much bigger in state lotto. —Tamfang (talk) 18:39, 20 November 2008 (UTC)[reply]
Long time later (searching for something else) but who said anything about 'slightly better'? If you win the powerball or equivalent 10 times in a row, I don't think anyone is going to question you are doing far better than chance. You will probably be investigated and perhaps after the 3rd or 4th time they will stop you playing more but I think most lottos are clean whereas as I'm said I'm not certain Vegas is. Then again, you could use your predicting device to predict whether anyone is going to murder or beat you up for 'cheating' Nil Einne (talk) 04:25, 18 June 2011 (UTC)[reply]

See the screen capture. the column on the left shows (top to bottom) the hit to miss and miss to hit transitions, profit using simulated Martindale systems at limits 1 to 12 (limit one being right the first time, and even this is over unity, as are the majority of Martindale limit predictors, which is even less likely by chance (Martindale normally works for a bit and then loses violently.) It seems to have got a lot better than that, I will download a new picture but its 04:13 GMT (I am a UK citizen) and I need to go to bed. Incidentally Scott Wilbur of Comscire is always happy to let his team test my device and confirms my anomalous results on previous less efficient predictors , but I think I'll do Vegas before I send him this one.

By Martindale do you mean martingale (betting system) and/or martingale (probability theory)? Have a look at Kelly criterion. —Tamfang (talk) 18:57, 20 November 2008 (UTC)[reply]

bronchitis/passing out - need information

My friend was diagonised with bronchitis 3 months ago. he was passing out every time he coughed. he was treated after one month and was discharged. he has started coughing and getting the feeling he's going to pass out. Is this normal? what is the cause of the passing out? please advise. —Preceding unsigned comment added by Njmagondu (talkcontribs) 09:46, 14 November 2008 (UTC)[reply]

We cannot give medical advice on the Reference Desk GaryReggae (talk) 10:08, 14 November 2008 (UTC)[reply]
Your friend needs to see a doctor. Axl ¤ [Talk] 10:49, 14 November 2008 (UTC)[reply]
Try looking at passing out and bronchitis. 152.16.15.23 (talk) 16:25, 14 November 2008 (UTC)[reply]

Cure for AIDS by giving him transplanted blood stem cells

Recently, doctors have reported that they could cure at least one man of AIDS using a rare treatment. See the full story [here]. The case has been reported first in the Wall Street Journal.

Why did they choose a non-scientific publication to describe the case? Mr.K. (talk) 12:20, 14 November 2008 (UTC)[reply]

It is not uncommon for authors to announce their findings in popular media before the findings are formally published in peer-reviewed literature. --98.114.98.124 (talk) 13:12, 14 November 2008 (UTC)[reply]
The key phrase here is "peer-reviewed literature". The Wall Street Journal will print anything that sells papers. It doesn't have to be true. A real medical journal will require review by experts in the field before agreeing to publish an article. So, when a story such as this comes out in the press before the medical journals, there is a very high probability that it will fail peer review. -- kainaw 13:36, 14 November 2008 (UTC)[reply]
A more generous viewpoint would be that peer review takes a long time and they wanted to publish quickly and the peer reviewed version will follow later. I wouldn't be that generous though... --Tango (talk) 13:44, 14 November 2008 (UTC)[reply]
They didn't choose the WSJ. The case was presented at the Conference on Retroviruses and Opportunistic Infections in February and a special meeting convened by the Foundation for AIDS Research in September. The case was discussed again at a recent HIV conference and this time the mainstream noticed. Dragons flight (talk) 15:54, 14 November 2008 (UTC)[reply]
While I'd normally offer a similar answer to Kainaw's, I'm inclined to give the benefit of the doubt here. Hütter and Thiel seem to have a respectable publication track record, and the mechanism they propose for this cure seems reasonable. Bone marrow transplants have a high rate of morbidity and mortality, so this isn't a silver bullet treatment. (This therapy is only worth the risk if the patient also needs treatment for leukemia.) Donors carrying suitable mutations are relatively rare, meaning that even among patients who need a bone marrow transplant, a suitable donor will be difficult to find.
In other words, scientifically speaking, this story isn't that huge. It isn't going to directly help more than a very small fraction of HIV-positive individuals, and it operates by a mechanism that was already essentially known to work. The mainstream media have picked it up because of the OMG they CURED teh AIDS1!!!! factor; it's roughly akin to the Scientists cure cancer stories that we see every couple of months, whenever someone has a promising result in an in vitro trial. To summarize: these guys did cure an HIV-positive patient; it's pretty cool from a technical standpoint; the method is only applicable to a tiny subset of infected individuals; we've still got a long way to go for a generally-applicable cure; this proof-of-principle lends (further) credence to some gene-therapeutic approaches which are in the works. TenOfAllTrades(talk) 14:34, 14 November 2008 (UTC)[reply]
I agree with much of this, but it may be premature to say "these guys did cure an HIV-positive patient", because only time will tell. I recall when some thought nef-deleted virus would be a vaccine strain for HIV, until people infected with this strain of HIV progressed to AIDS. I've given other reasons to take a deep breath before cheering too loudly here: Talk:AIDS#Cure_Cases. It is an interesting report, no doubt, but "cure" should be used carefully. --Scray (talk) 05:28, 15 November 2008 (UTC)[reply]

Supposing it was possible to find a few uninfected bone marrow stems cell in an AIDS infected patient. Could these not be cultured without the need for a foreign donor, and then re-injected?Trevor Loughlin (talk) 15:42, 14 November 2008 (UTC)[reply]

I'm not sure if that will work for two reasons. First is the transplanted marrow from the article came from an HIV resistant patient. Second, I'm not sure if there is a method that can surely locate bone marrow with uncompromised cells. Maybe you could remove the marrow cells, treat them, then culture them... I dunno though. I've heard from someone that donating bone marrow feels like your soul is being sucked out. Can anyone corroborate that description? 152.16.15.23 (talk) 16:42, 14 November 2008 (UTC)[reply]
Marrow donation really is not that bad for most donors, but isn't generally necessary now, anyway. Stem cells can be obtained by treating with factors like G-CSF then drawing blood, isolating the uncommon stem cells using antibodies like anti-CD34. BTW, simple treatment of stem cells with antiretrovirals won't do the trick, because of HIV latency. --Scray (talk) 05:23, 15 November 2008 (UTC)[reply]

RNAi in flatworms

What species of flatworms have been used in RNAi studies? I know of S. mansoni, S. japonicum and F. hepatica. Does anyone know about monogenea or planaria being used in RNAi? Thanks Donek (talk) 13:58, 14 November 2008 (UTC)[reply]

Isn't species an artificial distinction?

OK, I'm not exactly sure how best to phrase this question, but I'll do the best I can. Isn't species an artificial distinction? That is to say that evolution is constantly happening, that life is always evolving, and that one species doesn't magically evolving to a completely new species. Everything is in a state of transition and usually very slowly, like the hour hands on a clock. 216.239.234.196 (talk) 14:04, 14 November 2008 (UTC)[reply]

Not quite sure where you are going with this, but you might be interested in our species problem article. Gandalf61 (talk) 14:07, 14 November 2008 (UTC)[reply]

The clock is a very good example. If you imagine each tick of the second hand as a genetic mutation, then every movement of the minute hand might represent different strains of a species (e.g. MRSA) and, as in your example, every hour a new species is formed. Whether the distinction is artificial or natural depends upon whether the definition of species is shared characteristics between two organisms or the ability of two organisms to successfully reproduce, respectively (I think). Donek (talk) 15:11, 14 November 2008 (UTC)[reply]


Yes - "species" is an extremely arbitary distinction - and the precise point when two sets of gene-lines have split far enough to call them "different species" is horribly vague and certainly inconsistently used.
However, like a lot of things it's very convenient.
We know that there is no distinction between a Zebra and a Lion that's materially different than the distinction between this Lion over here - and that other identical-looking Lion over there...or that cockroach or that bacterium. The two lions, the zebra, the cockroach and the bacterium are all related back in history - they have a common ancestor - they are all (including the two lions) genetically different...it's just a matter of degree.
Even using genetic similarity is a bit tricky because there is more genetic similarity between a male human and a male chimpanzee than there is between a male human and a female human (a fact which explains a lot about interpersonal relationships!). But having to call everything from a bacterium to a bactrian by the same name for the sake of some kind of linguistic purity is simply not productive. So we use this artificial distinction - in the full knowledge that it's arbitary and that there will be tricky 'corner cases' where the distinction is hard to nail down.
SteveBaker (talk) 15:16, 14 November 2008 (UTC)[reply]
The term "species" is not arbitrary. It has a very specific meaning: Any animals that can procreate and produce fertile offspring are in the same species. If mutations cause an offspring to be unable to procreate and produce fertile offspring, then it has mutated into a new species. The problem is that people who do not comprehend the definition of species use it for whatever they want it to mean. Then, a word with a specific meaning suddenly has dozens of possible meanings and nobody knows what anybody is talking about. So, ignorant use of "species" may be arbitrary, but proper use is not. -- kainaw 15:20, 14 November 2008 (UTC)[reply]
Sorry, but this is not exactly true. There are actually very intense and ongoing debates within the scientific community as to the reality and definition of the species concept, all of which revolve on the inability of a simple linguistic construct to take into account the mind-boggling diversity in nature. Ability-to-interbreed seems to work sometimes but not always as a way of determining things. There are many examples that don't fit well into the above definition, in part because biology is, well, complicated. Darwin himself recognized that under his theory, "species" was to some extent necessarily an arbitrary marker of genetic differentiation—the lack of a fixity of species guarantees this fact. The "can it interbeed" is a useful heuristic for talking about species, but for those who are really dealing with the fuzzy borders between species, and for those engaged in the philosophy of biology, it is insufficient... --98.217.8.46 (talk) 15:37, 14 November 2008 (UTC)[reply]
The "problem" is when the interbreeding concept is applied to asexual or bacterial things. For animals, interbreeding is well-accepted as the primary indicator of species. Difficulty identifying the species of a bacteria has no bearing on identification of the species of an animal. Attempting to mix up and confuse the two is rather misleading. For Darwin, he did not take the time to grab up two similar birds and see if they could breed. He just assumed that since they were very similar, they probably can breed, and went from there. Using Darwin's lack of time to perform experiments to claim that he didn't understand what an animal species was is also misleading. -- kainaw 16:17, 14 November 2008 (UTC)[reply]
(edit conflict) Is this relation "being in the same species" transitive, though? If animal X and animal Y can interbreed successfully, and animal Y and animal Z can interbreed successfully, is it always and necessarily true that animals X and Z will be able to interbreed successfully? Maelin (Talk | Contribs) 15:40, 14 November 2008 (UTC)[reply]
No, it is not transitive, and so does not give rise to well-defined equivalence classes (i.e. species). See ring species. Algebraist 16:14, 14 November 2008 (UTC)[reply]
Excellent question. I was about to ask that myself. As a layman, my response would be no, otherwise evolution wouldn't progress.
Ring species are used as an example of demonstrating that species are not discrete. You cannot assume that because two different animals procreate they are in a unique species. One of them may be in a species that the other doesn't belong to. The inability to discretely map every animal to a unique species causes problems for people who need to put each animal in its own little box. But, that is a problem for the person doing the classifications, not the animals. -- kainaw 16:23, 14 November 2008 (UTC)[reply]

216.239.234.196 (talk) 16:16, 14 November 2008 (UTC) (Outdent) I'm the OP. If it helps, what I was referring to was species within a single branch of the evolutionary tree. So, I'm not necessarily talking about a frog versus a tiger, but rather all the previous versions of frogs and all the previous versions tigers. 216.239.234.196 (talk) 16:09, 14 November 2008 (UTC)[reply]

That doesn't really affect anything. How do you know your separate species aren't the start of two branches? Dmcq (talk) 18:57, 14 November 2008 (UTC)[reply]
There are many attempts to draw trees that show how different organisms relate to one another. It is required as common homework and tests in biology (especially bioinformatics). But, any respectable professor will explain that the trees are only for the benefit of those trying to do research on some specific evolutionary trait. As explained above, an organism may bridge other species. I've used geometry since I work in bioinformatics and the students have a lot of math background. Consider the "species" of all shapes with all sides equal. An equilateral triangle and square fit that species. Now, consider the "species" of all shapes with four sides. A rectangle and a square fit that species. So, you have a square that bridges two species. You can see that species overlap. That happens with real organisms (not just abstract shapes). Trying to divide up the species so there is no overlap will lead to many false assumptions and artificial distinctions. -- kainaw 19:20, 14 November 2008 (UTC)[reply]
I think the poster is actually commenting on a related but slighty different issue. Take a tiger. It is descended from millions of other animals that we would call "tigers", and before that millions of other "cats", and before that other "mammals", and so on. Within that continuous chain of descendancy we insert names for different species, however the gradiations are often gradual rather than abrupt so choosing when a cat becomes a tiger is rather arbitrary. Dragons flight (talk) 19:32, 14 November 2008 (UTC)[reply]
There are 3-D models that I've seen. They are rather useless in my opinion. But, they show evolution over time and the speciation fades in - showing how a single species can evolve into multiple species. It isn't supposed to be arbitrary though. The term "mule" used to refer specifically to any offspring between two animals that was not fertile. That indicated that there was the start of a separation of species since animals in the same species must have fertile offspring. As evolution progresses, the two groups of animals may have DNA drift that causes procreation to fail. So, as time progresses, there is supposed to be a point where a single species separates into two groups - with possible overlap. -- kainaw 19:44, 14 November 2008 (UTC)[reply]
Then again, there are things like Ligers which are not necessarily impotent. A species can be defined based on biological capability of reproduction, but there are barriers other than biology to consider. A saint bernard and a chihuahua may be the same species, but a wolf is different? Behavior and ecology are the barriers between wolves and dogs, not anatomy. The concept of a species is a useful tool, but there are lumpers and splitters and the idea that there is a single ironclad definition of the word is a gross oversimplification. SDY (talk) 01:05, 15 November 2008 (UTC)[reply]

heat and static transfer through extension cables

i have an extension cable that when in use and it has not been fully extended it blows the socket it uses and i was also wandering about static electricity being in the extension cable if that also causes the problem? —Preceding unsigned comment added by Kendo23 (talkcontribs) 15:26, 14 November 2008 (UTC)[reply]

An undamaged extension cable should have no appreciable external effects. If you've got a cable that is blowing fuses, tripping circuit breakers, accumulating static charge, getting hot, or doing anything else abnormal, stop using it. Replacements are cheap. — Lomn 16:01, 14 November 2008 (UTC)[reply]
A coiled up extension cord can't shed heat as quickly as an extended one. Most extension cords that come wrapped up inside a reel of some kind either say in big letters that you can only pull such-and-such number of amps of current through it - or they tell you that the cord must be fully extended before it can be used safely. So I'm guessing that you're trying to run something that needs far to much current (like a room heater or something) - or that your extension cord has to be unreeled before use. There should be something written on the reel telling you what the restrictions are. Failing that - it might be damaged and shorting out against the casing or something - if so, then as Lomn says...get it fixed or throw it away! (It's definitely not static electricity by the way.) SteveBaker (talk) 16:14, 14 November 2008 (UTC)[reply]
Agree with Lomn. A hot e-cord will allow fewer amps to flow than a cold one. There appears to be a short circuit in the cord that allows a high current flow and "blows the socket" (trips the circuit breaker?). Toss it! "Replacements are cheap" (which may also be the problem hear). Static is not the problem. Saintrain (talk) 17:30, 14 November 2008 (UTC)[reply]
I've seen cords that specify different maximum currents for coiled and uncoiled use, so it could be a combination of the two. Whatever the instructions are, it's important to follow them (unless you are a qualified electrician and know when it's safe not to). --Tango (talk) 18:26, 14 November 2008 (UTC)[reply]
If it's blowing the fuse or circuit breaker, it's not due to heat accumulation. There's probably a fault in the cord's internal insulation at some point along its length. When the cord is coiled, the two conductors are brought into contact, causing a short circuit and blowing the fuse. When the cord is extended, the two wires are held apart and everything appears to work. (Until someone wiggles the cord, or steps on it, or picks it up, or...and brings the conductors together again.) I concur with all the above comments — if an extension cord is behaving oddly, stop using it. (It's possible that the cord could have been damaged by overloading it while it was coiled; the heat buildup could have melted some of the internal insulation, causing this intermittent fault.) TenOfAllTrades(talk) 18:44, 14 November 2008 (UTC)[reply]

Being infected and sweating

When I have been close to somebody with a cold, for instance when my face got showered when a young child with a cold had a sneeze, I think it is good to sweat a bit to get rid of some of the virus. I checked common cold, transpiration, perspiration and sweat therapy, and didn't find anything, but the article on sauna said: "The skin of our bodies is in effect another eliminatory organ so even when other organs are compromised in chronic illnesses or contamination, the skin through sweating can rid the body of such chemicals and toxins." And this is basically my idea about why sweating would help to prevent a cold. Is this true or is this nonsense???
(I know that maybe strictly speaking some could call this asking for medical advice, but I disagree, because I don't ask if sweating is good for curing a cold, just about if the mechanism of sweating leads to getting rid of some of the virus, possibly helping in preventing getting a cold.)
Lova Falk (talk) 16:53, 14 November 2008 (UTC)[reply]

Well, I think an important issue here is a question of scope -- can we expect sweating to operate on a scale large enough to be useful? It's entirely plausible that some amount of virus is sweated out; that much is true. But even if you sweat 1% of your body moisture (a level I would expect to be dangerously high), you'd also likely have sweated out just 1% of whatever virus concerns you (a level that can be dismissed as meaningless). If a child sneezes on your face, the most efficacious response is to promptly wipe your face. As for the sauna article, it is heavily referenced -- but it's not specifically referenced. I'm quite skeptical of three paragraphs that drop 40 references at the very end with no indication as to what relevance any of them have to the material presented. Long story short, I think it's nonsense. — Lomn 17:47, 14 November 2008 (UTC)[reply]
It is common for an active athlete to lose 2-3% of their water per hour to sweat. Dragons flight (talk) 18:00, 14 November 2008 (UTC)[reply]

There is no evidence that sweating per se improves outcome. However fever has a beneficial effect in recovery from many infections. Axl ¤ [Talk] 23:38, 14 November 2008 (UTC)[reply]

Here's a rule of thumb for you... If any process, product, or comprehensive lifestyle uses the word "toxin", it is total bullshit. Real, tested, curative processes are targeted at real chemical compounds made of real atoms, or real biological agents with real DNA and genetics and stuff like that. Anything which supposedly rids your body of "toxins" is just snake oil. --Jayron32.talk.contribs 23:54, 14 November 2008 (UTC)[reply]
Well, the liver is not made of snake oil, and it could broadly be said to remove "toxins" as one of its functions. Pharmacokinetics might be of interest to anyone looking to clear "toxins" from the body, though the article on Wikipedia is pretty general and I've yet to see any good "layman's introduction" to the subject. SDY (talk) 00:27, 15 November 2008 (UTC)[reply]
The liver removes specific compounds and infectious agents from the body, which can be named and identified and given chemical formulas, and the mechanisms for their removal can be explained using chemistry and biology. If any advertisement for anything tells you it removes "toxins" from the body, but does not name those "toxins" it is bullshit. --Jayron32.talk.contribs 00:46, 15 November 2008 (UTC)[reply]
While I agree with your implication generally, IMHO you're stating the "BS" case too strongly. For instance, hemodialysis is frequently said to work (in part) by removing "toxins", the vast majority of which we cannot name. We don't even know which are the most important in mediating uremia. --Scray (talk) 05:36, 15 November 2008 (UTC)[reply]
(After edit conflict higher up in the queue) I'm not expert. However the mechanism you are describing looks highly unlikely for preventing an infection. Cold viruses attack the Mucous membranes of your upper respiratory tract. There are no sweat glands in those. So you'd either have to get rid of the virus before it gets into your respiratory tract, for which washing would be more effective than sweating it seems; or you'd have to wait till enough of the little blighters have multiplied to be present in your sweat. (At this point you'd likely be quite sick and even the ones you loose with the water you sweat wouldn't make that big a difference.) What might work, though is that sweating is a "symptom" of a whole range of processes in your body. Since sweating is the body's cooling system it is likely that your body temperature is increased, creating an inhospitable environment for the viruses. If your body is cooling itself by sweating then your respiration rate may also be increased, that should reduce the number of viruses per volume of air. (Think hurricane vs. light breeze.) More importantly blood flow throughout your body is probably higher. This helps getting components of your Immune system to the site where the viruses are trying to get a foothold. If we are talking sauna, the higher air humidity also makes for an unfriendly environment for cold viruses. It helps keep your mucous membranes moist and flush infectious agents into your acidic stomach. The cells lining your mucous membranes are also "happier" in a moist environment and won't be as easy to attack as stressed cells. To what extent the salt that gets excreted in sweat is also excreted in mucous secretions by association I don't know. Saltwater is however more effective in nasal lavage than plain water. (see Osmotic pressure). And just to boot when you sweat your feet are likely warm: cold feet cause the blood vessels in the nose to constrict, which makes the nose colder and reduces the blood circulation, inhibiting the body's immune response. [13]  :-)Lisa4edit (talk) 00:57, 15 November 2008 (UTC)[reply]

Thank you all! Next time I get close to a child with a cold, I'll wash my hands and face, and make sure to stay warm - but not by running or biking when it's cold outside.Lova Falk (talk) 18:02, 15 November 2008 (UTC)[reply]

searching for the name of a old device with large glass discs

im serching for the name of a device the has a serie of large glass discs that turn in opposite directions and generate sparks from static electricity , i saw it in a documentory about the victorian era —Preceding unsigned comment added by 216.113.44.180 (talk) 18:43, 14 November 2008 (UTC)[reply]

Are you looking for a Wimshurst machine? TenOfAllTrades(talk) 18:46, 14 November 2008 (UTC)[reply]

yes thats it , thanks , there is just one thing i cant figure out , im not sure if the charge in this machine is generated by the disc itself or metal fixed to it and if the type of material used influences the amount of electricity generated —Preceding unsigned comment added by 216.113.44.180 (talk) 20:28, 14 November 2008 (UTC)[reply]

The discs have to be an insulator and the metal affixed to it has to be conductive. The insulating quality or dielectric strength and breakdown voltage of the insulators as well as the geometry determine the maximum voltage and current produced for a given speed of rotation. The charges are produced by electrostatic induction in the wedges of tinfoil pasted on the rotating glass discs. a 1903 encyclopedia explains it in some detail and says the discs might be made out of rubber. A good machine could produce a 1 foot (30 cm) spark. The glass was likely to be varnished to reduce moisture on the surface which would drain off the charge.Edison (talk) 22:36, 14 November 2008 (UTC)[reply]

oxyfuel combustion engine( IC engine)

out of quriosity i am posting this question, what happens, what load is applied on the piston, how much is engine efficiency when; 1) the 78% of nitrogen in the air that is supplied to the combustion chamber is replaced by oxygen ie. there will be 100% oxygen plus fuel for combustion. 2) the 78% nitrogen is removed and there is only 21% oxygen and fule in the combustion chamber of the same size as in the above case. please answer with an equation to prove it. thanx in advance. —Preceding unsigned comment added by Sharath.sgh (talkcontribs) 18:53, 14 November 2008 (UTC)[reply]

See Oxy-fuel, which while a short article, leads to several OTHER articles which may be helpful. --Jayron32.talk.contribs 23:50, 14 November 2008 (UTC)[reply]

How does water put out a fire?

Is it through cooling or does it smother the fire? —Preceding unsigned comment added by 64.234.6.82 (talk) 23:29, 14 November 2008 (UTC)[reply]

From our article on fire, "Fire extinguishing by the application of water acts by removing heat from the fuel faster than combustion generates it." Jkasd 00:36, 15 November 2008 (UTC)[reply]
Cooling, I believe. See Fire triangle. --Tango (talk) 00:38, 15 November 2008 (UTC)[reply]
Doesnt the application of water all over the burning mass also starve the fuel of oxygen?--GreenSpigot (talk) 01:42, 15 November 2008 (UTC)[reply]
If that were the case little children around the world would be suffocated as they played in water sprays. --Trieste (talk) 01:56, 15 November 2008 (UTC)[reply]
I didnt say sprays of water. I meant a mass of water. Any way, a spray of water will reduce the available oxygen content around the fire wont it?--GreenSpigot (talk) 02:02, 15 November 2008 (UTC)[reply]
Fire fighting#Use of water says that both factors (cooling and asphyxiation) come into play. Rather than the water itself, it is the water vapour that reduces the supply of oxygen to the fire - so more of a sauna effect than a shower. Gandalf61 (talk) 11:24, 15 November 2008 (UTC)[reply]
So, shouldnt the article on Fire state that fact?--GreenSpigot (talk) 13:00, 15 November 2008 (UTC)[reply]
You should mention it on the Talk:Fire page - but they're going to want a reference to show that this is true. SteveBaker (talk) 18:18, 15 November 2008 (UTC)[reply]

November 15

Vapor pressure

If I understand the subject at hand correctly (which, not having taken a chemistry course, I probably don't), all substances, including solids, have a vapor pressure, and if atmospheric pressure is reduced to below this vapor pressure, the substance at hand will evaporate/sublimate away. If so, why can solid objects exist in space (a complete vacuum)? Shouldn't they have sublimated away long since? 69.177.191.60 (talk) 00:54, 15 November 2008 (UTC)[reply]

From Vapor pressure#vapor pressure of solids, "due to their often extremely low values, measurement can be rather difficult". Outer space isn't actually a complete vacuum. Objects that are sent into space (spacecraft, satellites, etc.) are designed to have even lower vapor pressures than everyday objects. Axl ¤ [Talk] 01:04, 15 November 2008 (UTC)[reply]

Does gravity hold things in space together? Even small objects have gravity, which may be great enough to prevent the escape of atoms on their surface. It is theorized that the atmosphere of the earth's moon, and other rather small objects in space, has gradually escaped the gravitational pull. But atoms of a solid material may be unable to escape the gravitional pull of the parent body. —Preceding unsigned comment added by 98.17.34.3 (talk) 14:28, 15 November 2008 (UTC)[reply]

In order to change state from solid to gas, energy is required, see phase transition. There is very little heat in the 'vacuum' of space and no energy to cause the sublimation. Everyday objects also not not sublime when in a vacuum chamber for example. Most everyday objects are pretty stable, it is only when objects are heated close to their melting / boiling temperature that the most energetic particles close to the surface of the material will begin to vapourise. Of course a vacuum will lower these temperatures but if you do not supply energy to a system it cannot gain the energy suuficient for vapourisation. Jdrewitt (talk) 14:50, 15 November 2008 (UTC)[reply]
I remember somebody on this reference desk mentioning that solids do sublime in space, just so slowly as to be unnoticeable. — DanielLC 23:37, 15 November 2008 (UTC)[reply]
That might be true but once the most energetic particles have escaped there is no extra energy into the system to provide additional particles with the required energy to escape. It's the same principle I guess as why a puddle of water evaporates on an overcast day. Some of the particles of water in the puddle have enough energy to escape and vapourise. This is a Boltzmann distribution. The surroundings provide the puddle with enough energy to maintain the distribution and so there are always some particles with enough energy to escape from the liquid phase. Going back to the solid in space there might be, at the beginning, some particles with enough energy to sublime but there is no (or very little) energy to top up the system so none of the particles will have the energy required to sublime. Jdrewitt (talk) 10:07, 16 November 2008 (UTC)[reply]

Frameshift mutation.

Would a frame shift mutation affect the building of only one protein, or does the frame shift extend beyond the first stop codon? Can the entire chromosome be read as codons in groups of three, or can the codons start at positions offset from a multiple of three from the very first nucleotide at the 5' end? Basically I'm sure that if a point mutation occurs, it presumably it doesn't affect the entire chromosome, but I'm not sure how. Jooler (talk) 01:06, 15 November 2008 (UTC)[reply]

Think about what nucleic acid is used by the ribosome to generate protein. How is that nucleic acid formed? Is it a complete chromosome copy, or some smaller unit? --Scray (talk) 05:05, 15 November 2008 (UTC)[reply]
Jooler, did you figure out the answer to your question yet? If not, I would like to explain it, but I would rather not write all the extra stuff if you don't need it now... 152.16.15.23 (talk) 16:57, 17 November 2008 (UTC)[reply]
Yes. please explain! Thanks. Jooler (talk) 00:33, 18 November 2008 (UTC)[reply]

General equation for two bases (or metal hydroxides) reacting

Hi guys, just doing some basic chemistry and the general equation (i.e. Acid + Base -> Water + Salt) for when two bases (or metal hydroxides) react has eluded me. Specifically, I am looking for the gener equation that would help me determine the below reaction:

Thanks. Foxy Loxy Pounce! 02:01, 15 November 2008 (UTC)al[reply]

Hydroxide, beside being a Bronsted base is also a good Lewis base and thus often forms complex ions with transition metals. My best guess is some form of iron(III)hydroxyl ion:
Fe(OH)x(3-x)
it probably depends on the coordination number of Iron(III) as to the specific formula, but my guess is that this forms a complex ion of some sort. See [14], especially the description of Reactions of iron ions in solution and the following section Reactions of the iron ions with hydroxide ions. Good luck! --Jayron32.talk.contribs 02:20, 15 November 2008 (UTC)[reply]
Actually, I am probably wrong there. It looks like no reaction will occur in this case, the article I cite makes the claim that iron does not form soluble hydroxide complexes, and this page: [15] also states: "Another reaction type frequent seen is the formation of a hydroxide followed by the formation of a soluble hydroxide complex ion. Fe(OH)3 and Mg(OH)2 do not form soluble complexes, but aluminum and zinc do". Looks like no reaction will occur given those reactants! --Jayron32.talk.contribs 02:30, 15 November 2008 (UTC)[reply]
Well that explains why I could not think of the equation! Thank you very much. Foxy Loxy Pounce! 03:40, 15 November 2008 (UTC)[reply]

Fuller's formuls

This is a civil engineering question.Do anybody help by providing the proof of the Fuller's formula that is used in getting densest mix out of various fractions of construction aggregates?The formula is as follows:

      p=(d/D)^n
      where p-fraction of the material passing a certain seive of size d
      d-maximum size of the aggregate
      n-gradation coefficient usually equals 0.45 or 0.5 

202.70.74.136 (talk) 02:12, 15 November 2008 (UTC)[reply]

Well, you obviously want a function that gives p=0 when d=0 and p=1 when d=D. And scale invariance suggests a power law. So that just leaves the question of the correct exponent n. I think the value of 0.45 or 0.5 is reached empirically - this text book says Fuller's original value of 0.5 was "based on wide scale experiments"; this source says "Why 0.45 exponent in Aggregates Gradiation ? The typical answer is: "that's the way we've always done it"". Gandalf61 (talk) 11:14, 15 November 2008 (UTC)[reply]

Could u provide me the text of the article by Fuler and Thompson of 1907 about this? —Preceding unsigned comment added by 202.70.74.184 (talk) 14:41, 17 November 2008 (UTC)[reply]

Would an organism that could successfully repair all cellular damage be theoretically immortal?

The general consensus seems to be yes, and many think it even attainable. I’m open to be convinced and would like healthy immortality as much as anyone. But my objection is that there is more to an organism than its component cells. Aren’t there general effects which work on the ENTIRE system? For example, over many decades, gravity bends the spine, and causes breasts to sag, and so on. Could it be that these effects work on levels ABOVE the cellular one, on the body as a machine?

If you had a wooden bridge that gradually bowed under the weight of traffic, would you make that bridge healthy just by replacing it splinter by splinter? Aren’t there cracks BETWEEN the splinters, even between molecules, that mean that the replacement of an organism’s cells, one at a time, will not entirely negate the effects of aging? Myles325a (talk) 06:44, 15 November 2008 (UTC)[reply]

Hmm, this is suspiciously like a homework question. However I'll assume good faith and give you a couple of pointers. See biological immortality. Partly it depends on how you define "immortality". The "goal" of all organisms is to reproduce. Most unicellular organisms undergo asexual reproduction, provided they live long enough with favourable circumstances. The "daughter" organisms have identical DNA to the parent. Perhaps this is a form of immortality? Or has the organism foregone immortality in favour of reproduction? With multicellular organisms (including humans), the intervening connecting tissue between cells needs to be maintained as well as the cells themselves. However in many complex organisms, cells often undergo apoptosis. Axl ¤ [Talk] 07:25, 15 November 2008 (UTC)[reply]
Would it be the same organism? Julia Rossi (talk) 08:31, 15 November 2008 (UTC)[reply]
Aha! The good old Ship of Theseus paradox. SteveBaker (talk) 15:06, 15 November 2008 (UTC)[reply]

"We both step and do not step in the same rivers. We are and are not."

Axl ¤ [Talk] 10:08, 15 November 2008 (UTC)[reply]

You are defining a situation where the hardware of the body is somehow kept in perfect repair. That doesn't cover the "software"...our brains. I don't think it's clear whether or not our brains could continue to function indefinitely - even though the hardware is OK - would the software keep running. To pick an example - a Windows PC will gradually run more and more slowly over years of use if you don't periodically "defragment" the hard drive. Perhaps our minds have similar problems - but maybe there is no built-in "defrag" because we have not evolved to live long enough for it to matter. We know it'll run for 100 years or more - and it seems that the changes in mental acuity that come with age are related to structural matters that your magic cell repair gizmo would prevent. So I don't think we know whether the mind (as opposed to the brain) would continue to work indefinitely. SteveBaker (talk) 15:06, 15 November 2008 (UTC)[reply]
Maybe God was smart enough to install Linux (ext3) instead of Windows (ntfs) and avoid the disk defrag problem. Heehee. Oh no - me brain is open source! -- kainaw 16:26, 15 November 2008 (UTC)[reply]
Maybe...I'm pretty sure I got given Sinclair BASIC! SteveBaker (talk) 18:11, 15 November 2008 (UTC)[reply]
I thought of a better analogy for those who don't understand PC's (or who use a sensible operating system and have never defragged anything in their lives!). Imagine a large library. When they built the library, they filled it full of books and put them all in Dewey-decimal ordering with books in alphabetical order within each group. The library contains a million books - but only a couple of librarians - who are meticulous about fixing broken bindings and stopping people from dog-earing the books - they clean and polish the shelves and keep the book-worms at bay. But they never re-organize the shelves. When the library is new - you can find books really easily - you go to the right category - then you zip along the shelves to the right place in the alphabet - and there is your book. But over the years, visitors to the library pull out books and sometimes they put them back onto the shelves in the wrong order. After decades, centuries or millennia, the library gets further and further from its original state of order. The books, the shelves and the card indices are all kept in perfect repair - but things get harder and harder to find. Eventually, there are a million books in random order on the shelves and the library stops working - nobody goes there anymore and it "dies". This kind of "software" disorder could conceivably affect our minds and kill us - even if the 'hardware' of our bodies were immortal. SteveBaker (talk) 18:11, 15 November 2008 (UTC)[reply]
This might be unrelated to reparing cellular damage, but there is an organism that will revert back to its polyp stage as it ages, rendering it effectively immortal: turritopsis nutricula. ~AH1(TCU) 21:32, 15 November 2008 (UTC)[reply]
Indeed - and amoeba reproduce by splitting in two - at no point does an amoeba die of old age. Every amoeba that exists or ever has existed is (in essence) a piece of one single original immortal amoeba. That hurts my brain! SteveBaker (talk) 01:05, 16 November 2008 (UTC)[reply]

Tissue engineering could completely replace the organs and other body parts of a human as they wore out, but the brain would have to be very slowly replaced allowing the mind time to settle in to replaced areas to maintain personal identity, particularly the frontal lobes. As for defragmentation, in humans this process is called sleep.Trevor Loughlin (talk) 08:31, 16 November 2008 (UTC)[reply]

Yes - I've long believed that dreams are the consequences of the brain having to keep running while it's "defragged" (memories undergoing large scale reorganization) - because severe processing errors can happen during this process - it's necessary to "disconnect" the brain from the limbs and from 'consciousness' to avoid major practical problems. False memories formed during the reorganization are also supressed - which is why we generally forget our dreams so quickly.
However, my "defragging" analogy was not intended as a definite "reason" why we might fail to survive forever - merely an example of the kind of software process that could cause an eventual "crash" that might kill us even when the body is working just fine. It's also possible that there is no such process and that dreaming fixes the 'defragging' kinds of issues and we truly could become immortal if we had perfect cellular repair mechanisms. We don't know for sure either way.
SteveBaker (talk) 15:35, 16 November 2008 (UTC)[reply]

publishing thesis

how i can publish my pHD thesis in wikipedia. its title is Electron and light microscopical studies on the lung of one humped camel(camelus dromedarious).Dr doaa (talk) 07:04, 15 November 2008 (UTC)[reply]

You cannot publish your PhD thesis in Wikipedia. Wikipedia does not allow original research. A good way to publish elements of your thesis is to discuss with your supervisor and submit an article to a peer-reviewed journal. Axl ¤ [Talk] 07:08, 15 November 2008 (UTC)[reply]
If you want to publish original research online you could take a look at arXiv. Mr.K. (talk) 11:31, 15 November 2008 (UTC)[reply]
There are a LOT of peer-reviewed journals out there; one is likely to accept your work for publication so long as it is really novel work. It might not make it into Nature or Science, but there is probably someone that will accept it. Talk to your advisor, and see what he recommends. --Jayron32.talk.contribs 17:10, 15 November 2008 (UTC)[reply]
You could create your own website, publish your thesis and put a link to it on your wikipedia user page. Lova Falk (talk) 18:07, 15 November 2008 (UTC)[reply]
Yes, but you could not use that self-published website as a source to cite in an article... It would still need to be published in a reliable source before Wikipedia could officially "recognize" it... --Jayron32.talk.contribs 21:52, 15 November 2008 (UTC)[reply]
Even though wikipedia can't accept original research, you may have written up some introductions where you wrote about other's research and you've probably documented this well with references. For instance the physiology of the camel could go into the camel article, but make a note on the talk page that it's you adding it, so people don't think it's someone breaching your copyright. If you've also taken pictures of the lung tissue, those pictures could be added to wikipedia. EverGreg (talk) 22:27, 15 November 2008 (UTC)[reply]
I wouldn't expect the style of a PhD thesis to be appropriate for an encyclopaedia article, it would require substantial rewriting. --Tango (talk) 00:05, 16 November 2008 (UTC)[reply]

air compressors - energy required.

I want to know the energy(electricity,KW-H) usage rate for air compressors. Compressor: Centrifugal, Displacement = 1000 CFm, output pressure rating = 100psi. Suppose, the out put flow required is 500 CF per Minute. Then, what would be the power consumed in an hour. The compressor motor rating is 250 HP. Is there any thump rule or simple formula to find it out..? Whether the equation, P1.V1 = P2.V2 is applicable here..?(P1 = Atmos. pressure, P2 = 100psi. and, V1, V2 are volumes). —Preceding unsigned comment added by Ajivg1 (talkcontribs) 12:39, 15 November 2008 (UTC)[reply]

This depends on its efficiency, but 50 percent is probably a reasonable guess. 250 HP would correspond to 500 HP *(0.746 kilowatts/HP) = 370 kilowatts. That is, every hour it would use 370 kilowatt-hours. -User: Nightvid (unregistered) —Preceding unsigned comment added by 129.2.43.42 (talk) 15:10, 15 November 2008 (UTC)[reply]
The Centrifugal_compressor Is running at half-capacity though..."all 250 HP" isn't being used. At least for the compressor side of things (setting aside inefficiencies and the motor itself), Ajivg1 is on the right track for a first-principles approach. Need to figure out how much work (== change in potential energy) is being done to compress a certain amount of gas by a certain amount. See Isothermal process#Calculation of work for example. DMacks (talk) 21:27, 15 November 2008 (UTC)[reply]

Neurotransmitters

This came up recently in class and not even the PhD qualified teacher could think of an answer - why do we have so many different neurotransmitters? Apart from having different receptors which is a cop-out answer, we got so far as to say it was probably just an evolutionary mid-stage? RHB - Talk 15:49, 15 November 2008 (UTC)[reply]

What do you mean by "evolutionary mid-stage"? There is no destination in evolution, so I can't see how it's meaningful to talk about being half-way there. --Tango (talk) 16:57, 15 November 2008 (UTC)[reply]
The best answer I can think of is that we don't want any "bleed though" between the different functions of the brain. If we had only one neurotransmitter, then it could possibly "leak" into synaptic junctions where it would cause all sorts of havoc. With a wide array of neurotransmitters, each localized to a certain set of cells in the nervous system, there is less chance of "cross contamination." Although, the better answer is "because it works"; remember that evolution is not an intelligent process. Stuff gets preserved in the system only because it doesn't self-eliminate. Anything which does not cause insta-death is likely to be preserved by the system, not because it is the best possible way to do it, but because it works. --Jayron32.talk.contribs 17:07, 15 November 2008 (UTC)[reply]
My guess (maybe a bit like Jayron32's idea) is that at every single moment there is an extremely complicated activity going on in the brain, with lots and lots of processes simultaneously that influence each other. Every single movement, every single sensation, every single thought (and each thought also elicits some memory, some emotion), the arcadian rhythms, just to name some examples, all have their neural base. I think to have different neurotransmitters is far more efficient for the brain than to have only one.
As a metaphor, you could compare with colors. All information that colors contain can be expressed in binary codes, but it is far more efficient for us to see the color red, green, yellow or blue than to process 001001, 010001, 110111, or 100011.Lova Falk (talk) 19:39, 15 November 2008 (UTC)[reply]
It's not just "different receptor for different transmitter" in a lock'n'key sense. You also get different binding affinities and have to have different degration/re-uptake mechanisms. That means different neurotransmitters can be quicker- or slower-/longer-acting than others. DMacks (talk) 20:52, 15 November 2008 (UTC)[reply]
Yes, these are largely the reasons. Different neurotransmitters bind to different receptors, and therefore affect conductances of different channels on different time-scales. For example, glutamate activates NMDA (depolarizing, slow) and AMPA (depolarizing, fast) channels; acetylcholine binds to muscarinic receptors (G protein coupled, several distinct types) and nicotinic receptors (ionotropic, again several distinct types); etc... Also, different types of synapses exhibit different short- and long-term plasticity, and therefore form connections differently during development and change differently during learning. This is required for a normal development and function of a complex nervous system (say, of a vertebrate, arthropod, or cephalopod). --Dr Dima (talk) 21:55, 15 November 2008 (UTC)[reply]

in theory, could a discovery about reality invalidate the possibility of physics as a field of science?

In theory, though it's ridiculously unlikely, the entire field of mathematics could be invalidated by the discovery that no system of axioms could fail to be inconsistent, and that for any set of mathematical assumptions, there exists no p such that p can be proven using those assumptions, but not p cannot be also be proven.

Does the same thing exist in physics? Could any results (however ridiculously unlikely) completely invalidate for physicists the very possibility of physics as a science?

I'm thinking, if Einstein or Maxwell, or any other scientist, were in a coma, and for whatever reason on a long opium trip (a la Kublai Khan), living in a dream-world much like anyone's who dreams, then if they decide to do experiments in their dream, or any other rigorous analysis of their new world, they would soon conclude that whatever has happened to reality, it has invalidated the pursuit of physics forever. Outside of dreams, however, could reality (though this is ridiculously unlikely) produce any experimental results that completely invalidate the very idea of physics? —Preceding unsigned comment added by 83.199.126.76 (talk) 16:35, 15 November 2008 (UTC)[reply]

Physics (and the rest of science, by extension) is based on the assumption that the universe can be explained. That is, there is a fixed set a physical laws that govern how the universe behaves. If it turns out that that isn't the case, and the universe just makes things up as it goes along, then the whole of science goes to pot. However, all our observations so far seem to support the assumption, so it's extremely unlikely. --Tango (talk) 16:55, 15 November 2008 (UTC)[reply]
There's not a single "lynch-pin" that holds up all of science which, if shown to be incorrect, would bring the whole thing down like a house of cards. It just doesn't work that way. Sure, individual ideas are constantly being refined, but none of those ideas is SO important and SO key to the entire process would be rendered wrong. Science is descriptive not prescriptive and as such, it doesn't depend on being "right" all the time. If God came to earth and said "Look guys, I was just kidding with all those laws of physics, here's a new set of laws. Work it out!!!", it would not invalidate all of the old laws, since they worked very well to describe the universe as it existed at that time. Science would adapt to take in the new set of data. --Jayron32.talk.contribs 17:03, 15 November 2008 (UTC)[reply]
Then phsyics is in direct contrast with mathematics, in which when an old result is overturned, it is not the case that this does "not invalidate the old result, since it worked very well to describe the truth as it existed at that time." —Preceding unsigned comment added by 83.199.126.76 (talk) 17:31, 15 November 2008 (UTC)[reply]
Old results in mathematics aren't overturned, they're just built upon. Once something has been proven, that's it, it's not going to suddenly turn out to be false after all. (There is always a chance the proof will be incorrect, but once it's been checked by a decent number of mathematicians you can be pretty sure it isn't, and there is always the standard caveat from Godel, but that makes no difference to day-to-day mathematics.) --Tango (talk) 18:02, 15 November 2008 (UTC)[reply]
(ec) The most fundamental idea of physics is that is there are fundamental laws of nature that can be discovered through application of the scientific method. New laws are discovered, or old laws modified, when new data is provided by experiment. The important feature of a law of nature is that it universally true at all places and in all conditions. More extreme experiments test the boundaries of the applicability of what we believe the laws to be and occasionally they have to be updated as a result. An example is Newtons law of gravity being modified by Einstein's general relativity. But always a new set of laws is put in place that is now universally true in the new set of data. The foundation of science requires that such a set of laws exist - it is the unprovable act of faith in science if you like. It is logically possible that a discovery could be made that showed that such a set of laws, however formulated, could not exist in all circumstances. This would certainly invalidate the basis of physics, but I would be hard pressed to think of any way such an experiment could ever be performed. SpinningSpark 17:09, 15 November 2008 (UTC)[reply]
I think the point is best made by talking about what happened to Newton's laws of motion when Einstein proved them "wrong" with relativity. Physics can't be completely wrong because they predict reality with such great precision - but they certainly can (as happened with Newton's laws) be shown to be missing some important terms that only show up under conditions that we have not yet fully explored (in that case - it was the fact that Newton had not explored the nature of light adequately). There were times in past history (eg with Aristotle's laws of motion) where the laws were simply wrong (certainly true with Aristotle's stuff) - but those were always in times when the modern scientific method was not being employed. The ancient Greeks actually frowned upon the idea of doing experiments and making measurements. Modern science really doesn't allow us to make such gross errors. Once we have a law that explains a large swath of phenomena - and which makes predictions that actually come true - it's exceedingly unlikely that the law can be utterly wrong - except in realms of mass, speed, size, temperature - that we have not carefully explored.
The situation is actually rather similar in mathematics. Godel showed that any sufficiently powerful logical system will allow theorems that can neither be proven nor disproven. This is a profound (and disheartening) conclusion - but mathematics didn't end on that day - neither did very much of the theoretical or practical applications of mathematics collapse. It works too well for that kind of major collapse to happen.
And in physics - we discover that at it's heart, quantum theory says "it's all just random anyway" - and that devastating blow to the idea of the "clockwork universe" didn't really make a dent in other areas of physics. Sure, we know that in principle a massively unlikely quantum event could cause a grand piano to spontaneously materialise in the middle of my subtle experiment to find an extrasolar planet - but that fact doesn't alter the validity of that experiment in any important way - so it can be neglected. If string theory were proven true (or false...either would be profound) - our fundamental understanding of absolutely everything would change - but Hooke's law of spring stiffness wouldn't change - we could still rely on Ohm's law. That's because whatever this new theory is, it has to explain WHY voltage, current and resistance behave as they evidently do. The new theory can't say that for "normal" conditions Ohms law is wrong - because we've shown (and continue to show) that it must be correct because our computers are working OK.
Physics (and science in general) is about two things: "HOW things behave" and "WHY they behave like that". The "WHY" part certainly can (and does) change...but the HOW part can only change in such a way that's consistent with all of the 'HOW' laws we already have. We can say that what we thought were quarks and photons and electrons and such are really only vibrating 26-dimensional strings...but that change to our understanding of "WHY" can't take away the indisputable fact that HOW "electricity" flows along copper wires is according to I=V/R. Of course it's possible that the string theory might produce the discovery that Ohms law should really be: I=V/R+0.00000000000000000000000000000000000000001xP/L where 'P' is the charge on the electron and L is the amount of time elapsed since the big bang - but this would hardly be a profound 'HOW' change - only a profound 'WHY' consequence - and most people would not do physics much differently as a result. That's what happened to Newton's laws of motion. We discovered this bizzaro thing about the speed of light and that space-time is distorted by gravity fields and the changes to Newton's laws all end up being modified by this factor of (1-v2/c2) - but since for almost all practical purposes, v2/c2 is just about zero - so the 'modification' is essentially nothing. Physics changed radically - but it made absolutely zero difference to all of that beautiful victorian science that taught us how to make steam engines and such.
SteveBaker (talk) 17:41, 15 November 2008 (UTC)[reply]
I think SteveBaker has most of this correctly, but I firmly but respectfully disagree with his assessment of the difference between "why" and "how". All physics describes is the "How's". For example, one can describe "how" light is generated from a heated object blackbody radiation and also "how" photons and electrons interact to produce that light and also "how" the way the universe organized itself after the big bang led to the creation of photons and electrons. However, the big "why" questions, such as WHY the laws of the universe work the way they do, WHY it was created in the first place, WHY humans exist to study it; etc. etc. are entirely unanswerable via science. Which is OK. Science is not equipped to answer these questions, and never has and never will be. But that does not make science invalid; rather science is the best system we have to answer the "how" and "what" questions. See the works of Stephen J. Gould especially Rocks of Ages and The Hedgehog, the Fox, and the Magister's Pox for more on these ideas... --Jayron32.talk.contribs 20:15, 15 November 2008 (UTC)[reply]
I disagree - Science does have two parts, the hows and the whys. The scientific method involves making some observations, finding a law to describe those observations (a "how") and then coming up with a theory to explain that law (a "why") and predict further laws which you can then test. --Tango (talk) 20:31, 15 November 2008 (UTC)[reply]
Oh yes, science does have those two parts. Laws answer the "what" question by describing the world, and theories answer the "how" questions by explaining the world. However, the big why questions, such as "Why does the universe exist at all" are not answered beyond "just because". Science is not equipped at all to answer these questions. We can answer HOW the universe exists (i.e. the Big Bang brought it into existence), but we can't answer "For what purpose does the universe exist (i.e. WHY)." Any answer to that question, including the default "there is no purpose at all" are not testable by science, and so lie outside of the realm of scientific discipline. They are real questions tied to the human experience, and their answers cannot be gotten by any known aspect of the scientific method, be it experimentation or inference from observation. You are perfectly justified in coming to the "there is no purpose" conclusion, but science lacks the ability to back up that conclusion... --Jayron32.talk.contribs 20:46, 15 November 2008 (UTC)[reply]
That's just semantics. I'm pretty sure you and Steve agree, you're just using different words to describe the same thing. --Tango (talk) 21:07, 15 November 2008 (UTC)[reply]
You're probably right on that. I just wanted to make clear that science does not answer ALL questions about the human experience, nor does it try to. It is incorrect to assume that some great revelation about "why the universe exists" (as opposed to "how it came into existence") would invalidate a single scientific observation, law, or theory... --Jayron32.talk.contribs 21:49, 15 November 2008 (UTC)[reply]
Science isn't good at the really HUGE "why" questions. We truly don't know why the charge on the electron is -1.6x10-19Coulombs - we may never know that. We do know (to a pretty amazingly deep degree of explanation) why the DNA of humans is so similar to that of chimpanzees. We know why putting my 20kg kid on one end of a seesaw (aka teeter-totter) requires me to put my friend's 40kg kid at half the distance in order for them to balance. I can talk about the details all the way to the structure of the wood in the cross-beam and how the various nuclear forces make that work. It's an amazingly detailed understanding. However, to say that physicists aren't in the business of finding out WHY the charge on the electron is what it is - that's crazy talk. If we thought we had a shot at figuring that out - we absolutely would. The only reason we aren't working hard on the problem is that we don't have a clue how to approach it. Worse still - every time we answer a pretty tough "why" question - we're just handed another deeper one hidden beneath it. SteveBaker (talk) 00:59, 16 November 2008 (UTC)[reply]
God could prove he exists and decide to actively meddle with reality. For science to work, you pretty much have to assume that "God did it" is never the explanation for any observable event. Dragons flight (talk) 21:56, 15 November 2008 (UTC)[reply]
Unless you assume that God did everything, and science merely describes what He does in fine detail... --Jayron32.talk.contribs 00:36, 16 November 2008 (UTC)[reply]
Yes - although we also have to assume that (for example) we're not just simulations of humans inside "The Matrix" in a universe that bears no resemblance to ours...that the exterior world really exists at all...that I'm not just an abstract intelligence who is imagining all of you guys (along with the rest of the universe)...that, that, that...well, there are an infinite number of other things that we have to assume are untrue. It's easier to simply stop worrying about all of the things for which there is zero evidence and get on with trying to understand what we CAN perceive. Occam's razor - it's the only sane way to proceed. SteveBaker (talk) 00:59, 16 November 2008 (UTC)[reply]
This is more than just Occam's razor - it's a matter of falsifiability, it's not just that we don't have any evidence about them, we can't conceivable ever have any evidence. If something isn't falsifiable, then it isn't within the realm of science and should be left to the philosophers (who should them be confined to a padded room for their own protection). My favourite possibility - it's not that you're imagining all of us, but that you're being imagined - try and get your head round that! --Tango (talk) 01:17, 16 November 2008 (UTC)[reply]
What it will look like when She(mhhnbs) comes to us at last
The problem (as relating to our OP's question) is that "unfalsifiable" doesn't mean "not provable" - it means "not disprovable". If seven billion pineapple and ham pizzas were to materialize out of thin air on Wednesday lunchtime - one for every man, woman and child on earth (with a few to spare to have with beer for breakfast the following day) - then this could be taken as almost certain evidence that the IPU(pbuh) really exists. There is always a possibility that we might find proof that we are all inside "The Matrix" (eg if there was evidence of numerical roundoff error in some seemingly fundamental process...I could imagine a really cool explanation for Quantum Theory that centered around roundoff errors in the computer that's simulating our universe). Proof of existance of IPU(bbhhh) would certainly make a significant dent in modern science - but it wouldn't change Ohms law - perhaps it would be found that Ohms law works the way it does because She(mhhnbs) requires it to be so - but the fundamental laws would stand and science could continue to discover more about how She(bbhhh) chooses to run the universe. (We atheists would have SERIOUSLY conflicted emotions if THAT ever happened!) SteveBaker (talk) 02:01, 16 November 2008 (UTC)[reply]
Which is the whole point. If the Invisible Pink Unicorn exists or not, it wouldn't effect science one iota. Science only describes the world as it is, but it does not attempt to explain the big questions of "Why are we all here, and why did things turn out exactly this way". Even the default answer of "there is no reason, its all random" or "we do exist, so it doesn't make any sense to presume that we wouldn't" or even "The Invisible Pink Unicorn has made it so." The problem is that science is silent on these issues. I can have a full understanding of Evolution and Big Bang and fully accept these as scientifically valid and completely true understandings of how the universe works, and also have a full and complete relationship with God and the two ideas in no way conflict with each other... --Jayron32.talk.

contribs 02:29, 16 November 2008 (UTC)[reply]

I disagree that science doesn't attempt to explain the big questions - I think it does - but doesn't necessarily succeed at it because these answers so often seem out of reach. Also, as soon as we do explain something - it's not a big question anymore. Figuring out "where people came from" is a big question - and we pretty much have a check box in there. I think that by making god or gods unnecessary in our description of the universe around us - we've effectively answered that question. We can pretty much explain everything beyond maybe the first picosecond after the big bang to the present without requiring any intervention from supernatural beings. I'm sure that we'll shrink that one tiny moment in time when gods could have done something so a smaller and smaller slice. Removing it utterly may be harder - but I'm more than satisfied that there are no more gaps for gods to hide in. Perhaps they roll the dice for quantum mechanics - but since the consequences of that action is limited to killing the odd laboratory cat - that's a pretty miserable role for a supposedly omnipotent being. Perhaps it was the gods who decided the charge on the electron and the universal gravitational constant - but since the 'many worlds hypothesis' works pretty well (and explains a lot more than gods to) - we may assume that we're in THIS universe because of the anthropic principle. SteveBaker (talk) 04:52, 16 November 2008 (UTC)[reply]
My point - which you seemed to have missed - is that the pink unicorn could suddenly become visible and decide that pi = 3, hydrogen atoms are now heavier than lead, apricot is the 21st element, and light goes twice as fast on Tuesdays. The original poster asked for "facts" that could destroy physics as a science. My point is that the meddling of an activist God is such a problem. Most people who are religious don't just believe in a watchmaker God who set everything up and then stood back to let physics run, rather most people believe in an intercessionary God who takes an active role to change things in response to prayer and some divine plan. If God is actively rewriting reality in violation of physical laws, then that limits the applicability of physics. If God were to do enough of that, the study of physics would be worthless because past experiments would no longer be predictive of future behavior. Dragons flight (talk) 02:57, 16 November 2008 (UTC)[reply]

(unindent) The IPU cannot 'suddenly become visible' (she's the Invisible pink unicorn - that's WAY different from a mere mundane pink unicorn)!

There is a subtle difference between the OP's original question and where we've strayed in answering it.

  • The original question is about a profound discovery we might make - and proof of the existence of the IPU would be a good example of that. If we proved her existence - but she didn't do anything different as a result - then life goes on, science continues to probe the universe (albeit with a very different view of what's going on) - everything we already know continues to work - although we now have some very different underpinnings for why it works like that.
  • But that's a very different thing if the universe abruptly changed in profound ways like the value of pi changing as a result of a rare IPU intervention in human history. The OP asks what happens if we discover something profound - and that's different from something fundamental that we thought we understood well changing in an unexpected and profound way. I can't conceive of a universe where PI is 3 - so I'm going to duck out on that speculation. But the charge on the electron suddenly changing is perhaps something worth thinking about. Of course if the value did change - we'd probably rather abruptly cease to exist for reasons I can't be bothered to speculate on...but if enough other fundamental numbers somehow changed so that we'd still exist and be able to function - what would happen? I guess, initially we'd be running round like crazy trying to explain this incredible event - and if it's a direct IPU intervention - we'll never figure it out. We'd add it to our pile of axiomatic facts about the universe - not only do we not understand why the fundamental constants are amicable to life - now we also don't understand why they periodically change in value and yet still remain suitable for our further existance. We'd be puzzled by that - but the anthropic principle can still hold (although it's not too satisfying as answers go).

What would be very tough indeed would be if both things happened at once. The IPU turns up and changes the charge on the electron. Now we have to accept that not only is there some kind of supreme being - but also we're unfortunate enough to have one of the bloody annoying interfering kind that just trashes any hope we have of making progress as a species because we have this arbitary creature out there screwing us up at every turn. This would be an odd situation. It's tough to speculate. SteveBaker (talk) 04:52, 16 November 2008 (UTC)[reply]


I did some voodoo for fun as a kid. I poked a needle into a plastecine replica of my right hand and to my surprise and total horror an invisible cone created by an invisible needle appeared in my left hand and pricked it. I therefore conclude that we are in the MATRIX and though the laws of physics normally apply, the administrator of the hyper-computer we are simulated in sometimes takes the piss. I suppose it could have been a highly improbable quantum event. I was not taking drugs incidentally. Perhaps I am Neo. —Preceding unsigned comment added by Trevor Loughlin (talkcontribs) 04:08, 16 November 2008 (UTC)[reply]

Or you could have just been imagining it... --Tango (talk) 16:34, 16 November 2008 (UTC)[reply]
Certainly - without doubt! This is a typical utterly bogus story - just like the people who claim to be abducted by UFO's. Trevor's response to this astounding thing doesn't fit the results. Look - you go to all that trouble to try this out - you're clearly expecting, hoping even - that it works. The slightest twinge or tickle in the hand would seem like proof to you - so that's why it "worked". But here's what makes it seem bogus to me. Had that happened to me - I'd not have shrugged my shoulders and gotten on with life! Such a profound discovery demands something more! So I guess I'd have closed my eyes and asked someone else to stick the pin in - I'd yell the moment I felt anything - and have my friend tell me if my yell coincided with sticking the pin in. I'd want to know how far the effect works - does it work through walls? Does it work if I'm a mile away? How does the effect "know" to affect my hand and not someone elses? Can I make a clay foot and get the same effect? Can other things than sticking a pin in it produce an effect? So many questions - yet we're lead to believe that you did it once, it "worked" and that was that. It simply doesn't ring true. This is bogus - utterly bogus. SteveBaker (talk) 01:28, 17 November 2008 (UTC)[reply]

What gets me is, -each generation thinks they know what reality is. Yet there are many examples in history of major paradigm shifts! Relativity, Evolution, Quantum Mechanics, and realizing that Greek mythology gods arent running anything just to name some.

One book, by an author that sounded credible (I'll name the exact reference, as soon as I relocate it) mentioned how we know how it is across the universe because we know the way scientific laws work here. Now that's an Occams Razor!

Dr. Carefree (talk) 18:25, 16 November 2008 (UTC)[reply]

Well the thing is that those "major" shifts didn't affect anyone's daily lives very much. Sure, they changed our "understanding" of the universe - but not our practical interactions with it. Most people experienced no changes to their lives whatever as a result of the discovery of relativity - the effect is so minor at 'human scales' that we did not have to go out and buy new alarm clocks for rail travellers that measured our speed and arranged to ring at the right time to get us to work! Evolution doesn't really impact people's daily lives very much - there is no risk of your cat having T-Rex 'kittens' anytime soon. Quantum theory isn't understood at the most superficial level by 99% of the population. So sure - they are interesting and all - but the effects of these major new understandings are actually fairly small.
There is a good reason for that - as I've been trying to explain in previous posts to this thread - we already understood how wheels and pulley and weights and motors and...all that stuff...worked before relativity came along. Relativity is ONLY relevent at huge relative velocities. Evolution is only (generally) important over very long timescales - and where it does impact us (things like drug resistant disease models) - these things came along LONG after evolution was well understood. Quantum mechanics only affects VERY small things - and our ability to make things like flash memory that actually USES quantum theoretical effects required technology that was far out of our reach when we figured it out. Since the start of the application of the scientific method (so let's rule out the ancient greeks and start this debate in the 1600's) - not one single major paradigm shift has replaced the earlier theory - it's only ever expanded on it in the 'corner cases' at the extremes of experience. Even such major things as the abandonment of alchemy for chemistry didn't change what we understood about the effects of heat on substances for the majority of things that we heat up in most of our lives.
Quantum theory enabled flash memory development - that's caused a revolution in photography, music distribution, communications - HUGE parts of our lives have been revolutionized by cheap memory that doesn't "forget" when you turn it off. But does ANYONE attribute this to Quantum theory? No! They just assume that the technology fairies came along one night and made this possible.
SteveBaker (talk) 01:28, 17 November 2008 (UTC)[reply]

My vooodo experiment was not repeatable, when I had regained my composure and stuck the needle in it did not work, ever again. I was not expecting anything and had made the hand as an artwork. The simplest explaination would be some form of self-hypnosis. I try to make it seem like a joke, but it certainly was no joke at the time. The shape of the cone could not have been created naturally. The pricking sensation seemed absolutely real. It belongs with other unprovable phenomena such as stigmata, ghosts and UFO's. —Preceding unsigned comment added by Trevor Loughlin (talkcontribs) 03:37, 18 November 2008 (UTC)[reply]

Another one for the bird psychologists here...

What's going on with the parrot here? Anger thing or sex thing would be my guess. --Kurt Shaped Box (talk) 16:56, 15 November 2008 (UTC)[reply]

It could have just been trained to do it for some strange reason. --Tango (talk) 17:09, 15 November 2008 (UTC)[reply]
Yeah, that's certainly a possibility - though some of the comments left by other users suggest that this behaviour is not unique to that particular bird. I have very little experience with Amazon parrots myself, so I'm not really sure either way... --Kurt Shaped Box (talk) 22:53, 15 November 2008 (UTC)[reply]
Googling indicates that it may be a courtship ritual. They certainly mention the spread tail feathers and the strutting up and down the perch. This would assume that the parrot is a male, of course, which (despite my name) I can´t judge. --Cookatoo.ergo.ZooM (talk) 00:11, 16 November 2008 (UTC)[reply]
In my (very limited) experience deducing the gender of a bird from it's display of sexual behavior is highly unreliable. Particularly when it comes to captive ones. Since this bird is not reacting to another parrot, there's not telling. If it were your bird you could run a series of experiments to narrow down what sets off this behavior. I for one take it as a given though that part of the answer is that the poor thing is lonely. (OR We used to have a female budgie that tried to **** anything and everything, including another female we had boarding for the holidays. She viciously attacked a male we tried to introduce. When we tell the story we find that more often than not other people who had/have birds are not the least bit surprised but nod and relate similar stories.) Since parrots can be very playful it may also just be playing "peekaboo" 76.97.245.5 (talk) 06:37, 16 November 2008 (UTC)[reply]
I think that I may have solved this one myself. Take a look at this video of a female Amazon of the same species 'in heat', as they say. Notice the similarity of her vocalizations to the sound of a drawer being open and shut... Could that be it, do you think? --Kurt Shaped Box (talk) 10:16, 17 November 2008 (UTC)[reply]

Conservation of momentum

Okay, let's say you were hanging off a cliff, attached to a rope which is wrapped around a pulley attached to the edge of the cliff, and the rope is connected to a large rock (there's also no gravity). If you were to pull on the rope, obviously you would start rising and the rock would start moving. But how would momentum be conserved? The magnitude of your momentum is the same as the magnitude of the rock's momentum, but their in different directions.

The planet to which the pulley is firmly bolted is a part of the 'system'. It moves in the opposite direction to you and the rock...albeit rather slowly! (And this is true whether or not we are ignoring gravity) SteveBaker (talk) 20:02, 15 November 2008 (UTC)[reply]
How do you hang without gravity? --Tango (talk)
Maybe not hang, but the point is that you're not moving. And I don't see why the planet would move. —Preceding unsigned comment added by 76.69.241.185 (talk) 20:59, 15 November 2008 (UTC)[reply]
The pulley imparts a force on the rope, so there is an equal an opposite force on the pulley, which is attached to the planet. I still don't understand the scenario, though... --Tango (talk) 21:05, 15 November 2008 (UTC)[reply]
Why wouldn't the planet move? It's not fixed in place. When you pull the rope - there is a downward force on both ends of the rope (due to your inertia and that of the rock) - that pair of forces push down on the pulley - the pulley pushes downs on the bolts - the bolts push onto the cliff-face and the cliff-face pushes onto the planet. F=mA applies to planets just as it does to you and the rock - so if you and the rock have a combined mass of (say) 100kg - and the planet (Earth, for example) has a mass of 5,000,000,000,000,000,000,000,000 kg - then if you and the rock move upwards at (say) 1 meter per second - then the planet MUST move the other way at 100/5,000,000,000,000,000,000,000,000 meters/second. That's Newton's third law - and planets are not exempt from it! You don't notice the planet moving because 0.0000000000000000000002 meters/second is pretty amazingly slow - but it most certainly does move - and it does so in exactly the way necessary to keep the center-of-mass of the earth/you/rock/pulley/rope system exactly stationary - which conserves momentum. Anyway - whether you happen to believe it or not - that IS the correct answer to your question!
If it helps - imagine the pulley is not fixed to the planet - in fact, there is no planet. You, the rock, the rope and the pulley are just floating in space. As you pull on the rope, you and the rock move S-L-O-W-L-Y towards the pulley - but the lightweight pulley comes flying at you at great speed! Now imagine the pulley is bolted to a 100lb anvil - when you pull on the rope, you and your rock go towards the pulley and the anvil - and the pulley/anvil come slowly towards you. Keep doing this experiment with bigger and bigger things bolted to the pulley - and pretty soon you are imagining an entire planet bolted to the pulley. There is no point at which the rules changed...planets aren't special when it comes to the laws of motion.
SteveBaker (talk) 23:09, 15 November 2008 (UTC)[reply]

transistor configuration116.71.178.160 (talk) 20:33, 15 November 2008 (UTC)

i am going to make this http://www.redcircuits.com/Page38.htm project .but i dont know that how the transistor Q3 being used in the circuit is baised and why it could control the amplitude of louder sound in a limit while allowing all little sound fully amplified. plz.plz.plz.plz.plz.plz.help me]

Wow. I would have to dig out some circuit analysis books from the attic. Anyone current on the use of voltage divider and diodes in biasing a base?Edison (talk) 05:13, 16 November 2008 (UTC)[reply]
The diode D1 is acting as a peak detector and the resulting dc biases the base of transistor Q3. As this dc is proportional to the sound level the output of Q3 proportionatly decreases, this output is then low-pass filtered by R6/C4 and passed to transistor Q1 via resistor R2. This provides bias to the base of Q1, increasing sound level driving Q1 nearer to cut-off and thus reducing the gain. The purpose of the whole loop is for agc. SpinningSpark 01:58, 17 November 2008 (UTC)[reply]
Agree. The gain of Q1 is altered by altering its emitter current and hence its Re. This is achieved by Q3 passing a dc value proportional to the signal amplitude back to the base of Q1. This is a typical agc circuit found in many transistor radios.--GreenSpigot (talk) 02:13, 17 November 2008 (UTC)[reply]

photoelectric cell

is photoelctric cells can oparate on shorter waves than normal light like uv x-rays and gama radation or mabe longer waves?--אזרח תמים (talk) 20:44, 15 November 2008 (UTC)[reply]

Photoelectric cells individually have a wavelength at which they are most efficient. Based on choice of materials, that can generally range from ultraviolet to near infrared. By stacking cells with different optimal wavelength on top of each other, it is also possible to create efficient absorption across a wider range. In general, gamma rays are too high energy to be captured by normal photovoltaics. Dragons flight (talk) 21:39, 15 November 2008 (UTC)[reply]

bat some of the photons most be catch in no?--אזרח תמים (talk) 16:10, 17 November 2008 (UTC)[reply]

Rainfall runoff through a four inch pipe

I'm no engineer, but....Given a roof area of 1000 sq. ft. I understand that 1 inch of rain will create 625 gals of water. I've also read that a 4 inch pipe can carry 160 or 195 gals. per min. at(I think)a flow rate of 5 ft.per sec. I live in New England and am wondering about how many inches of rain per hour, perhaps combined with snow melt, would it take to overwhelm the pipe? At three inches of rain per hour, which I would think would be extraordinary, am I correct that I would get about 1900 gals. of rain per hour which, if evenly distributed, would give about 31 gals. per min. It seems like a 4 inch pipe would be plenty big enough. Also how do I calculate flow rate through the pipe or is it even necessary? Thanks for any help. —Preceding unsigned comment added by 98.229.160.179 (talk) 21:34, 15 November 2008 (UTC)[reply]

My experience with the (often spectacular) Texas rainstorms is that no system you could devise would take care of it. Four inches in an hour is quite common here - and we've had 14 inches overnight on one occasion a few years ago. Most Texas homes don't even have guttering and down-spouts. The problem with that amount of water doesn't seem to be the capacity of the pipes - but the speed that the water comes off the roof. It comes down so quickly that it completely overshoots the guttering and spills over the edge. Also, no matter how clean you keep your guttering - in a good storm, leaves are blown off trees and can block the guttering so quickly that the gutters will block before they get to do their job.
But I can't fault your math - in theory it should be OK. This page suggests the most rain you'll see would be around the same as the worst Texas can do - and the worse you'll see anywhere in the USA is around 164mm/hr (about 6 and a half inches in an hour!)...of course snow melt might make for more runoff than that. But even at that rate, your downpipe is only operating at half capacity.
Calculating the flow rate through the pipe is tough - but because gravity is accelerating the water as it falls, the limiting factor is how fast the water can get through the hole at the top of the pipe - once it's in the pipe - the water beneath will be moving faster - so if anything, it'll get sucked down. Probably the biggest upgrade you could do would be to have some large funnel-like contraptions at the tops of the pipes.
A bigger concern is the guttering - it's only a half-cylinder - so it has only half the volume of the down-pipes - and since they slope so gently - there isn't much help from gravity to get the water moving down the gutter. Where the gutter meets the downpipe is the worst place. So I would suggest that you use more down-pipes rather than making the down-pipes bigger.
SteveBaker (talk) 22:50, 15 November 2008 (UTC)[reply]
I'll just note that if this is a practical question rather than a theoretical one, your local building codes may have something to say about it. --Anonymous, 05:20 UTC, November 16, 2008.
Another good reason to get more down-pipes is that if one section of your gutter gets clogged the water has another way to go than over the edge of your gutter (some of it will anyway). Also if you run into a problem with the working of a section of the gutter you only have to fix the section between downspouts. (But do check with your building code and don't utterly ruin the appearance of your house. You might want to sell it some when.)76.97.245.5 (talk) 05:31, 16 November 2008 (UTC)[reply]
Wouldn't you have a problem with the fact that the gutter can only be tilted downwards in one direction? I'm not sure what angle of tilt is required for guttering to work effectively, but if it's too great (how great depends on the distance between downpipes) the water would overflow and some point downhill from the downpipe and the downpipe would be useless (it could still be useful for the guttering coming from the other direction, of course). --Tango (talk) 16:32, 16 November 2008 (UTC)[reply]

Effects of global warming on microbes

Hi. Have any studies been done on the effects of global warming on important species of microbes, especially those that act as the basis of some ecosystems? Is there an article on this? If so, what have such studies concluded? Thanks. ~AH1(TCU) 21:38, 15 November 2008 (UTC)[reply]

See Effects of global warming for an introduction to the topic... --Jayron32.talk.contribs 21:44, 15 November 2008 (UTC)[reply]
I don't see anything about microbes, and by that I mean the extinction risk posed to certain key microbes, which is not covered by extinction risk from climate change. ~AH1(TCU) 21:54, 15 November 2008 (UTC)[reply]
I think it's pretty unlikely that they would become extinct. They have really short reproductive cycles and (typically) their DNA replication mechanisms are primitive so that DNA transcription errors are common. They have massive population sizes - so the probability of a beneficial mutation showing up is high. This allows them to evolve and adapt spectacularly quickly (in a matter of days in some cases). Larger plants and animals have a harder time. Our live spans are long - so we might only have a couple of generations before the problem kicks in - and our DNA transcription mechanisms have built-in protection against copying errors that means that we don't mutate easily. There are so few of the larger lifeforms (compared to bacteria at least) that there are comparatively few 'mutants' who might provide a source of evolutionary improvements.
It's also the case that the simpler lifestyle of bacteria allows them to adapt radically - there are 'extremophile' bacteria pretty much everywhere we look - but only a very few species large plants and animals can survive near volcanoes or hot-springs or in ice/snow conditions.
"Extinction" is a slippery term. If some precise species of bacteria exists today - but due to changing environmental pressure is completely evolved into a new species in 5 years from now - did it "become extinct"? The question is not whether they'll survive (they will) - but what impact will their evolution have on us?
So I'm pretty sure that bacteria will adapt quickly, easily and smoothly to the worst we can do to them - and whether we call that "extinction" is just a matter of language.
If useful bacteria (like maybe gut flora) evolve to prefer to live outside the human body then maybe we're in deep trouble. If disease bacteria evolve faster than in the past under the pressure of climate change - we're in trouble. So they'll be fine - but maybe we'll be in even more trouble than we expect...it's not easy to know.
SteveBaker (talk) 22:25, 15 November 2008 (UTC)[reply]


November 16

Bee stings on different body parts

This question has been removed. Per the reference desk guidelines, the reference desk is not an appropriate place to request medical, legal or other professional advice, including any kind of medical diagnosis, prognosis, or treatment recommendations. For such advice, please see a qualified professional. If you don't believe this is such a request, please explain what you meant to ask, either here or on the Reference Desk's talk page.
This question has been removed. Per the reference desk guidelines, the reference desk is not an appropriate place to request medical, legal or other professional advice, including any kind of medical diagnosis or prognosis, or treatment recommendations. For such advice, please see a qualified professional. If you don't believe this is such a request, please explain what you meant to ask, either here or on the Reference Desk's talk page. --~~~~

If you're looking for explanations for your different reactions to bee stings – particularly if you're anticipating getting more, as you keep bees – you should speak to your doctor. We cannot offer an interpretation of your symptoms on Wikipedia. TenOfAllTrades(talk) 00:00, 16 November 2008 (UTC)[reply]

I'm not allergic to bee sting, so a bee sting isn't dangerous, only annoying. I know Ref desk doesn't give medical advice, I'm not looking for medical advice. And it's not the sort of thing I'd waste my doctor's time on. Likewise, I'm not seeking advice on how to treat a bee sting. So it's rather inappropriate to delete my question as asking medical advice. I find it likely that someone reading this reference desk would have an idea as to how apitoxin works, and why it would have significantly different effects on different parts of the body. Biology question, not a medical question.

I'm looking for an answer to a (hopefully simple) question: Why would bee stings to the face just above the moustache and 1cm to the right of the nose; to the upper arm on the biceps; and to the forearm result in very different reactions. Any stings to the forearm are rather nasty, with swelling; stings to face and biceps make a small bump. Why such a vast difference? And note that all these reactions are local reactions and are not dangerous. I'm not seeking advice on what to do about it, only ideas as to why the difference. --Psud (talk) 01:17, 16 November 2008 (UTC)[reply]

Obvious question : Are you sure it was the same species of bee each time? APL (talk) 02:39, 16 November 2008 (UTC)[reply]
Yep. Same colony. --Psud (talk) 03:35, 16 November 2008 (UTC)[reply]
There are too many factors involved to give an answer. Bees do not deliver the same amount of toxin with each sting. Stingers are delivered at different depths. Taking time to remove a stinger increases the amount of toxin delivered. Oily skin (such as that around the face) may have work as an antacid - depending heavily on the person. Rubbing a sting can increase swelling. That is just what comes to mind immediately. I hope it is obvious now that this question is asking for a diagnosis of your specific body and is not a general question about general human anatomy. That is why it is considered medical. It is impossible to diagnose why one part of your body reacts more to a sting than another without performing a physical exam. -- kainaw 04:24, 16 November 2008 (UTC)[reply]
I saw someone stung on the face at or near the location described, and he looked really funny with his eye swollen shut. If the swelling had been on the bicep it would scarcely have been noticeable. That is certainly one difference. Edison (talk) 05:08, 16 November 2008 (UTC)[reply]
Could blood flow or muscle make a difference? Face has plenty of muscle near the surface around the mouth, the biceps is a muscle, the forearm has practically no muscles... doesn't it? --121.127.209.126 (talk) 07:29, 16 November 2008 (UTC)[reply]
That ain't it. The forearm has plenty of muscles, including almost all of those that control motions of the fingers and wrist. There are a few intrinsic muscles of the hand, but most hand strength comes from the forearm muscles. --Scray (talk) 01:21, 17 November 2008 (UTC)[reply]

Surface area of the internet

Is it at all possible to calculate this, or am I being mocked by this guy at work? —Preceding unsigned comment added by 81.77.235.192 (talk) 00:27, 16 November 2008 (UTC)[reply]

It would require a lot more clarity on the terms to know if it even made sense. Does "this guy" mean the physical architecture of the internet? If that's the case then it can be calculated as an elaborate Fermi problem. Does he mean the amount of physical space the entire internet would take up if, say, printed onto paper? Or is it implying there is a physical space in the logical manipulation of bits that make up our electronic conversations, the abstract logical space between my screen and yours that constitutes the system of communication and distribution that we call "the internet"? (In the latter case, I don't think it is calculable.) In any case, it is a query that, if you want to appear smart, you answer with more queries that show how smart you are, that you recognize that this is by itself not a clear question. --98.217.8.46 (talk) 00:38, 16 November 2008 (UTC)[reply]
This looks like a nonsensical question. Its sort of like asking what shape is the color red. It seems to lack any connection to reality. If we take the internet as merely the sum total of the information it contains, well information has no "shape" and as such, has no surface area... --Jayron32.talk.contribs 00:47, 16 November 2008 (UTC)[reply]
Isn't the Internet a series of tubes, whose surface area can easily be calculated? Clarityfiend (talk) 01:15, 16 November 2008 (UTC)[reply]


I can't think of any aspect of the Internet that has "area". The volume of all of the equipment is something one could imagine calculating - or maybe the total distance between nodes - the total number of bytes of data? But the AREA?!? I don't know what that means. Unless you're doing some kind of 'space-filling-curve' fractal kinda thing? Like this: (ob. XKCD link). Maybe the total area of all of the web pages if you printed them out on legal-sized paper? (But with dynamic-content pages, the answer could maybe be "infinite")...Meh - put this one on the "unanswerable-unless-clarified" pile. SteveBaker (talk) 01:21, 16 November 2008 (UTC)[reply]
If you have lots of spare computer time and don't want to donate it to more reasonable causes (e.g. distributed computing for science projects) you might take the following approach: Since the internet is all about moving information in electronic form, you could roughly estimate the average number of electrons involved in moving the bits of internet information at any given time. You might then also estimate an area for 2D mapping as described by Steve. (And have another decade worth of discussions as to what would count as internet related movements of electrons, what volume of space that would cover and how to convert that into a 2D figure.) You could then expand on that by including the space occupied by atoms dedicated to storing web pages and other auxiliary elements necessary to keep the internet going. So I'd go with 42. Challenge your co-worker to prove it's wrong! 76.97.245.5 (talk) 05:00, 16 November 2008 (UTC)[reply]
He might want a Fractal dimension on networks. Dmcq (talk) 09:58, 17 November 2008 (UTC)[reply]

How much is 8x1017 coulombs?

The Orders of magnitude (charge) article is woefully incomplete. I've calculated that an 8x1017C charge is enough for the Abraham-Lorentz force to cause the Earth to spiral into the Sun in about 5.5 billion years, and I want to know how much that really is, besides enough to make 80 million tonnes of aluminum. Also, does outer space follow the normal laws of electricity well enough to give it a resistance? If so, what is it? I want to use that to figure out exactly how hard it would be to apply a charge to the Earth. — DanielLC 01:59, 16 November 2008 (UTC)[reply]

Well, according to the Faraday constant, 96,485 coulombs equals the charge carried by exactly 1 mole of electrons. So 8x1017 coulombs is the charge of 4.6x1046 electrons, or roughly the number of electrons in 7.7x1019 kilograms of hydrogen. This is about the mass of a medium-to-smallish sized asteroid. To put this in other terms, an ampere is a coulomb/second, so if this charge were carried by a current of 8x1017 amperes, at say a potential of 120 V (house current), it would use 9.6x1019 kilowatts; or in one second would use 2.6x1016 kilowatt hours of electricity. According to my latest electric bill, the current rate of $.099 per kWh means that driving 8x1017 coulombs of electic charge through my household circuit for 1 second would cost me $2.64x1015. That's 2.6 quintillion dollars, or significantly more than the entire supply of all cash, goods, and services of the entire world. Do either of these calculations help put it into scale for you? --Jayron32.talk.contribs 02:50, 16 November 2008 (UTC)[reply]
1 gram of hydrogen is roughly one mole, so it would hold 96,485 coulombs. This would mean that it would take roughly 8e12 grams, or 8e9 kilograms of hydrogen, not 8e19. — DanielLC 03:28, 16 November 2008 (UTC)[reply]
Dang. Yer right. Musta hit the extra "1" key somewhere. But the household cost is right, no?!? --Jayron32.talk.contribs 21:06, 16 November 2008 (UTC)[reply]

I just calculated that the energy required to launch that many electrons out of orbit is about 3.6e-10 joules. Moving the electrons to your house would require all the supplies of the entire world, but you could move them into space with the energy gained from the nuclear fission of ten atoms. I suppose getting them through the atmosphere would be the hard part. Anyway, making the Earth fall into the Sun conventionally would take about 8e26 kilowatt hours, so this still might work better. — DanielLC 03:56, 16 November 2008 (UTC)[reply]

If you're waiting 5.5 billion years, the Sun will have expanded into a red giant and possibly engulfed the Earth anyway (and if it doesn't, it will certainly be much closer), so you can save yourself the trouble! --Tango (talk) 23:56, 16 November 2008 (UTC)[reply]
I figured taking longer than that would be cheating, so I set that as the upper limit. I suppose what I should be saying is that it would take at least 8e17 C. So, about the resistance of space? — DanielLC 05:06, 17 November 2008 (UTC)[reply]
The resistance in space is pretty frigging huge. You need a series of actual atoms to transfer electrons along; and atoms are few and far between in space, so the resistance is likely, um, excuse the pun, "astronomically high". --Jayron32.talk.contribs 15:36, 17 November 2008 (UTC)[reply]

Does black hole evaporation violate the second law of thermodynamics?

A black hole sucks enormous amounts of matter and energy into it, presumably (correct me if I am wrong) including the microwave background radiation from the ongoing cooling of our universe after the big bang. But having re-concentrated all this energy, the hole slowly shrinks and evaporates due to Hawking radiation, violently releasing this energy all at once when it is small enough. Surley this violates the second law of thermodynamics? Could the universe be a perpetual motion machine? —Preceding unsigned comment added by Trevor Loughlin (talkcontribs) 04:43, 16 November 2008 (UTC)[reply]

Well - the energy goes into the black hole as organized, structured matter - stars and such - and comes out as photons hurtling off in random directions. That means that mass/energy was conserved but disorder increased - and that's pretty much what you'd expect with the second law. I guess I don't see the problem. SteveBaker (talk) 05:16, 16 November 2008 (UTC)[reply]

True about the stars. But what about the low grade energy in the form of microwaves that it sucks in? I assume that as the black hole shrinks the hawking radiation it gives out is of a higher grade, X-Rays for example.

Yes, during the last moments of evaporation, the black hole will almost certainly produce extremely energetic particles - "almost" as in "we don't have a good theory of quantum gravitation to know better". But you need to look at the complete lifetime of the black hole. You cannot make a black hole from just low-energy photons, but need some initial mass that collapses. Whatever started the black hole lost an enormous amount of information in this collapse. So yes, some of the Hawking radiation particles will have more energy than some of the cosmic microwave background that goes into it. But on average, particles escaping from the black hole will be of individually lower energy than those that went into it, and hence have a higher overall entropy. BTW, currently (in cosmological times, so give or take a few billion years) a stellar-sized black hole gets more energy from the microwave background than it loses due to Hawking radiation. Before a black hole can effectively evaporate, the universe has to expand quite a bit. --Stephan Schulz (talk) 11:19, 16 November 2008 (UTC)[reply]
The mass at which a black hole's temperature matches the current CMBR temperature (2.7 K) is about 2×10−8 solar masses. So, regardless of a black hole's initial size, it will radiate all but 2×10−8 solar masses of its energy at a temperature below 2.7 K. For a black hole of one solar mass that's 99.999998%. That's ignoring the further absorption of CMBR radiation that Stephan Schulz mentioned. -- BenRG (talk) 14:02, 16 November 2008 (UTC)[reply]

Paradox

Moved to the humantities desk here by an I.P. user. —Cyclonenim (talk · contribs · email) 12:59, 16 November 2008 (UTC)[reply]

Bad smelling, but delicious?

Since smell is so closely related to our sense of taste, is it possible for anything to smell terrible, but actually taste delicious? --69.151.187.196 (talk) 17:04, 16 November 2008 (UTC

Well, there is the durian, which is the edible fruit of Durio zibethinus, a tree which grows in southeast Asis. The durian smells awful but tastes delicious.
My parents love Gorgonzola cheese. I hate the smell(!), and they don't like it either, but they love the taste. -- Aeluwas (talk) 17:46, 16 November 2008 (UTC)[reply]
I can confirm that durian smells awful, I've never been able to pluck up the courage to see what it tastes like! --Tango (talk) 18:32, 16 November 2008 (UTC)[reply]
Kimchi is the worst smelling popular food that I've smelled. People who like really like it. I can't get past the smell to eat it. I do agree with Aeluwas. Many cheeses smell terrible, but I haven't had a cheese that I didn't like. Now that I think of it, fish smells terrible also, but I like fish also. A especially like to put stinky little sardines on roasted red peppers and sprinkle with blue cheese. -- kainaw 18:00, 16 November 2008 (UTC)[reply]
There are a number of cruciferous vegetables which "smell like farts" when you cook them, like broccoli, cabbage, and brussels sprouts, but taste good when you eat them. StuRat (talk) 19:48, 16 November 2008 (UTC)[reply]
Read the article on the durian, and also there's stinky tofu. Thanks. ~AH1(TCU) 19:56, 16 November 2008 (UTC)[reply]
I like a good "stinky cheese" like Gorgonzola or Roquefort or Camembert. A friend of mine, who also likes them, described them thusly "They smell like old sweat-socks, but not in a bad way". That about sums it up. --Jayron32.talk.contribs 21:02, 16 November 2008 (UTC)[reply]
I really like durian in a paste form, but maybe being mixed with other stuff dilutes the smell. There's also shrimp paste or belacan. Julia Rossi (talk) 07:25, 17 November 2008 (UTC)[reply]
There's often specific rules forbidding the possession of durians in places where the smell could cause problems like in airplanes. Dmcq (talk) 09:52, 17 November 2008 (UTC)[reply]
I had a small part of a durian milkshake once. Only once. And only part. Leave the durian for the orang-utans and give me a strawberry smoothie any day. The smell was mostly covered by the sweetener and dairy/thickener, but the taste was horrid. Fresh (and not-so fresh) fish can smell bad, but cooked fish smells delicious, so I don't think that's a great example. It seems to me that the best examples would include foods heavily flavoured by tastes our taste buds can interpret directly (i.e. sweet, salty, sour, bitter, as those might be pleasant flavours that could overwhelm a noxious smell. Matt Deres (talk) 21:17, 17 November 2008 (UTC)[reply]

Medical advice

Where I am now, if you only have the public health insurance, you have to wait some weeks until a doctor sees you. And then he will take only a few minutes - between 3 and 5 - to reach a diagnosis. Would such a diagnosis be more accurate than googling or consulting wikipedia? Are laypersons able to make a better (self-)diagnosis, if they invest more time to understand their illness?--Mr.K. (talk) 18:31, 16 November 2008 (UTC)[reply]

  • Where are you? It sounds like somewhere without any real medical facilities, so is there are treatment available even if you do get a diagnosis? A qualified doctor is likely to know if the diagnosis they reach in a few minutes is reliable or not and they'll know whether or not to refer you on for more tests, but if there aren't any more tests available it doesn't make a lot of difference. --Tango (talk) 18:57, 16 November 2008 (UTC)[reply]
  • First, I doubt that your description applies in general. In nearly all public health systems, emergencies are treated out of order. And the time taken for a diagnosis will depend on the symptoms of your disease, although too little time for this is a very frequent problem in many (not just public) health systems. As for the quality of the diagnosis, I would trust an expert more. Experience counts for a lot, as does formal education. And an individual simply does not have the tools for a proper diagnosis in non-trivial cases, like a laboratory for a complete blood test, EEG and EKG equipment, or culturing. Indeed, an individual probably have trouble doing trivial things like looking into his or her ears or listening for pulmonary sounds. Of course physicians are not perfect, and if a patient has an idea and concrete symptoms, (s)he should mention it to the diagnostician. --Stephan Schulz (talk) 19:13, 16 November 2008 (UTC)[reply]
  • I'd say some combo of self-diagnosis and reliance on a doctor is best. If you have info that might be relevant, tell the doctor, whether they ask for it or not, and also tell them when you think they've made a mistake. For example, if you have symptoms of anemia and the doc prescribes iron and B12, you might want to mention if both your parents had sickle-cell anemia, as you are likely to also have that form. StuRat (talk) 19:44, 16 November 2008 (UTC)[reply]

Do you live in the United States by any chance? Are there any walk-in clinics or family doctors? In Canada we usually get free health care provided that you have a health card. ~AH1(TCU) 20:05, 16 November 2008 (UTC)[reply]

The primary problem with self-diagnosis is that people tend to believe that they have whatever illness they are reading about. Also, people do not usually have the ability to run tests. Do you know what your creatinine clearance is? Do you know what your HbA1c level is? What I mainly see is people who go to doctors they don't like and then schedule another appointment with them. Why? Get a new doctor and then another one and then another one until you find one that you like. -- kainaw 20:25, 16 November 2008 (UTC)[reply]
That's not always an option. In the US, many types of insurance limit the number of doctors you are allowed to see to a small list of "approved doctors", and, in some cases, there may only be one accessible doctor on the list. StuRat (talk) 22:11, 16 November 2008 (UTC)[reply]
Then switch insurance companies? --Tango (talk) 23:51, 16 November 2008 (UTC)[reply]
Tango: That's pretty naive - most people who have health insurance get it through their employer - so they have little or no choice as to what insurer they go with - and therefore, little or no choice as to the doctor they get assigned to. Sure, you could change jobs in order to change insurer in order to change doctors - but that's a pretty drastic step and it's REALLY tough to walk the maze of (a) finding a good doctor, then (b) finding the insurers he/she accepts, then (c) finding the companies that offer those insurers, then (d) getting a job with one of those companies. And that assumes that you have the time to make the change (tough when you actually NEED a doctor) - and it assumes that there aren't a million other constraints that prevent you from changing jobs. So, it's naive to assume that people can fix this - they mostly can't. SteveBaker (talk) 01:50, 17 November 2008 (UTC)[reply]
Ok, emigrate to a country with a sensible health system, then! ;) --Tango (talk) 13:37, 17 November 2008 (UTC)[reply]
Which is an EVEN MORE naive answer! Have you any concept at all of how hard it is to emigrate? Visas, work permits...are mostly unobtainable at any price - then you have to find a job from hundreds of miles away - figure a way to transfer money - find somewhere to live - probably learn an entirely new language...and all of this to get to see a different doctor?! You're living in cloud-cuckoo-land! SteveBaker (talk) 00:42, 18 November 2008 (UTC)[reply]
Wikipedia cannot diagnose a condition. Here on the reference desk we're strictly prohibited from attempting to diagnose or suggest treatment for medical problems. The main articles may give you information on which to base a diagnosis - but if you have some particular symptom, how would you know which pages to look up to find the causes? You have a headache - there are probably 100 or more things that could cause it. What are the odds that you'd find all 100 articles? A doctor, on the other hand is supposed to be aware of all 100 conditions - and can look for secondary or tertiary symptoms that might lead to a more accurate determination. Furthermore - the doctor might have a range of treatment options ranging from asperin to brain surgery - and if there is some uncertainty as to the precise cause of your headaches may be able to come up with a treatment that covers all of the possible causes. Furthermore, a doctor can recommend more studies - blood and urine samples, an X-ray or a CAT scan. You can't do that yourself - so you may be working with only a piece of the information. Also, you are biassed - what may seem to you to be an extreme condition because it's been bugging you for a long time - is something that the doctor can assess dispassionately. So - no - you can't use Wikipedia - that's not the business we're in. SteveBaker (talk) 00:52, 17 November 2008 (UTC)[reply]
The doctor isn't likely to consider the rare causes of headaches, either. Or at least not until many years of unsuccessful treatments for the more common causes. StuRat (talk) 01:32, 17 November 2008 (UTC)[reply]
True - he won't investigate the super-rare things - and as you say - he's not going to do CAT scans looking for brain tumors for every patient who walks through the door complaining of a headache. But a decent doctor ought to at least ask whether you have weird visual problems or an adverse reaction to bright light during the headaches - and thereby suspect a migrane - or he might ask whether they occur in the morning right after waking up - and diagnose sleep apnea. There are a few very common diagnostic things that he might ask that you and your unqualified friends might never think to look up. Services like 'Web-MD' might to a better job - but even so, there is no substitute for an expert - even if you only have his attention for a short period. SteveBaker (talk) 01:45, 17 November 2008 (UTC)[reply]
The doc might also send you home with a prescription and schedule an exam later, but if the problem is a carbon monoxide leak in your home you could be dead by then. If you diagnose the problem yourself, however, by doing web searches based on the symptoms, and call the gas department to investigate, you may survive. StuRat (talk) 03:16, 17 November 2008 (UTC)[reply]
The operational words here being "might" and "may". You might also diagnose yourself with a bacterial infection and take antibiotics until you've cured your harmless but persistent viral infection into a multi-antibiotic-resistant pneumonia. Neither is very likely. --Stephan Schulz (talk) 07:48, 17 November 2008 (UTC)[reply]
That why it's important to combine info a doctor has access to, like medical tests, with info only the patient will think is important, like "Something is seriously wrong here, I've never felt like this before". StuRat (talk) 18:13, 17 November 2008 (UTC)[reply]

Here's an unpopular answer: the doctor may only take 3-5 minutes to look at each patient because most people either don't have something he can help with (viral infection) or have something he can write a prescription for quickly (headache, cough, runny nose, ear ache). In places where the health care system is under a great deal of stress it makes sense to rush those sorts of people in and out, so you can focus your efforts on the ones who really need it. If you find yourself being rushed through the exam, consider for yourself what else a reasonable person could have done for you. How much time do you want him to spend on your sniffles? And, as StuRat says, if you do feel something is very wrong, be sure to impress that upon the physician. Matt Deres (talk) 21:27, 17 November 2008 (UTC)[reply]

cables and auto transformer

kindly help me know the advantages, disadvantages and applications of pvc insulated,pvc sheathed,paper insulated and lead sheathed cables.

what could be the danger that may arise by the the use of an auto transformer on high transformation ratio. —Preceding unsigned comment added by 196.46.196.66 (talk) 18:54, 16 November 2008 (UTC)[reply]

All of those insulators, except lead, are flammable, with paper being flammable at the lowest temps. The paper will deteriorate with age and the PVC will deteriorate with exposure to UV light (such as in sunlight). The lead insulation will be heavier, more expensive, and must be properly disposed of to avoid lead contamination of the environment. One advantage to the PVC is that dyes can be mixed in during production to produce brightly colored insulators that won't fade with time, making identification of various wires far easier. StuRat (talk) 19:35, 16 November 2008 (UTC)[reply]
And isn's lead a decent enough conductor to create its own problems? --Jayron32.talk.contribs 20:59, 16 November 2008 (UTC)[reply]
Lead also has a pretty low melting point for a metal. shoy (reactions) 22:44, 16 November 2008 (UTC)[reply]

The lead would be an outer jacket to keep moisture out. Under it, surrounding each conductor, would be an insulator such as paper or plastic. Oil-impregnated paper insulation with a lead outer sheath to keep out moisture has been used over a hundred years, but may be obsolescent, since it takes highly skilled splicers to make the joints and terminations, and they are exposed to lead fumes. Rubber and plastic insulation has been used since the 1940's. XLPE is cross linked polyethylene and is widely used for distribution voltages such as 12 kV. Semiconductor layers may be used around the insulation to control electrical stress gradient, and there may be copper braid for shielding. What kind of cable is insulated with PVC? I have heard of TW and THHN, among other plastic insulation systems. For the autotransformer question, what do you regard as a "high transformation ratio? An autotransformer has only one winding, and has relatively low reactance. I suppose that might be a problem in some circumstances. See Autotransformer#Limitations where it says what could appear on the low voltage tap if there were an open circuit in the turns it covers. This would be more of a problem if the autotransformer were stepping down high voltage, as at a substation, than if it were stepping up low voltage, as at a generating station.Edison (talk) 00:47, 17 November 2008 (UTC)[reply]

November 17

Water

Is toilet water poisonous? JCI (talk) 00:11, 17 November 2008 (UTC)[reply]

Yes and No.

If by toilet water, you meant sewage then yes.

If by toilet water, you meant clean pipe water used to flush the toilet then no. 122.107.203.230 (talk) 00:38, 17 November 2008 (UTC)[reply]

Before or after you flush? The water comes from the exact same place as the water that comes out of your kitchen - so initially, it's clean and perfectly drinkable (assuming you live in a place where the mains water supply is clean and perfectly drinkable!) The toilet bowl itself is a lot cleaner than some people suspect because it's a hard surface that's not porous and there isn't much in the way of nutrients or sunlight for bugs to live on - and we typically expend an entirely disproportionate effort into sanitizing it. So this boils down to the issues with human waste - not the toilet itself. If it's your own waste that you're ingesting then whatever you're picking up was in your body before - so in small quantities it's not likely to be radically nasty. If the waste came from someone else - it's a different matter. Obviously, this isn't something you'd want to try if you didn't have to - and (of course) you'd have to be aware of any chemicals such as bleaches and toilet cleansers you might have tossed into the toilet or the cystern in advance - which might be extremely hazardous. I think that in an emergency - if there was no other source of clean water - I'd weigh up the pro's and con's and probably risk it. SteveBaker (talk) 00:43, 17 November 2008 (UTC)[reply]
Assuming you have access to the cistern, it contains a tap, just drink the water directly out of the tap and it's exactly like water out of any other tap. --Tango (talk) 01:00, 17 November 2008 (UTC)[reply]

And if you mean toilet water, then yes, some of it is fairly poisonous. -Arch dude (talk) 00:52, 17 November 2008 (UTC)[reply]


(EC)Depending on the type of toilet water, the high alcohol content and perfume and other ingredients such as liquid waxes or cocoanut oil might be harmful, especially if it were any alcohol but [Ethanol]. Edison (talk) 00:54, 17 November 2008 (UTC)[reply]
Ooh! I'd forgotten that meaning of 'toilet water' - yes, that stuff would be horribly bad for you. However, it's been a very long time since I last heard anyone describe perfume as 'toilet water'...for reasons that are fairly obvious! SteveBaker (talk) 01:35, 17 November 2008 (UTC)[reply]
Hah! Caught Steve in an area he knows nothing about. Strictly speaking (imagine the French accent, please), Perfume (as in eau de Parfum) is not really the same as eau de toilette - it's much more concentrated. --Stephan Schulz (talk) 07:57, 17 November 2008 (UTC)[reply]
People almost never translate "eau de toilette" into English, though. They just use the French. --Tango (talk) 11:36, 17 November 2008 (UTC)[reply]
"Both dogs and people occasionally splash toilet water on their faces, but dogs seem to enjoy the scent more." StuRat (talk) 03:09, 17 November 2008 (UTC)[reply]
"I've got a headache. I was dabbing some of that fancy toilet water behind my ears just before the show, and the lid came down and hit me on the back of the head." [Little Jimmy Dickens, Grand Old Opry] Edison (talk) 19:52, 17 November 2008 (UTC)[reply]
@SteveBaker - Dude, if you ever really do get that desperate, please at least consider going for the water in the tank rather than the water in the bowl... Matt Deres (talk) 00:26, 18 November 2008 (UTC)[reply]

Geology

What are the three types of stress force —Preceding unsigned comment added by 65.12.161.166 (talk) 01:01, 17 November 2008 (UTC)[reply]

Please do your own homework.
Welcome to the Wikipedia Reference Desk. Your question appears to be a homework question. I apologize if this is a misinterpretation, but it is our aim here not to do people's homework for them, but to merely aid them in doing it themselves. Letting someone else do your homework does not help you learn nearly as much as doing it yourself. Please attempt to solve the problem or answer the question yourself first. If you need help with a specific part of your homework, feel free to tell us where you are stuck and ask for help. If you need help grasping the concept of a problem, by all means let us know.
May I suggest Structural geology - I believe the answer you seek is in one of the articles linked from it. Better still - I'm 100% sure that if you crack open the textbook for your course, you'll find the answer listed neatly in the chapter you've just been working on in class. SteveBaker (talk) 01:33, 17 November 2008 (UTC)[reply]
Can't we just start giving them wrong answers? The three types of stress are work stress, family stress, and Microsoft-related stress which is the worst stress of all. 216.239.234.196 (talk) 16:41, 17 November 2008 (UTC)[reply]

Home battery recycling

Is there any way to recycle or revive sealed lead acid or lithium batteries at home, for an electric bicycle for example? And are there any batteries that are self-recycling? And what chemical or thermal process would be involved after I wash the chemical out of the battery? Trevor Loughlin (talk) 05:30, 17 November 2008 (UTC)[reply]

There is nothing chemical or thermal that a non-specialist can do to fix a battery. "Deep cycling", i.e. charging a rechargeable battery followed by mostly discharging it under steady load and then recharging it again, is an electrical process that can help some rechargeable batteries under some conditions. Though with lithium rechargeables it is important that they never be completely discharged as this will almost certainly tend to shorten their life. Opening a lead acid battery is dangerous. The acid is corrosive and the lead toxic. Lithium batteries are only a little better as lithium will burn skin and the dust should not be inhaled. If the batteries are really dead send then to a professional recycler, don't mess with them yourself. Dragons flight (talk) 07:05, 17 November 2008 (UTC)[reply]

What killed the airship?

You know, dirigibles, zeppelins, etc... How did they come to be surpassed by contemporary jumbo jets? Kenjibeast (talk) 06:58, 17 November 2008 (UTC)[reply]

The Hindenberg disaster killed public opinion. Plus they are generally impractical. Dirigibles are huge, slow, and hard to operate compared to jumbo jets of the same cargo capacity. They are probably a reasonable solution to aerial surveillance of a fixed location (e.g. Goodyear blimps above sporting events), but they are not competitive as a means of transportation. Dragons flight (talk) 07:14, 17 November 2008 (UTC)[reply]
See also Airship#Practical_comparison_with_heavier-than-air_aircraft for a rather lengthy discussion of why airships were abandoned. SDY (talk) 07:48, 17 November 2008 (UTC)[reply]
Airships were not "surpassed by contemporary jumbo jets", though. They were abandoned about the time World War II started, and jet aircraft didn't even exist then, let alone jumbo jets, which came along 25 years after the war. But propeller aircraft did, and the war stimulated their development in a big way... and a bomber and an airliner can have many similarities in design. --Anonymous, 06:22 UTC, November 18, 2008.

Airships again

What was the operating altitude for the USS Akron and the Hindenberg and other airships from that era? What are the practical considerations for a high-altitude airship (i.e. how much does the lifting power drop as atmospheric pressure decreases)? SDY (talk) 07:48, 17 November 2008 (UTC)[reply]

In the airships link above, it states: "The highest flight made by a hydrogen filled passenger airship was 5,500 feet (1,700 m) on the Graf Zeppelin’s around the world flight. The practical limit for rigid airships was about 3,000 feet (900 m), and for pressure airships around 8,000 feet (2,400 m)." And a rigid airship in 1917 "was forced to 24,000 feet", but it crashed as a result. -- MacAddct1984 (talk &#149; contribs) 18:31, 17 November 2008 (UTC)[reply]
The practical operating altitude was "as low as possible": the Graf Zeppelin typically flew within 500 feet of the ground. Since hydrogen airships were typically operated with their lift cells as full as possible, gaining altitude meant venting off hydrogen to keep the lift cells from rupturing from overpressure -- and venting hydrogen means you also need to drop ballast to keep the airship aloft. When you run out of ballast, you either need to start dropping cargo, or you need to land where you are, neither of which is a desirable situation.
Staying low has other benefits, as well. Turbulence is minimized, which is important when you're flying something larger than the typical turbulence cell, and downdrafts are mostly converted into horizontal winds. It also gives the passengers a great view of whatever you're flying over. --Carnildo (talk) 01:04, 18 November 2008 (UTC)[reply]

Explain this optical illusion?

I was reading the Wikipedia article on The Fall of the House of Usher. In the "Analysis" section there is an image of a copy of the magazine in which the story was first printed. I had this image at the bottom right of the browser window and was reading the end of the preceding section when I noticed something odd: when, in reading, my eyes move left to the beginning of the next line, the image appears to momentarily take on a distinctly green tinge. Why is this? - 79.79.176.53 (talk) 08:59, 17 November 2008 (UTC)[reply]

This can happen particularly if red green and blue images are shown in sequence and one moves ones eyes. Some projector systems in particular suffered from it. I would have thought there wouldn't be too much problem with LCD monitors though. Dmcq (talk) 09:48, 17 November 2008 (UTC)[reply]
Seems you could be experiencing the after image phenomenon, due to the red cast of the image in question. Does that fit what you are seeing? --Scray (talk) 11:28, 17 November 2008 (UTC)[reply]

Jupiter causes an additional parallax

Stellar distances are measured most directly by parallax, using the fact that Earth's position changes by 3*108 km within 1/2 years. Now of course when planets move around the sun, the sun itself also moves (but of course slower and in a smaller ellipse), and Jupiter causes the Sun's position to change by about 1.5 million km within 6 years. Earth orbits Sun in a close orbit compared to Jupiter, so it can be expected to follow this movement of the Sun (in addition to orbiting it). This should produce an additional parallax of about 7 milliarcseconds at α Centauri - this should be measurable e. g. with Hipparcos. Does this additional parallax have a special name? Maybe Wikipedia's article should mention it.

Additionally, Jupiter causes the velocity of the Sun to change by an amount of more than 20 m/s within 6 years, and the resulting Doppler effect should also be measurable when observing other stars.

Icek (talk) 09:57, 17 November 2008 (UTC)[reply]

Smoking and breast milk

Maternal smoking results in exposure to nicotine in breast-fed infants. Smoking mother's breast milk contains nicotine - when this fact was first discovered and who discovered it? Otolemur crassicaudatus (talk) 10:06, 17 November 2008 (UTC)[reply]

Who the heck smokes breast milk? Isn't too wet to ignite? I mean, I suppose you could get it into a bong, so that would eliminate the soggy paper problem, but wouldn't you still have problem lighting it? I suppose you could freeze-dry the breast milk, and smoke that, but it seems like a lot of trouble to go through... --Jayron32.talk.contribs 15:32, 17 November 2008 (UTC)[reply]
I have asked the breast milk of the mothers who (who denotes mother) smoke cigarette contain nicotine. It not the breast milk which the mother smoke, it is the cigarette which the mother smoke. Now if a mother smokes cigarette during her maternal period, it is observed that her breast milk may contain nicotine. I want to know who first discovered this and when (i.e. relation between maternal cigarette smoking and consequential nicotine concentration in breast milk)? Otolemur crassicaudatus (talk) 16:07, 17 November 2008 (UTC)[reply]
No shit, sherlock... I was being funny. You used the phrase "Smoking mother's breast milk" which is an example of a Dangling modifier since it is impossible to determine from the phrase which word "smoking" is intending to modify. Is the mother, the mother's breast, or the mother's breast milk the thing which is smoking. Such imprecise language is often the source of humour; phrases like "Eats, Shoots & Leaves" or "man eating cabbage" or "British Left waffles on Falklands" are other famous examples of imprecise language. Damn it, jokes are not as funny when you have to explain them... --Jayron32.talk.contribs 17:07, 17 November 2008 (UTC)[reply]
See also Amphibology. --Jayron32.talk.contribs 17:14, 17 November 2008 (UTC)[reply]
To be honest, it wasn't all that funny to start with... ;) If you're going to mock someone's imperfect but completely unambiguous (in context) grammar, it helps to answer the question as well. If you don't know the answer, keep quiet. --Tango (talk) 17:22, 17 November 2008 (UTC)[reply]
Alright smarty-pants. I did a google scholar search where the earliest reference I can find is this study done in 1933 by someone named WB Thompson, and appearing in American Journal of Obstetrics and Gynecology. Several other studies cite a 1942 JAMA paper as being fairly authoritative on the subject. Looking through the first 3-4 pages of the Google Scholar search turns up nothing earlier than 1933, but that doesn't necessarily mean there wasn't anything earlier. The online version of the 1933 paper is only an abstract; if you got the original you would probably find at least a cursory literature review, and you could see if there was anything earlier. Going backwards through the papers, you could probably find the seminal study. --Jayron32.talk.contribs 18:33, 17 November 2008 (UTC)[reply]
This study: [16] seems to cite the 1933 Thompson study, and its title "The Excretion of Drugs in Human Milk—A Review" would seem to indicate that it would be a great place to start if looking for that first study... --Jayron32.talk.contribs 18:35, 17 November 2008 (UTC)[reply]
Here's one from 1927: [17], and IT cites studies from as early as 1908. Again, I would start somewhere like this one, and work backwards until I got to the earliest study to mention it. It may take time; however access to a university library would make it possible to work it out. here's one from 1911 that talks a bit about the mechanisms of milk secretion, and the action of certain drugs thereupon. --Jayron32.talk.contribs 18:44, 17 November 2008 (UTC)[reply]

toilet seats

Assuming it is used normally, is it actually possible to catch any disease from a toilet seat? Reading transmission (medicine) suggests that it is not.--Shantavira|feed me 10:11, 17 November 2008 (UTC)[reply]

Per this, toilet seats are relatively germ-free; an average toilet seat contains approximately 49 microbes per square inch compared with a keyboard, which may contain 25,127 microbes per square inch. Toilet seats are not common transmission channels of infectious diseases. To infect someone, the bacteria must have to enter his body through mouth, anus or urethra. But the bacteria cannot jump from the toilet, so someone using the toilet in usual manner is basically safe. But, yes there are possibility of skin diseases like scabies, herpes. Regarding Gonorrhea, a study in 1979 titled The gonococcus and the toilet seat claimed "nonsexual transmission from toilet seats is not impossible, just very unlikely". Here is an interesting reference which will help you a lot. Otolemur crassicaudatus (talk) 10:47, 17 November 2008 (UTC)[reply]
You really ought to be more worried about door-handles than toilet seats, especially in public bathrooms. Think about it, thousands of people touch those things, and many probably don't was their hands properly (if at all). You get it on your hand, and then you eat a popcorn or something, and bam!, you're infected. Much bigger hazard than toilet-seats. 83.250.202.208 (talk) 11:53, 17 November 2008 (UTC)[reply]
I wonder if there's a greater transmission problem with the widespread use of auto-flush toilets? I find they tend to perform quite a few, er, premature evacuations. On the one hand, the extra flushing presumably helps whisk away any residual nasties in the water; on the other hand, it tends to deposit a fine coating of "toilet dew" on one's nether regions, thus overcoming the bacterial obstacle of migrating from bowl to anus. Matt Deres (talk) 00:34, 18 November 2008 (UTC)[reply]

Red / Blue Shift

Apparently astronomers can determine the distance to a star or galaxy by its red shift or blue shift. Do we have any concrete proof, such as from experiments that can be replicated in a laboratory, that we are correctly measuring the distance to these celestial bodies, ie the red / blue shift measurement is accurate? Or is it all based upon theories and resulting formulae? —Preceding unsigned comment added by 75.67.217.220 (talk) 10:41, 17 November 2008 (UTC)[reply]

Ummm, well all science is based on a combination of evidence, experimentation, and theories, so I'm not sure what you are really asking. However, as discussed at cosmic distance ladder, a wide variety of techniques are used to validate measurements of cosmological distances and the agreements between different types of measurements can be seen as evidence that astronomers know what they are doing. Dragons flight (talk) 11:20, 17 November 2008 (UTC)[reply]
You don't measure distance with redshifting or blueshifting, you measure if something is moving towards you (blueshifting) or moving away from you (redshifting). As for concrete proof or a laboratory experiment confirming this effect, we absolutely have it! In fact, you can observe it for yourself! The principle behind it is something called the Doppler effect, and it happens with all sorts of waves, including sound-waves. Next time you're stuck in traffic, and hear an ambulance or a police-car rushing at great speed near you, think about how the pitch of the siren changes as the ambulance (or police) passes by you. It will be at a much higher pitch when approching you, and then as it passes you, it will suddenly have a much lower pitch. This is exactly what happens to light when they are emmited from stars that are travelling away from you. Instead of having a lower pitch, it will be slightly shifted to red side of the color-spectrum. 83.250.202.208 (talk) 11:50, 17 November 2008 (UTC)[reply]
"You don't measure distance with redshifting"? Doesn't Hubble's Law say that "the redshift in light coming from distant galaxies is proportional to their distance"?--Shantavira|feed me
Yes, for sufficiently distant galaxies, redshift gives you an pretty good approximation of their distance, but only by comparing them to closer galaxies that you can measure the distance to via different methods (see the article Dragons flight linked to). --Tango (talk) 13:29, 17 November 2008 (UTC)[reply]
Yes, I should've been clearer about that point. I just meant that that wasn't what you were measuring directly, and without a reference it doesn't work. 195.58.125.52 (talk) 13:55, 17 November 2008 (UTC)[reply]

Tricks by Nikola Tesla in the movie "The Prestige"

I just rewatched the brilliant film The Prestige, and had some questions regarding some of the science-tricks that Nikola Tesla performed.

  1. In one scene, Andy Serkis (Tesla's Igor, basically) takes Hugh Jackman out to a huge snowy field, with hundreds of lamps that are just sitting in the snow. Hugh Jackman asks "But where are the wires?". Andy Serkis replies "Exactly." (this particular scene is illustrated in the article, under the production heading). What is going on here? Has Tesla somhow electrified the snow? If so, why would the electricity pass through the light-bulbs, instead of going straight into the ground? The only plausible thing I can think of is that he has an enourmous electro-magnet nearby that generates a fluctuating magnetic field and there are some sort of generator in every one of the lamps (I haven't studied these things since high school, so I'm completely just guessing here). But Hugh Jackman lifts one of the lamps off the snow and then it turns off, but when he puts it back, the lamp turns on again. If anyone could explain that scene to me, I'd be mighty grateful.
  2. In another scene (also helpfully illustrated in the article, wikipedians are apparently obsessed with Tesla) the great engineer walks underneath a Tesla Coil that's shooting bolts like crazy, apparently completely unharmed (the man knows how to make an entrance). I understand the principle here, that Tesla is wearing insulating shoes and so the electricity doesn't pass through him, but this is still got to be hugely dangerous, right? I mean, the air in that chamber where he walks has to be extremely hot from all the electricity, and what if a bolt passes through him and jumps through the air from his ankles down to the floor (the floor is definitely conductive, you see sparks hitting it all the time). I mean, this is not something a sane person would do, right?
  3. After he has made his glorious entrance through the storm of lightning-bolts, he walks up to Hugh Jackman and grabs his hand, and hands him a lightbulb. Still clasping hands, Jackman picks it up with his other hand, and lo!, the bulb lights up (seriously, now he's just showing off). Asking where the electricity comes from, David Bowie in his best Serbian accent says "Ze body is a most excellent conductor!". Again, what's going on here? Has Tesla become so charged by walking through the lightning-bolts that he can keep a lightbulb on for several seconds, even after the electricity has passed through two bodies? And wouldn't the charge pass through Jackman's body straight into the ground (wouldn't that be the path of least resistance?), completely skipping the light-bulb?

I realize that this is a film with science-fiction elements in it, but I would expect the scenes introducing Tesla to be somwhat grounded in reality (as he was a really cool guy who could do really cool tricks with electricity, so why would you go the sci-fi route?). Can anyone explain this to me? 83.250.202.208 (talk) 11:41, 17 November 2008 (UTC)[reply]

Let me try to answer these the best I can...
  1. He used a Tesla coil to power them. (wonder where THAT name comes from... hm...) A Tesla coil generates a strong electric field around it. Basically, a Tesla coil is a really freaking powerful broad spectrum radio antenna. Radio works by inducing an electric current in your reciever, but the current is small; it needs to be "amplified" by an external current in order to have enough energy to drive the speakers. If you drive enough static charge into the atmosphere, you can induce a current in the same way in just about any metal availible.
    You can do the experiment yourself relatively simply. If you have an AM radio tower anywhere nearby, take a simple hand-held transistor radio and get right underneath the radio tower. You will hear the broadcast from the tower without any batteries in your radio at all. At this distance, the power of the antenna is strong enough to power the circuit without amplification.
    The same principle works in a tesla coil; it will provide enough electrical potential in the air itself to power lightbulbs just inside the field, without any wires. Tesla had some pretty wild ideas about using these for civilian power generation. Basically, you'd build on in the town square, and everyone in town got free electricity. The downside is that these things have a nasty habit of discharging into anyone that gets too close, so it basically acted like a giant bug zapper that takes down birds, pets, wandering drunks, etc. etc.
  2. Is likely bullshit, but only slightly. He could wear a "conductive suit" like linemen wear when working on high-tension wires, they act like a personal faraday cage, directing current away from themselves into the ground.
  3. Is more likely total bullshit. I can't imagine that his body would act as a good enough capacitor to hold a charge long enough to do that. --Jayron32.talk.contribs 14:02, 17 November 2008 (UTC)[reply]
But here's my question about 1. Hugh Jackman picked up the lightbulb from the snow, and it turned off. Then he put it down again, and it turned on. I.e. it didn't work when it was in the air, it only worked when it sat in the snow. Wouldn't your Tesla coil electric field also work when the lightbulb was in the air? Or is the electric field somehow confined to the ground? 195.58.125.52 (talk) 15:37, 17 November 2008 (UTC)[reply]
Ah, I see the problem. No, then, that part is bullshit too... --Jayron32.talk.contribs 18:22, 17 November 2008 (UTC)[reply]


2) There is a very famous photograph of Tesla sitting under the coils in his laboratory. It's fake (double exposure), but most people don't realize that. (or don't care.) Virtually every portrayal or parody of Tesla recreates this photo somehow. 72.10.110.109 (talk) 14:27, 17 November 2008 (UTC)[reply]
Tesla demonstrated in the 1890's that if you set up a strong enough AC field in a room, specially built light bulbs would light without wires connected. This works for glow tubes like neon lights or Geissler discharge tubes, and even for fluorescent tubes. This was back before they worried about the effects of EMF on living things. Back then, they did not even know that X-Rays were harmful. Edison (talk) 19:47, 17 November 2008 (UTC)[reply]
Of course User:Edison would play up the danger of Tesla's research. 72.10.110.109 (talk) 20:23, 17 November 2008 (UTC)[reply]
Note that User:Tesla has made no contributions.Edison (talk) 00:57, 18 November 2008 (UTC)[reply]
There was an art installation in the UK by artist Richard Box back in 2004. The installation, called Field, placed more than a thousand fluorescent tubes on end under some long-distance high-voltage transmission lines. The electric field (and electric field gradient) below the wires was sufficient to light the tubes. Have a look: [18]. TenOfAllTrades(talk) 20:50, 17 November 2008 (UTC)[reply]

Box "discovered" this in 2004? It was demonstrated by opponents of overhead high voltage lines back in the 1970's, and had been demonstrated by Tesla and others back in the 1890's. There is a danger if the tubes are raised too high in the air, since they conduct electricity and if brought too near an energized conductor they could electrocute the person holding them. The conductors are elevated for a reason. Edison (talk) 00:51, 18 November 2008 (UTC)[reply]

Earth-like planet forming in the absence of life

In 2006, someone asked an interesting question on the talk page for the terraforming article:

http://en.wikipedia.org/wiki/Talk:Terraforming#Earth-like_planet

The question hasn't gotten much attention there and I'm very interested in it, so I'm asking it here: is there a way for a planet with conditions that could support human life (oxygen/nitrogen atmosphere, liquid water, appropriate gravity and atmospheric pressure etc) to form in the absence of life? It might be really unlikely, far-fetched and rare, but it's a huge galaxy and an even huger universe, so surely it's happened somewhere.

The critical thing here would be the generation of oxygen. Oxygen has to be constantly replenished on Earth because it reacts so well with many other elements and so gets locked up in rocks, but perhaps there could be some complex geological process that generates large amounts of oxygen. If the oxygen is too far fetched, what about a planet with otherwise Earth like temperature and pressure conditions but no oxygen so that it would be possible to go outside with only a tank of oxygen? Sure, it'd be limiting, but it's a start. 63.245.144.68 (talk) 13:11, 17 November 2008 (UTC)[reply]

I agree, oxygen is the critical thing. Without the oxygen requirement, it's easy - the Earth during the Archean period fits your description quite well (there was some primitive life, but I don't think it had had time to change the planet much - I may be wrong). --Tango (talk) 13:36, 17 November 2008 (UTC)[reply]
Only if you aren't too picky about pressure, speculation generally puts Archean atmospheric pressure at 3-10 times modern. Though one could almost certainly come up with inorganic ways to make a pressure closer to modern, so that probably isn't an impediment to the question being asked. Dragons flight (talk) 14:11, 17 November 2008 (UTC)[reply]
I've moved your edit conflict message to your own comment - I didn't conflict with myself! ;) A human could survive in 3 atm pressure with no problem, and probably 10 atm too (moving between that and 1 atm requires some care, of course). I think the OP is just looking for habitable planets for colonisation, so that qualifies. --Tango (talk) 14:36, 17 November 2008 (UTC)[reply]
That's not my EC note. It was already there before I got here. Dragons flight (talk) 15:07, 17 November 2008 (UTC)[reply]
Ah, it was Jayron's, my bad! I'll move it again! --Tango (talk) 15:45, 17 November 2008 (UTC)[reply]
(EC with tango) From a purely chemical/thermodynamic perspective the presense of free oxygen is an impossibility in any system which does not compensate in some way. If you look at almost any system with the following equilibrium:
The numbers skew SO FAR to the right for most elements that it is an impossibility for any element to exist in the presence of oxygen WITHOUT producing its oxide. The half-dozen or so elements it doesn't work for (copper, silver, gold, platinum, maybe one or two more) are not going to exist in significant amounts in any plantary system. The reaction is always spontaneous and highly exothermic. There's usually a high activation energy to this reaction, but that really only effects the rate of reaction. Given earth-like temperatures, and given a substantially long-enough time frame (say measured in the thousands of years) free oxygen will just stop existing. In fact, I can't come up with any spontaneous chemical process that will generate free oxygen from something that does NOT involve life in some way... --Jayron32.talk.contribs 13:41, 17 November 2008 (UTC)[reply]
It may actually be possible to have abiotic free oxygen production; this is a major issue for extra-solar planet spectroscopy missions like TPF and Darwin. For example, in (Selsis, F, Despois, D & Parisot, JP 2002, "Signature of life on exoplanets: Can Darwin produce false positive detections?", Astronomy & Astrophysics, vol. 388, no. 3, pp. 985-1003), it is estimated that a maximum equilibrium concentration of about 5% of can be maintained through photochemical processes. The actual photochemical equations (mainly revolving around photo-disassociation of and ) are apparently quite complicated (the source listed above includes about 150 in their model). I don't think they considered surface processes since these are potentially very variable; there are huge differences between a water-rich planet covered entirely by ocean and a water-poor planet with lots of geological activity (recycling the surface often). 58.96.70.254 (talk) 15:59, 17 November 2008 (UTC)[reply]

So if we were to find a planet with otherwise habitable conditions but with no oxygen, how difficult would it be to add the amount of oxygen necessary to support animal life? Industry on Earth is putting out massive amounts of gases into the atmosphere and is demonstrably changing the composition of it, so wouldn't it be possible to do the same on another planet by perhaps separating oxygen from oxide compounds in the crust and releasing it into the atmosphere? 63.245.144.68 (talk) 20:30, 17 November 2008 (UTC)[reply]

No, the amount of energy required to generate an oxygen atmosphere directly is ridiculously large. Completely unapproachable by technological means. The global microbial biosphere powered by the sun was able to do it, and any terraforming project is likely to have to rely on the same kind of process. Dragons flight (talk) 22:04, 17 November 2008 (UTC)[reply]

I think there are two separate issues here:

  1. Do all planets with the right chemical and thermodynamic environment to produce life necessarily do so in a reasonable time-frame?
  2. Do all planets which DO produce life inevitably see it evolve to produce intelligent civilisations?

Neither question is very well understood.

Sadly, science hasn't quite figured out the precise step that lead from "lifeless" chemicals to replicating and evolving lifeforms. Personally - I feel that the oceans are so enormous - and the amount of time involved is so immense that sheer statistics means that sooner or later a replicating molecule would appear - and from that point onwards, evolution takes hold and you have lots of life forms. But the precise chemistry of the situation might mean that this spontaneous formation of a small replicating molecule could take (on average) a trillion years of random chemistry - in which case we are an amazing flook that may have happened only once in the observable universe...or perhaps the statistics works out such that these molecules form within a matter of months - and absolutely every planet that's remotely capable of supporting life must therefore be crawling with the stuff. Which of these things is the truth is hard to know because we don't really know how simple life can be and still cause evolutionary change. The folks who are working on creating completely synthetic life-forms may well have an answer to that within a very few years...and they we'll have a better answer for this one.

If life is everywhere and evolution is in top gear - will there necessarily be intelligent civilisations? I kinda doubt it. The dinosaurs were around for a vastly longer time than the mammals have ruled the place - and they didn't manage to evolve this ability...presumably because there was no advantage to them in having it. So again - it might take a highly unlikely set of environmental pressures to make intelligence worth the price you have to pay for it in reproductive terms.

So I've gotta conclude that a planet with the perfect conditions for forming life might not ever do so - and even if it did, it's not obvious that it wouldn't be locked into a particular evolutionary path that does not favor intelligence. It's not certain that this is a true statement - but there is currently zero evidence that it's not true.

SteveBaker (talk) 22:18, 17 November 2008 (UTC)[reply]

While those are two interesting questions, I don't think either of them are what the OP is asking. I think the OP is talking about finding planets suitable for colonisation, not finding extraterrestrial life. --Tango (talk) 00:55, 18 November 2008 (UTC)[reply]

information on the temperature and precipitation in the BOREAL SHEILD ecozone

information on the temperature and precipitation in the BOREAL SHIELD ecozone doesn't exist can someone add this info soon? —Preceding unsigned comment added by Shadow56121 (talkcontribs) 14:27, 17 November 2008 (UTC)[reply]

Neolithic period and a Young Earth

Without wanting this to turn into a religious debate, do those who adopt a Young Earth ignore the wealth of archeological evidence from the Neolithic period? While I know YECs are hesitant to accept radiometric dating, surely there must be enough archeological evidence to construct a fairly accurate timeline? -- MacAddct1984 (talk &#149; contribs) 17:27, 17 November 2008 (UTC)[reply]

You might have more luck on the Humanities page, as the beliefs of creationists have no basis in science. StuRat (talk) 18:08, 17 November 2008 (UTC)[reply]
On the contrary, there is no desire of adherants to any form of creationism to attempt to construct a coherant theory from evidence. Evidence is either modified or discarded as needed to fit the pre-existing theory; its a "cart-before-the-horse" method of "science" which is why no one with any real understanding of how science works takes it seriously. They don't derive the age of the earth from availible evidence, and modify that age as new evidence arises; they START with the date in mind, and bend the interpretations of the evidence to fit that date. --Jayron32.talk.contribs 18:20, 17 November 2008 (UTC)[reply]
I guess what I'm actually asking is: is there significant archeological evidence of the Neolithic period where a timeline can be formulated without having to resort to radiometric (carbon) dating? Which, as StuRat said, may be more of a question for the humanities page. -- MacAddct1984 (talk &#149; contribs) 18:28, 17 November 2008 (UTC)[reply]
I'd imagine that some evidence from dendrochronology could be used for events that are that recent (i.e. <10kya), though preserved wood construction from the neolithic is either very limited or virtually nonexistent. SDY (talk) 18:37, 17 November 2008 (UTC)[reply]
Proponents of the Young Earth theory like to ignore radiocarbon dating evidence not because of anything inherently wrong with the methodology of radiocarbon dating, but because it provides evidence that is in conflict with their pre-conceived date as to the birth of the Earth. If you were to present to them any other method which arrived at the same results as radiocarbon dating, they would dismiss it as a flawed method because it arrives at a different date than the exact one that they have in mind. You cannot use reason to talk someone out of a conclusion they did not use reason to arrive at in the first place! --Jayron32.talk.contribs 18:51, 17 November 2008 (UTC)[reply]
Indeed. While YECs come up with all kinds of complicated explanations for why carbon dating, etc. get the "wrong" results, those explanations are purely intended for us non-believers, they aren't the reasons the YECs believe what they do. The believe what they do because that's what they believe (probably because that's what they were told to believe by their parents and they never questioned it until it was too late and it was already so deeply ingrained in their psyche that they can't believe anything else), not because they have been convinced by the arguments they try to convince us with. --Tango (talk) 19:41, 17 November 2008 (UTC)[reply]
They seem to blame any archeological evidence that doesn't mesh with YEC as an artifact of (or 'proof' of) the Great Flood. They often don't call it the great flood, though. They come up with adorably pseudo-scientific ways of describing a giant, world-covering 40 day flood without actually sounding like they're talking about a biblical event. 72.10.110.109 (talk) 20:20, 17 November 2008 (UTC)[reply]
How much water would it take to flood the entire Earth? Assuming all the clouds rained until there were no clouds, would that be enough water? 216.239.234.196 (talk) 21:13, 17 November 2008 (UTC)[reply]
Not even remotely. To flood the earth, you would need about 8500 meters of water. 10 meters of water is about one atmosphere of pressure. Hence, for that much water to be suspended in the air, whether as clouds or as vapor, would cause about 850 atmospheres worth of extra pressure. In other words, it would require a major miracle... --Stephan Schulz (talk) 21:20, 17 November 2008 (UTC)[reply]
At any one time, the atmosphere collectively only holds enough water to raise sea level a few centimeters. You would need some totally different source of water to flood the Earth. Dragons flight (talk) 21:21, 17 November 2008 (UTC)[reply]
Out of curiosity, how much depth of water is currently on the earth that is not in liquid form? In short, if you melted all of the ice caps and glaciers and took the atmoshpere as well, could you get the 8500 meters necessary to cover Everest? SDY (talk) 21:53, 17 November 2008 (UTC)[reply]
According to freshwater, about 2% of the earth's water is ice. According to ocean, that means melting all the ice would give you about 25 million cubic kilometres. That's enough to raise the sea level by about 70m; a pretty significant increase, but a long, long, way from the top of Everest. The atmosphere gives you almost nothing. Algebraist 22:01, 17 November 2008 (UTC)[reply]
Well - let's be a little bit careful. Sure, there isn't enough water vapor in the air to do that. But we believe that if all of the ice and snow in the world melts - the oceans could rise 20 meters. Because water gets less dense as it gets warmer, if the climate heated up a lot - the water would expand and get quite a bit deeper than that. As the continents get inundated, the downward force of all of that water pushes the continents downwards - and that makes for still less visible land. It's not THAT unreasonable to assume that so much of the land would be underwater that a family with a fricking ENORMOUS boat and a lot of very fractious animals might imagine there was no land left anywhere! But that's OK - every other aspect of that story is ridiculous. SteveBaker (talk) 21:57, 17 November 2008 (UTC)[reply]
Ah, ah, ah! Your calculations are based on the earth being a spheroid. I think you'll find things work out just fine when you correct it to the proper shape. Look at all that extra water there; just imagine where it must end up. Matt Deres (talk) 00:45, 18 November 2008 (UTC)[reply]
It ends up...on....the.....back......of......the turtle! OH OF COURSE! Now it's all so clear. SteveBaker (talk) 05:17, 18 November 2008 (UTC)[reply]
The problem with creationists is that they know full well that if they spell out their beliefs carefully enough - they'll become testable and falsifiable. That's something they don't like to risk - so they have to keep it vague or science will crush their puny belief system under a large pile of rock-solid reasoning. So long as you keep saying "God did it" - then it's hard for science to pin it down. The conversation generally goes something like: "Why are there fossils of damned great dinosaurs - but no sign of any actual dinosaurs on earth right now?"..."God did it". "How come we can trace a clear history of human and proto-human ancestry back through the fossil record and back to the common ancestor of chimpanzees and man?"..."God did it". "Why does the cosmic background radiation appear so smooth - indicating a common point of origin for all of the universe?"..."God did that too". "Why the heck did God do such bizarre and unnecessarily annoying things?"..."To test our faith in him". QED. Where it gets tricky for them is when they want to get this doctrine taught in US schools in defiance of the constitutional separation of church and state - so instead we get "creationism-lite" - or "intelligent design" - where they say all the exact same things - but instead of "God" they say "some intelligent designer" (except when they slip up and say "God" by mistake). Sadly, this eliminates the "to test our faith in him" defense - so there have to start being some actual reasons - and that's where things start to get messy. SteveBaker (talk) 21:57, 17 November 2008 (UTC)[reply]
I've always wondered about those Young Earth Creationists you see occasionally who have real degrees from respectable schools. What do those people really believe? Assuming they're sincere, they must experience enormous mental dissonance every time some new fact comes out from the various fields they believe to be arrayed against them (biology, geology, astronomy, paleontology, archaeology, etc.). How do they reconcile that almost all natural evidence seems to point them away from something they believe? --Sean 13:19, 18 November 2008 (UTC)[reply]
I know of one case of a quite senior scientist from a very different field of science. He looks at very narrow bits of evidence and picks them apart until he can find a torturous interpretation that is in principle compatible with YEC. He ignores all the other evidence and the interconnections that make a mature field of science so resilient. If you read his texts on the topic, you get the feeling that he consciously and painfully avoids thinking one step beyond his immediate goal of dissecting just one single bit of evidence. --Stephan Schulz (talk) 13:40, 18 November 2008 (UTC)[reply]

The Laws of proportioning aggregates for concrete

Could anybody hep me find article by Fuller, W.B. and S.E. Thompson. The Laws of Proportioning Concrete. Journal of Transportation Division, American Society of Civil Engineers, Vol. 59, 1907. Thank you. —Preceding unsigned comment added by 202.70.74.132 (talk) 17:45, 17 November 2008 (UTC)[reply]

It sounds like you've found it! An article written in 1907 is not likely to be online - you'll probably have to take a trip to the library and get a copy on interlibrary loan. SteveBaker (talk) 21:37, 17 November 2008 (UTC)[reply]

comparative world livestock populations

Do we have a world list of livestock by quantity? i.e. how many domesticated pigs, goats, sheep, cattle, Bactrian camels, Arabian camels, donkeys, yak, horses, llamas, etc. are alive in the world today?--206.248.172.247 (talk) 17:57, 17 November 2008 (UTC)[reply]

This list provides information on cattle population by country, however it is not complete list. Otolemur crassicaudatus (talk) 18:06, 17 November 2008 (UTC)[reply]
Here is a good reference [19]. Click the arrow button to navigate to the other pages for information on other animals. Otolemur crassicaudatus (talk) 18:14, 17 November 2008 (UTC)[reply]

Compost pile

What happens if you put diseased plants into a compost pile? —Preceding unsigned comment added by 189.58.24.177 (talk) 20:27, 17 November 2008 (UTC)[reply]

That would depend on the disease. Some diseases might be specific to that type of plant or might be destroyed by the decomposition process, while others could possibly infect other plants which have the resulting compost spread around them. StuRat (talk) 20:34, 17 November 2008 (UTC)[reply]
If the compost gets "hot enough" during the decomposition process, it can kill disease organisms. However it is hard to be sure if your compost gets "hot enough". The cautious approach is not to put diseased plants into your compost system. CBHA (talk) 21:08, 17 November 2008 (UTC)[reply]

Math and physics podcasts

I've already found Mathgrad, Math mutation, Math factor, and Discrete Mathematics, but I'm having a harder time finding good physics podcasts. Could anyone please recommend podcasts about Math and Physics, not necessarily aimed at experts or professionals but maybe enlightened amateurs. Thank you. 190.220.104.35 (talk) 20:44, 17 November 2008 (UTC)[reply]

Well, Physics podcasts I don't know of, but usberkeley on youtube has physics lectures from 2007. I hope that helps 66.216.163.92 (talk) 23:39, 17 November 2008 (UTC)[reply]

type of screen

Does anyone know what technology is used for these station displays [20], [21]? They don't look like the traditional LED or plasma displays. And if possible can a link also be provided to the wikipedia article which explains the technology. Thanks in advance. Clover345 (talk) 20:54, 17 November 2008 (UTC)[reply]

I would guess they are digital rear projection screens utilizing DLP technology. They are large and bright and relatively cheap, but do have the disadvantages of being energy hogs, getting hot, and burning through bulbs. StuRat (talk) 03:50, 18 November 2008 (UTC)[reply]

Buoyancy

What causes the buoyancy force? The article doesn't really explain the origin of the force, just the equations used. —Preceding unsigned comment added by 76.69.241.185 (talk) 23:32, 17 November 2008 (UTC)[reply]

The medium (usually liquid) pushes on the object from all sides, but presses slightly less on the top than on the bottom because the bottom in this case is at higher pressure. If you push less on the top than on the bottom, the net force is up. SDY (talk) 00:11, 18 November 2008 (UTC)[reply]
Our buoyancy article does explain this in more or less those exact words. Because pressure increases with depth - the upward pressure on the underside of the object must be more than the pressure pushing down on the upper side because of the thickness of the object. Since the total upward facing area must equal the total downward facing area (measured at right angles to the gravitational field) - there must be a net upward force. The math falls out pretty simply from there. I suppose if you want the ULTIMATE source of the force - it's pressure - which is molecules in the fluid bouncing off of the object, imparting little bits of kinetic energy. But they bounce because of the electromagnetic repulsion between the outer electrons of the molecules - so that's a layer of explanation below that. SteveBaker (talk) 00:32, 18 November 2008 (UTC)[reply]
The explanations above do not explain why a stone doesn't float. And if an object does float in water, why doesn't it go on rising above the water, up into the air to the stratosphere, for the reasons given. (Air pressure on the bottom of the object is greater than air pressure on the top.) In fact, a submerged object that floats to the surface would leap into the air when the top of it leaves the water. (Directly below the surface, water pressure will not be greatly more than air pressure on the surface. But parts of the object that are deeper down will experience a rapidly increasing pressure with increasing depth because of the weight of the water around it. The pressure on the bottom of the object will be very much greater than the pressure on the top of the object, so it will leap out of the water, then continue to rise.) Actually, there will be an upward surge when the object breaks the surface, but it will not continue to rise into the stratosphere. Not many people can say "gotcha" to SteveBaker. —Preceding unsigned comment added by 98.16.67.220 (talk) 02:11, 18 November 2008 (UTC)[reply]
Well, that's because the difference in pressure in air is smaller than the difference in pressure in water, because water is denser than air...but I agree that the explanation doesn't make sense (in my mind, at least :)). See, wouldn't your explanation require that the shape of the object be significant. If the object was a sheet of something slightly denser than water with the same dimentions as a piece of paper, then in one orientation (long side perpendicular to the surface of the water) it might float because the bottom is much deeper than the bottom, while if rotated 90 degrees (so the long side is parallel to the surface) then it might sink for exactly the opposite reason. —Preceding unsigned comment added by 76.69.241.185 (talk) 02:21, 18 November 2008 (UTC)[reply]
Well, technically you have to think of it as billions of little molecules bouncing off the object. A sloped surface will transfer some down and some "right." In a cone, for example, all the "right" would be canceled by "left" and you'd get a net force that's either up or down. With an irregular object, it'll tumble as it rises or sinks. If you add up all of the vectors, though, you get a "down" force that's less than the "up" force. The main "enemy" is gravity (always "down" more or less by definition): if the net "up" force doesn't exceed that, the object sinks. Much easier to explain with a drawing, really. SDY (talk) 02:53, 18 November 2008 (UTC)[reply]
The reason a stone doesn't float - and most things don't float in air (although some clearly do) - is because the difference in pressure between the top and bottom doesn't exceed the weight of the object...so it still sinks (albeit a bit slower than it otherwise would have). My explanation makes it sound like a cone or something might behave wierdly - but you do have to read EXACTLY what I wrote. I said "Since the total upward facing area must equal the total downward facing area (measured at right angles to the gravitational field) - there must be a net upward force." - note the boldface here. A cone has the same area on the pointy end as the flat end when you measure the area at right angles to the gravitational field. As for your concern about a flat object and a flat object on its side: The surface area (at right angles to the gravitational field) of (say) a thin, flat, HORIZONTAL sheet of metal is large - but the pressure differential over it's skinny thickness is small. That small pressure differential multiplied by the large surface area (at right angles to the gravitational field) gives you the value for the buoyancy force. Turn the plate on it's side and the cross-sectional area (at right angles to the gravitational field) becomes very small indeed - but now it's much greater "height" increases the pressure differential between the 'top' and the 'bottom' edges of the plate. Crunch the math and you discover that the buoyancy of the plate remains the same no matter how you turn it around. In fact - it depends on the volume of the object - which is why we can neglect all of the complicated shape stuff and concentrate on the density of the object alone. Why an object breaking the surface doesn't leap up into the air is because the pressure on the bottom of the object depends on the depth of the water - which is now less than the thickness of the object - so even though the downward pressure has gone to more or less zero - the upward pressure is less than it was when the object was completely submerged - so as a floating object breaks the surface, the upward force gradually drops in strength until it exactly supports the weight of the object. SteveBaker (talk) 05:13, 18 November 2008 (UTC)[reply]

November 18

CMBR

The article on cosmic microwave background radiation says that the radiation is isotropic except for a small blueshift because we're moving relative to the surface of last scattering. But shouldn't the side of the surface closer to the center of the universe be much warmer because there are more photons per cubic meter on the less-expanded side than on the more-expanded outer side? 71.176.180.12 (talk) 02:03, 18 November 2008 (UTC)[reply]

I think the universe is supposed to be expanding uniformly everywhere. StuRat (talk) 03:44, 18 November 2008 (UTC)[reply]
This is precisely the reason why measuring the background microwave radiation was so important. The mistake you're making is the all-too-common one of assuming that the big bang happened over 'there' and we're currently over 'here'. In fact, at the moment of the big bang, all of space was scrunched up in a teeny-tiny dot - which expanded rapidly in all directions. So there is no one particular direction you can point to and say "the big bang was over there". The big bang filled all of space at the time it happened - but space itself was very tiny. So the big bang is in every direction around us - which is why the cosmic background looks the same temperature in all directions. That discovery is the single thing that proves that the universe started with a big bang and not in some other manner. SteveBaker (talk) 04:56, 18 November 2008 (UTC)[reply]

Theremin range?

What exactly is the playing range of the theremin? I've heard it stated as just slightly higher and lower than the cello, is this true? Kenjibeast (talk) 02:23, 18 November 2008 (UTC)[reply]

The Theremin can play a range from subsonic into supersonic if it is designed to do so. In fact, even a Theremin designed for only the audio range will probably extend beyond the audio range, unknown to the designer of it, if the electrical values of the components used allow that. Basically, it's just an audio oscillator. —Preceding unsigned comment added by 98.16.67.220 (talk) 02:35, 18 November 2008 (UTC)[reply]

Alright, let's talk the Moog etherwave model in particular. That one. What's that one's range. Kenjibeast (talk) 02:40, 18 November 2008 (UTC)[reply]

That would take some complicated figuring, taking into account every component in the Theremin including its speaker. An easier way is it to have a very good microphone pick up the sound and supply it to an oscilloscope. The oscilloscope will show the basic frequency of the sounds. The microphone would have to respond to subsonic and supersonic audio, if you are interested in that and if the Theremin will go that low and that high. Or a good musician can match the lowest and highest frequencies of the Theremin on a musical instrument and name the pitch he is playing. —Preceding unsigned comment added by 98.16.67.220 (talk) 03:25, 18 November 2008 (UTC)[reply]

The Etherwave's tone is produced in a circuit configuration called a beat frequency oscillator. It consists of two high-frequency oscillators, plus a detector circuit which extracts the difference frequency, or beat frequency. One of the highfrequency oscillators (called the fixed pitch oscillator) operates at about 285 kHz, while the other high-frequency oscillator (called the variable pitch oscillator) operates over a range of about 282 - 285 kHz. The difference frequency ranges from zero to about 3 kHz, which is three and a half octaves above middle C.

The pitch antenna circuit is connected to the variable pitch oscillator in such a way that increases in hand capacitance will decrease the variable pitch frequency as much as 3 kHz. This is how the pitch antenna circuit, in conjunction with the beat frequency oscillator circuit, enables the player to cover a usable pitch range of some five octaves (two octaves below to three octaves above middle C) simply by moving her right hand through a distance of two feet or so.

From here : UNDERSTANDING, CUSTOMIZING, AND HOT-RODDING YOUR ETHERWAVE ® THEREMIN. Hope this helps. APL (talk) 03:55, 18 November 2008 (UTC)[reply]

How can air barriers be permeable to vapor?

I just read the following:

The water vapor molecule is smaller than airs other molecules - nitrogen and oxygen. Therefore, water vapor can squeeze through smaller microscopic spaces than air.

How can this be true? It certainly isn't true in a space-filling model: If H-O-H fits through an opening, so will O-O, because the former has to be at least as wide as an O atom, and the latter can't be wider than that.

It is conceivable that the picture looks different when we regard the orbital model. However, I don't think the effect is favorable for water, which has big lobes sticking out of the O on the opposite side of the two H. Moreover, it has high van der Waals forces, which should, if anything, rather contribute to the water getting stuck in narrow openings. — Sebastian 04:03, 18 November 2008 (UTC)[reply]

I'm no chemist - but hydrogen atoms are very, very tiny compared to oxygen or nitrogen. Certainly the weight of H2O is a LOT less than O2 or N2. SteveBaker (talk) 04:50, 18 November 2008 (UTC)[reply]
Water creates strong hydrogen bonds. As such, I'd expect it would be harder to squeeze through. — DanielLC 04:58, 18 November 2008 (UTC)[reply]
Yup, that's what I meant by van der Waals forces. — Sebastian 10:00, 18 November 2008 (UTC)[reply]
EC:The size of atoms isn't necessarily simply additive in molecules, as they can overlap when they share electrons or they can have empty spaces inside the molecules as in a benzene ring, buckyball, or carbon nanotube. Also, it's not as simple as the size of the molecule and the size of the holes. The molecules of the gas and container may have various attractions for each other which make passage difficult or impossible. StuRat (talk) 05:06, 18 November 2008 (UTC)[reply]
In reply to both of you: The holes could of course be hydrophobic, but that wouldn't be the same argument as the one I quoted. So, can we conclude that that argument is humbug? — Sebastian 10:00, 18 November 2008 (UTC)[reply]
On second thought, hydrophobicity should not play a role, as it is, according to our pertinent article, only a consequence of the bonds among water molecules being stronger than those with the surface - which is not the case for vapor. So, the question is: How can air barriers be permeable to vapor? — Sebastian 10:11, 18 November 2008 (UTC)[reply]
For atoms and molecules, they aren't all going to line up, they are constantly tumbling and twisting, so its not the SMALLEST dimension that matters, its the LARGEST dimension. In the case of dioxygen, the longest dimension is larger than that of water vapor. Doing a quick search, the O-H bond length in water vapor is expected to be about 95 picometers, and in dioxygen its 121 picometers. Likewise, the atomic radius of hydrogen is 25 picometers and the atomic radius of oxygen is 60 pm. I know that these are not truly additive, but doing a fermi calculation or spherical cow type aproximation should at least show that the O=O size in dioxygen (60+121+60 = 241) is much larger than the O-H size in water (25+95+60 = 180). It turns out that at these scales, molar mass is as good of a first approximation to molecular volume as any; one generally only considers that when judging the "size" of a molecule; and under that test, dioxygen (32) is much larger than water (18). --Jayron32.talk.contribs 13:09, 18 November 2008 (UTC)[reply]

Advice for developing young scientists?

I am a recent graduate of microbiology at University of Texas at Austin. I want to plan a career in a relatively emerging and promising frontier of science, such as molecular genetics or cancer biology. However, the answer to these questions can be useful to any recent graduate.

What new and emerging field of science do you find have the biggest opportunity for future employment prospects i.e. what industry that is based on the science will always be in high demand? I personally had molecular genetics, cancer and developmental biology in mind.70.112.163.212 (talk) 05:15, 18 November 2008 (UTC)[reply]

Those all seem like jobs that will be in high demand, but there's another dimension you should consider. Even though such jobs may be in high demand, if they can be outsourced to someone for less pay in India, they will be, eventually (or you will need to take a drastic pay cut to compete with them). You need to find a job that can't possibly be outsourced, if you want a career for life. One that comes to mind is biowarfare, as defense related jobs can't be outsourced for security reasons. You might also be able to get some government job at the Atlanta Centers for Disease Control that would be somewhat secure. StuRat (talk) 05:35, 18 November 2008 (UTC)[reply]

What method of study besides job experience do you recommend for getting as up-to-date with that field as humanly possibly?70.112.163.212 (talk) 05:15, 18 November 2008 (UTC)[reply]

Reading trade magazines might help. If you can't find a mag specific to microbiology, you could go with some general science mags like Scientific American and Nature. Medical publications like the New England Journal of Medicine and JAMA might also be good (but beware that pharma companies routinely "plant" fake articles in those pubs to promote their products). StuRat (talk) 05:43, 18 November 2008 (UTC)[reply]

What books do you recommend for obtaining a deep "philosophical" and practical grasping of basic tenants of science, what is its role in our modern world, what customs exist in the profession, and how researchers carry out their methodology. For instance, what percentage of experiments are novel, and what percentage are simply an extrapolation and testing of ideas that were advanced by a single or select group of researchers? What makes a good scientist a good scientist?70.112.163.212 (talk) 05:15, 18 November 2008 (UTC)[reply]

Sidestepping for a moment, you could talk to your ex-professor or one in another university and ask what the gaps for research in your area are at the moment. E.O. Wilson the ant expert invented the word "sociobiology" afaik, and said if he had his life over, he would be a microbiologist. He's a cool role model as scientists go. I would start with him and anything he's written for iinspiration. Julia Rossi (talk) 05:49, 18 November 2008 (UTC)[reply]

@StuRat: You failed to take into account the government grants that go into R&D. Whenever the govt. needs to diversify (for GDP growth etc. or like the current credit crunch) it will spend more on R&D. --Movieideas (talk) 12:33, 18 November 2008 (UTC)[reply]

@OP: I would suggest you read this business article: Pharma struggles to adapt as patents expire http://www.reuters.com/article/Health08/idUSTRE4AF1QS20081117 --Movieideas (talk) 12:38, 18 November 2008 (UTC)[reply]

Resistance in space.

Does anyone know the resistance of outer space? Preferably as a function of distance from the Sun? I asked this as part of another question earlier. So far, the only answer I got was:

The resistance in space is pretty frigging huge. You need a series of actual atoms to transfer electrons along; and atoms are few and far between in space, so the resistance is likely, um, excuse the pun, "astronomically high". --Jayron32.talk.contribs 15:36, 17 November 2008 (UTC)

Let me continue from there. IIRC, space contains large numbers of ions and free electrons. This would cause the Earth's charge to slowly leak. This is quite different than how electricity normally works, so I don't know if the electrical conductance in space can be modeled the same, but I'm doing it on a planetary scale, so if there's anything slowing the charge down at all, it seems like the normal model might work. In addition, I'm dealing with a huge time-frame (5.5 billion years). Put simply, making a charge of 8e17 C once, and making it once every million years to keep it from going away, make the resulting difficulty in destroying the Earth differ by over three orders of magnitude. — DanielLC 05:26, 18 November 2008 (UTC)[reply]

Is the Earth also being constantly charged by the solar wind ? StuRat (talk) 06:42, 18 November 2008 (UTC)[reply]
Electromagnetic forces are vastly more powerful than gravity - if there was even a fraction of a volt difference in the charge on any of the planets or the sun, the solar system would be a very different place! If there is a 'charging' mechanism - then there must be a corresponding discharge mechanism...I kinda doubt that either are significant. SteveBaker (talk) 13:38, 18 November 2008 (UTC)[reply]

Tankers and aircraft carriers

In this morning's reports of the hijacking of the oil tanker Sirius Star, an American Navy official is quoted as saying that the tanker, "(is) three times the size of an aircraft carrier." The Sirius Star is 330 metres long, or 1082 feet. After some googling around, I can't find any type of aircraft carrier that is shorter than about 800 feet. It isn't too easy to compare the 'weights', as there seem to be deadweight tons, shortweight tons, metric tons, and displacement. In any case, is there any measure by which this supertanker can be fairly described as 'three times the size of an aircraft carrier'? Audacious pirates anyway... —Preceding unsigned comment added by 80.101.134.43 (talk) 09:28, 18 November 2008 (UTC)[reply]

The Sirius Star has a displacement of over 300,000 tons. A Nimitz class aircraft carrier (the largest aircraft carrier ever) displaces just over 100,000 fully loaded, according to its infobox. Clarityfiend (talk) 10:07, 18 November 2008 (UTC)[reply]

Selective breeding pros/cons

What are the pros and cons of selective breeding? is it ethical/unethical? Discuss —Preceding unsigned comment added by Fas327 (talkcontribs) 09:57, 18 November 2008 (UTC)[reply]

That little word 'discuss' signfies...homework. As noted at the top of the page we're not here to do your homework. If you want to consider the ethics of breeding etc. perhaps search for any pro or anti organisations mentioned in your text-books with regards to it. Also read through Selective breeding and the subsequent links that page will likely contain. 194.221.133.226 (talk) 10:42, 18 November 2008 (UTC)[reply]
Not to mention the rules agreed to when posting to the ref desks: "The reference desk does not answer requests for opinions or predictions about future events. Do not start a debate; please seek an internet forum instead." -- Aeluwas (talk) 12:35, 18 November 2008 (UTC)[reply]

Brain Tumor Cells vs. Normal Brain Cells

  • What is the difference between Brain Tumor Cells and Normal Brain Cells?
No, you can not use the tumor cells for thinking, as illustrated in this fascinating article about an awake craniotomy. --Sean 13:34, 18 November 2008 (UTC)[reply]

Where does the Earth's heat go?

This has puzzled me since I was a kid. The Earth is a closed system (more or less). The Sun constantly heats the Earth. So shouldn't the Earth be constantly getting hotter and hotter? Heat is the vibration of atoms and space is a vacuum. So where does all the heat go? 67.184.14.87 (talk) 14:08, 18 November 2008 (UTC)[reply]

It emits it into space as infra-red radiation. See thermal radiation. --Tango (talk) 14:20, 18 November 2008 (UTC)[reply]

is Jesus's 100% man/100% God dualty like quantum wave/particle dualty?

So Christians think that Jesus is completely, 100% a normal human being, he can die, everything about him is normal, he's just like you and me. At the SAME time, he's 100% God Himself. He's completely God. It's not that he's 20% Human and 80% God. He's completely a mortal human and completely immortal God. Obviously, this is ridiculously illogical and anyone who would believe that is off their rockers....I was thinking.

But wait: I remembered that reality itself has such a dualty! Light is 100% a wave, it interacts in wave patterns, by any standard you could measure it's completely a wave. But it's also a particle, it comes in photons, by any standard you could measure it comes in photons. So it seems you don't have to be ridiculously illogical to believe that, since our physical universe ACTUALLY has such a manifestation! But I'm not a quantum physicist so, could I ask:

Is my conclusion right? If Christians believe what I said about Jesus, is it comparable to what quantum physicists believe about light? Thank you!